Está en la página 1de 281

CIMA - PAPER P2

Performance Management
Exam Practice Kit

Tutor contact details

Tufal Choudhury
tufal@acornlive.com
07790 904122

1|Page
Paper P2

PERFORMANCE STRATEGY

Syllabus overview

While Paper P2 continues the analytic theme of Paper P1 Performance Operations (for
example in terms of identifying relevant costs), its main focus is on the application of
information in the management processes of decision-making and control, so as to
optimise performance. The first two sections deal respectively with the key contributors
to operational performance revenue (decisions of what to produce, at what price) and
costs (how to manage them to maximise profitability). The role of control in monitoring
and improving performance then comes to the fore in the final two sections, dealing with
principles and practices in the use of responsibility centres and budgeting.

Syllabus structure

The syllabus comprises the following topics and study weightings:

A Pricing and Product Decisions 30%


B Cost Planning and Analysis for Competitive Advantage 30%
C Budgeting and management Control 20%
D Control and Performance Measurement of Responsibility Centres 20%

Assessment strategy

There will be a written examination paper of three hours, plus 20 minutes of pre-
examination question paper reading time. During the 20 minutes you can: read the
question paper and annotate or highlight the question paper. However you will not be
allowed to: open the answer book; write in the answer book; add any loose
sheets/supplements to your answer book; or use calculators. Failure to comply with these
rules will be considered as a serious breach of the exam regulations.

2|Page
The examination paper will have the following sections.

Five compulsory medium answer questions, each worth ten


Section A 50 marks marks. Short scenarios may be given, to which some or all
questions relate.

One or two compulsory questions. Short scenarios may be


Section B 50 marks
given, to which questions relate.

Recommended study hours


Based on a study duration of 3-4 months, we recommend;

1 4 study sessions per week.


2 Each study session 1.5 to 2 hours.
3 Total hours per week between 6 to 8 hours.

The study time per week recommended, should be 80% question practice.

Exam strategy

Within your 20 minutes of official reading time allowed before the exam starts, read
through all requirements and information carefully. You can write on the exam paper
during this time so concentrate heavy on answer plans for the section A type questions,
look at Section B type questions only if you have time. You can write on the exam paper
during this time so produce brief outline answer plans and highlight key requirements and
scenario information.

You have 1.8 mins per mark (3 hours or 180 mins divided by 100 marks).
Allow 18 minutes per question for the five compulsory questions within section A
(total time 90 minutes).
Allow 90 minutes (the remaining time you have) to complete section B.

The examiner has said that it is unlikely section B will ever contain one compulsory
question and therefore more likely to contain two compulsory questions worth 25 marks
each.

3|Page
The section B questions are not just 45 minutes of time per question attempted; you have
to break down the 45 minutes of time allotted for each requirement before you begin.
This ensures you dont run out of time and this same principle should also be applied to
section A requirements.

Tackle all the easiest parts for numerical calculations and then attempt all the other
aspects of the requirement in the time you have. It is crucial that you manage your
time properly in the exam. Dont be afraid to stop calculating even though your answer
is not complete, otherwise over running will seriously prejudice other marks due to
inadequate time and dramatically reduce your chances of passing the exam.

Question requirements

Read them carefully.


Break down each requirement into headings and include and underline these
headings within your answer sheet.
Examine any information in the scenario for the question (if provided) constantly
to generate ideas to meet the requirements.
Ensure you deal with all aspects to a question requirement.
Always write a brief conclusion (sometimes additional marks are included in the
post exam guide if a candidate has does this).

Answer presentation

Page clearly labelled as to what part of the question you are completing
Start a fresh page for every part to a question.
Watch for write a report or memo etc.
Let the question requirement drive your headings.
Write in brief explanations 5-6 lines then a gap or white space every 5-6 lines.
Diagrams should be neat and spacious about 1/3 page in size.
Marks will be balanced according to the weighting of the syllabus.

Discussions

Make the relevant point.


Explain that point in as much detail as possible.
Relate to the scenario.
Give your recommendation where required.

4|Page
Learning objectives

Learning objectives are defined at the back of every CIMA exam on exam day, a copy of
the table below will be provided on exam day. Exam requirements will BOLD these
verbs below to help you decide the correct approach to a question. The common learning
verbs include Explain, Discuss, Describe, Identify and Construct. Familiarise yourself
with these commonly used verbs.

Knowledge List Make a list of


State Express, fully or clearly, the details/facts of
Define Give the exact meaning of

Comprehension Describe Communicate the key features of


Distinguish Highlight the differences between
Explain Make clear, state the meaning of
Identify Recognise, or select after consideration
Illustrate Use an example to describe or explain

Application Apply Put to practical use


Calculate/compute Ascertain mathematically
Demonstrate Prove with certainty by practical means
Prepare Make or get ready for use
Reconcile Prove consistent/compatible
Solve Find an answer to
Tabulate Arrange in a table

Analysis Analyse Examine in detail


Categorise Place in a defined class
Compare and contrast Show the similarities and/or differences
Construct between
Discuss Build up or compile
Interpret Examine in detail by argument
Produce Translate into familiar terms
Create or bring into existence

Evaluation Advise Counsel, inform or notify


Evaluate Appraise or assess the value of
Recommend Advise on the course of action

5|Page
CONTENTS

Section A type questions 10 marks each

Part A Pricing and Product Decisions

Relevant chapters from study manual 1, 3, 5 and 8

Question Title Exam year Question Answer


number page page
number number
A1 - 1 EXE (P2) Pilot Paper 13 103
A1 - 2 MNP (P2) May 2005 14 104
A1 3 VBJ (P2) May 2005 15 105
A1 4 QXY plc (P2) Nov 2006 19 106
A1 - 5 RST (P2) May 2007 19 108
A1 6 HS (P2) Nov 2007 20 109
A1 7 Bank charges (P2) May 2008 21 112
A1 8 WX May 2011 22 113

Section B type questions 25 marks each

Part A Pricing and Product Decisions

Relevant chapters from study manual 1, 3, 5 and 8

Question Title Exam year Question Answer


number page page
number number
A2 1 TQ (P2) Pilot Paper 24 117
A2 2 QP plc (P2) Nov 2005 25 121
A2 3 ZP plc (P2) Nov 2005 27 124
A2 4 AVX plc (P2) May 2006 29 127
A2 5 GHK (P2) May 2006 31 131
A2 6 H (P2) May 2007 33 137
A2 - 7 DFG (P2) Nov 2007 35 139
A2 8 Highly skilled workers (P2) May 2008 36 144
A2 9 RT May 2010 37 146
A2 10 LM Nov 2010 39 151
A2 11 Hotel May 2011 41 154

6|Page
Section A type questions 10 marks each

Part B Cost Planning and Analysis for Competitive Advantage

Relevant chapters from study manual 2, 4 and 6

Question Title Exam year Question Answer


number page page
number number
B1 1 SWAL (P2) Pilot Paper 44 157
B1 2 X group (P2) May 2005 44 159
B1 3 ML (P2) Nov 2005 45 160
B1 4 PK plc (P2) Nov 2005 46 161
B1 5 Financial advisors (P2) May 2006 46 163
B1 6 Compliance v conformance (P2) May 2006 47 165
B1 7 AVN (P2) Nov 2006 47 167
B1 8 W (P2) Nov 2006 48 170
B1 9 New product (P2) May 2007 49 171
B1 10 New small company (P2) Nov 2007 50 172
B1 11 XY (P2) May 2008 51 174
B1 12 Inventory levels (P2) May 2008 51 175
B1 13 Workshop (P2) May 2008 52 177
B1 14 Out-turn performace report May 2010 53 178
B1 15 PQ May 2010 54 180
B1 - 16 Timber products May 2010 54 181
B1 17 LMN May 2010 55 182
B1 18 Production manager Nov 2010 56 183
B1 19 CAL Nov 2010 57 185
B1 20 QW Nov 2010 58 186
B1 21 Accountancy services Nov 2010 58 187
B1 22 PT May 2011 59 188
B1 23 TQM and JIT (P1) Pilot Paper 60 189
B1 - 24 Standard costing (P1) May 2006 60 190
B1 - 25 Marginal v throughput (P1) May 2006 60 191
B1 26 MRPS (P1) May 2007 60 191
B1 27 JIT (P1) May 2007 60 192
B1 28 Key features of TQM (P1) May 2008 61 192

7|Page
Section B type questions 25 marks each

Part B Cost Planning and Analysis for Competitive Advantage

Relevant chapters from study manual 2, 4 and 6

Question Title Exam year Question Answer


number page page
number number
B2 1 The Q organisation (P2) May 2005 62 194
B2 2 F plc (P1) May 2005 63 198
B2 3 KL (P2) Nov 2006 64 202
B2 4 Retail outlet (P2) Nov 2007 65 206

8|Page
Section A type questions 10 marks each

Part C Budgeting and management Control

Relevant chapters from study manual 4

Question Title Exam year Question Answer


number page page
number number
C1 1 Solicitors firm May 2010 69 210
C1 - 2 DW Nov 2010 70 211
C1 3 JYT May 2011 70 212
C1 4 DVD May 2011 71 213
C1 5 SFG May 2011 72 215
C1 6 Feedback and feedforward (P1) Pilot Paper 72 216
C1 7 Profit centre managers (P1) Pilot Paper 72 217
C1 8 Balance scorecard (P1) May 2005 73 217
C1 9 Particiaption in budgets (P1) May 2005 73 219
C1 10 Beyond budgeting (P1) May 2005 73 220
C1 - 11 J Limited (P1) Nov 2005 74 221
C1 12 ST plc (P1) Nov 2005 74 222
C1 13 W Limited (P1) Nov 2005 74 223
C1 14 T plc (P1) May 2006 75 224
C1 - 15 Product M (P1) May 2007 75 225
C1 16 QBD (P1) Nov 2007 76 226
C1 - 17 Budgetary planning and control (P1) Nov 2007 77 227
C1 - 18 JIT systems (P1) Nov 2007 77 227
C1 19 Feedback and forward (P1) Nov 2007 77 228
C1 20 Nursing homes (P1) Nov 2007 77 228
C1 21 Participative budgeting (P1) May 2008 78 230
C1 22 Rolling budgets (P1) May 2008 78 231

9|Page
Section B type questions 25 marks each

Part C Budgeting and management Control

Relevant chapters from study manual 4

Question Title Exam year Question Answer


number page page
number number
C2 1 M plc (P1) May 2006 79 232
C2 2 RF Ltd (P1) May 2007 80 236
C2 3 Trackit (P1) May 2008 82 240
C2 4 X plc (P1) Nov 2006 85 244

Section A type questions 10 marks each

Part D Control and Performance Measurement of Responsibility Centres

Relevant chapters from study manual 9

Question Title Exam year Question Answer


number page page
number number
D1 1 EVA and RI (P1) Pilot Paper 87 248
D1 2 Controllability principle (P1) Pilot Paper 87 249
D1 3 Transfer pricing (P1) Pilot Paper 87 249
D1 4 EVA (P1) May 2005 87 251
D1 5 WD, PD & TD (P1) May 2005 87 252
D1 6 G group (P1) May 2007 88 254
D1 7 Digital equipment (P1) May 2008 89 255

10 | P a g e
Section B type questions 25 marks each

Part D Control and Performance Measurement of Responsibility Centres

Relevant chapters from study manual 9

Question Title Exam year Question Answer


number page page
number number
D2 1 Y and Z (P1) Nov 2005 90 257
D2 2 FP (P1) May 2006 91 262
D2 3 ZZ group (P1) Nov 2006 93 266
D2 4 Computer manufacturer (P1) Nov 2007 95 269
D2 5 Perfumes and cosmetics May 2010 96 273
D2 6 SWZ Nov 2010 99 276
D2 7 DE company May 2011 100 279

11 | P a g e
Mock exams to be submitted
Once all chapters have been completed, four mock exams are to be
attempted under timed conditions and sent to the address Tufal Choudhury,
57 Toms Lane, Kings Langley WD4 8NA. Please ensure you put your name
and address on your mock exam when submitting by post. Alternatively
scanned scripts by e-mail can be sent directly to tufal@acornlive.com

You should allow 20 minutes reading time before you begin and then 3
hours maximum to complete each mock exam. Make sure you have a
strategy and plan for how to manage your time.

Mock One
P2 Specimen (2010) exam paper

Mock Two
P2 September 2010 exam paper

Mock Three
P2 March 2011 exam paper

Mock Four
P2 September 2011 exam paper

Mock Five
P2 November 2011 exam paper

The above exam questions and solutions can be downloaded via your
mycima account which contains access to all past exam papers and
solutions, alternatively contact tufal@acornlive.com if you would like to
receive PDF copies.

12 | P a g e
Questions Section A

Part A Pricing and Product Decisions

A1 -1 EXE (CIMA P2 Pilot paper 2005)

You have received a request from EXE to provide a quotation for the manufacture of a
specialised piece of equipment. This would be a one-off order, in excess of normal
budgeted production. The following cost estimate has already been prepared:

Note $
Direct materials:
Steel 10m2 @ $5.00 per m2 1 50
Brass fittings 2 20

Direct labour:
Skilled 25 hours @ $8.00 per hour 3 200
Semi-skilled 10 hours @ $5.00 per hour 4 50

Overhead 35 hours @ $10.00 per hour 5 350

Estimating time 6 100


770
Administration overhead @ 20% of production cost 7 154
924

Profit @ 25% of total cost 8 231

Selling price 1,155

Notes:
1. 1 The steel is regularly used, and has a current stock value of $5.00 per square
metre. There are currently 100 square metres in stock. The steel is readily
available at a price of $5.50 per square metre.

2. The brass fittings would have to be bought specifically for this job: a supplier has
quoted the price of $20 for the fittings required.

3. The skilled labour is currently employed by your company and paid at a rate of
$8.00 per hour. If this job were undertaken it would be necessary either to work
25 hours overtime, which would be paid at time plus one half, OR in order to
carry out the work in normal time, reduce production of another product that earns
contribution of $13.00 per hour.

4. The semi-skilled labour currently has sufficient paid idle time to be able to
complete this work.

13 | P a g e
5. The overhead absorption rate includes power costs which are directly related to
machine usage. If this job were undertaken, it is estimated that the machine time
required would be ten hours. The machines incur power costs of $0.75 per hour.
There are no other overhead costs that can be specifically identified with this job.

6. The cost of the estimating time is that attributed to the four hours taken by the
engineers to analyse the drawings and determine the cost estimate given above.

7. It is company policy to add 20% to the production cost as an allowance for


administration costs associated with the jobs accepted.

This is the standard profit added by your company as part of its pricing policy.

Required:

Prepare on a relevant cost basis, the lowest cost estimate that could be used as the basis
for a quotation. Explain briefly your reasons for using EACH of the values in your
estimate.
(12 marks)

A1 2 MNP (CIMA P2 May 2005)

Z manufactures three joint products (M, N and P) from the same common process. The
following process account relates to the common process last month and is typical of the
monthly results of operating this process:

Common Process Account

Litres $ Litres $
Opening work in process 1,000 5,320 Normal loss 10,000 20,000
Materials 100,000 250,000 Output M 25,000 141,875
Conversion costs: Output N 15,000 85,125
Variable 100,000 Output P 45,000 255,375
Fixed 180,000 Closing WIP 800 3,533
Abnormal loss 5,200 29,412
101,000 535,320 101,000 535,320

Each one of the products can be sold immediately after the common process, but each
one of them can be further processed individually before being sold. The following
further processing costs and selling prices per litre are expected:

14 | P a g e
Product Selling price after Selling price after Further variable
common process further processing processing cost
$/litre $/litre $/litre
M 625 840 175
N 520 645 095
P 680 745 085

Required:

(a) State the method used to apportion the common costs between the products M, N and
P and comment on its acceptability. Explain why it is necessary to apportion the common
costs between each of the products.
(5 marks)

(b) Evaluate the viability of the common process, and determine the optimal processing
plan for each of the three products showing appropriate calculations.
(5 marks)

(Total = 10 marks)

A1 3 VBJ (CIMA P2 May 2005)

The CS group is planning its annual marketing conference for its sales executives and has
approached the VBJ Holiday company (VBJ) to obtain a quotation.

VBJ has been trying to win the business of the CS group for some time and is keen to
provide a quotation which the CS group will find acceptable in the hope that this will lead
to future contracts.

The manager of VBJ has produced the following cost estimate for the conference:

$
Coach running costs 2,000
Driver costs 3,000
Hotel costs 5,000
General overheads 2,000
Sub total 12,000

Profit (30%) 3,600


Total 15,600

You have considered this cost estimate but you believe that it would be more appropriate
to base the quotation on relevant costs. You have therefore obtained the following further
information:

15 | P a g e
Coach running costs represent the fuel costs of $1,500 plus an apportionment of the
annual fixed costs of operating the coach. No specific fixed costs would be incurred if the
coach is used on this contract. If the contract did not go ahead, the coach would not be in
use for eight out of the ten days of the conference. For the other two days a contract has
already been accepted which contains a significant financial penalty clause. This contract
earns a contribution of $250 per day. A replacement coach could be hired for $180 per
day.

Driver costs represent the salary and related employment costs of one driver for 10 days.
If the driver is used on this contract the company will need to replace the driver so that
VBJ can complete its existing work. The replacement driver would be hired from a
recruitment agency that charges $400 per day for a suitably qualified driver.

Hotel costs are the expected costs of hiring the hotel for the conference.

General overheads are based upon the overhead absorption rate of VBJ and are set
annually when the company prepares its budgets. The only general overhead cost that can
be specifically identified with the conference is the time that has been spent in
considering the costs of the conference and preparing the quotation. This amounted to
$250.

Required:

Prepare a statement showing the total relevant cost of the contract. Explain clearly the
reasons for each of the values in your quotation and for excluding any of the costs (if
appropriate).
(10 marks)

16 | P a g e
A1 4 QXY plc (CIMA P2 Nov 2006)

You are the Assistant Management Accountant of QXY plc, a food manufacturer. The
Board of Directors is concerned that its operational managers may not be fully aware of
the importance of understanding the costs incurred by the business and the effect that this
has on their operational decision making. In addition, the operational managers need to
be aware of the implications of their pricing policy when trying to increase the volume of
sales.

You are scheduled to make a presentation to the operational managers tomorrow to


explain to them the different costs that are incurred by the business, the results of some
research that has been conducted into the implications for pricing and the importance of
understanding these issues for their decision making. The diagram on the opposite page
has already been prepared for the presentation.

Required:

You are required to interpret the diagram and explain how it illustrates issues that the
operational managers should consider when making decisions. (Note: your answer
mustinclude explanations of the Sales Revenue, Total Cost and Fixed Cost lines, and the
significance of each of the activity levels labelled A, B, C, D).
(10 marks)

17 | P a g e
Diagram showing costs and revenues over a range of activity levels

A1 5 Z (CIMA P2 May 2007)

18 | P a g e
A1 5 RST (CIMA P2 May 2007)

Z is one of a number of companies that produce three products for an external market.
The three products, R, S and T may be bought or sold in this market. The common
process account of Z for March 2007 is shown below:

Kg $ Kg $
Inputs:
Material A 1,000 3,500 Normal loss 500 0
Material B 2,000 2,000 Outputs:
Material C 1,500 3,000 Product R 800 3,500
Direct labour 6,000 Product S 2,000 8,750
Variable overhead 2,000 Product T 1,200 5,250
Fixed cost 1,000
Totals 4,500 17,500 4,500 17,500

Z can sell products R, S or T after this common process or they can be individually
further processed and sold as RZ, SZ and TZ respectively. The market prices for the
products at the intermediate stage and after further processing are:

Market prices per


kg:

$
R 3.00
S 5.00
T 3.50
RZ 6.00
SZ 5.75
TZ 6.75

The specific costs of the three individual further processes are:


Process R to RZ variable cost of $1.40 per kg, no fixed costs
Process S to SZ variable cost of $0.90 per kg, no fixed costs
Process T to TZ variable cost of $1.00 per kg, fixed cost of $600 per month

Required:

(a) Produce calculations to determine whether any of the intermediate products should
be further processed before being sold. Clearly state your recommendations together with
any relevant assumptions that you have made.
(3 marks)

19 | P a g e
(b) Produce calculations to assess the viability of the common process:

(i) assuming that there is an external market for products R, S and T; and
(ii) assuming that there is not an external market for products R, S and T.

State clearly your recommendations.


(7 marks)

(Total = 10 marks)

A1 6 HS (CIMA P2 Nov 2007)

HS manufactures components for use in computers. The business operates in a highly


competitive market where there are a large number of manufacturers of similar
components. HS is considering its pricing strategy for the next twelve weeks for one of
its components. The Managing Director seeks your advice to determine the selling price
that will maximise the profit to be made during this period.

You have been given the following data:

Market Demand
The current selling price of the component is $1,350 and at this price the average weekly
demand over the last four weeks has been 8,000 components. An analysis of the market
shows that for every $50 increase in selling price the demand reduces by 1,000
components per week. Equally, for every $50 reduction in selling price the demand
increases by 1,000 components per week.

Costs
The direct material cost of each component is $270. This price is part of a fixed price
contract with the material suppliers and the contract does not expire for another year.

Production labour and conversion costs, together with other overhead costs and the
corresponding output volumes, have been collected for the last four weeks and they are as
follows:

Week Output volume (units) $000


1 9,400 7,000
2 7,600 5,688
3 8,500 6,334
4 7,300 5,446

No significant changes in cost behaviour are expected over the next twelve weeks.

20 | P a g e
Required:

(a) Calculate the optimum (profit maximising) selling price of the component for the
period.

Note: If Price = a - bq then Marginal Revenue = a - 2bq


(6 marks)

(b) Identify and explain two reasons why it may be inappropriate for HS to use this
theoretical pricing model in practice.
(4 marks)

(Total = 10 marks)

A1 7 Bank charges (CIMA P2 May 2008)

A bank is reviewing the bank account it offers to its business customers and the charges it
makes for routine transactions (for example paying into the account, writing cheques,
making electronic payments and transfers). Currently, the banks charges to its business
customers are 060 per routine transaction. The bank pays interest to the customer at
01% per year on any balance in the account.

According to the banks records, there are currently one million business customers. Each
customer makes one thousand routine transactions each year; 45% of business customers
maintain an average balance of 2,000 in their account. The accounts of the other 55% of
business customers are overdrawn with an average overdraft balance of 4,000. Interest
on overdrawn accounts is charged at 20% per year.

In addition, the bank has a number of savings account customers which, together with the
banks business customers, result in a balance of net funds that are invested by the bank
and yield an annual return by 3% per year.

The bank is concerned about a growing tendency for its competitors to provide routine
transactions free of charge to their business customers. As a result the bank is considering
twoaccount options:

Account Option One


An account that charges the business customer a fixed fee of 10 per month, with no
further charges for any routine transactions. Interest would be paid to the business
customer at 05% per year on any balances in the account. The bank expects that if it
adopts this charging structure, it will increase the number of business customers by
5%from its present level;

21 | P a g e
Account Option Two
An account that does not charge the customer for any routine transactions, but pays no
interest on any balances in the account. The bank expects that if it adopts this charging
structure, this will increase the number of business customers by 10% from its present
level.

The bank does not expect the profile of new business customers to be different from
existing business customers in terms of the balances in their accounts or the number of
routine transactions they make. Interest will continue to be charged at 20% per year on
overdrawn accounts. The bank does not expect that either of these options will result in
any changes to its existing staffing or other resources.

The bank also expects that if it takes no action and continues with its existing bank
account that the number of business customers will fall by 20%.

Required:

(a) Recommend which course of action the bank should take by preparing calculations to
show the annual profits from:

(i) continuing with the existing bank account


(ii) each of the two account options described above.

(12 marks)

A1 8 WX (CIMA P2 May 2011)

WX is reviewing the selling price of one of its products. The current selling price of the
product is $25 per unit and annual demand is forecast to be 150,000 units at this price.
Market research indicates that the level of demand would be affected by any change in
the selling price. Detailed analysis from this research shows that for every $1 increase in
selling price, annual demand would reduce by 25,000 units and that for every $1 decrease
in selling price, annual demand would increase by 25,000 units.

A forecast of the annual costs that would be incurred by WX in respect of this product at
differing activity levels is as follows:

Annual production (units) 100,000 160,000 200,000

$000 $000 $000


Direct materials 200 320 400
Direct labour 600 960 1,200
Overhead 880 1,228 1,460

22 | P a g e
The cost behaviour patterns represented in the above forecast will apply for the whole
range of output up to 300,000 units per annum of this product.

Required:

(a)
(i) Calculate the total variable cost per unit.
(2 marks)

(ii) Calculate the selling price of the product that will maximise the companys profits.
(4 marks)

Note: If Price (P) = a - bx then Marginal Revenue = a - 2bx

(b) Explain TWO reasons why the company might decide NOT to use this optimum
selling price.
(4 marks)

(Total = 10 marks)

23 | P a g e
Questions Section B

Part A Pricing and Product Decisions

A2-1 TQ (CIMA P2 Pilot Paper 2005)

(a) TQ manufactures and retails second generation mobile (cell) phones. The following
details relate to one model of phone:

$/unit
Budgeted selling price 60
Budgeted variable cost 25
Budgeted fixed cost 10

Period 1 2 3
Budgeted production and sales (units) 520 590 660

Fixed overhead volume variance $1,200 (A) $1,900 (A) $2,600 (A)

There was no change in the level of stock during any of periods 1 to 3.

The Board of Directors had expected sales to keep on growing but, instead, they appeared
to have stabilised. This has led to the adverse fixed overhead volume variances. It is now
the start of period 4 and the Board of Directors is concerned at the large variances that
have occurred during the first three periods of the year. The Sales and Marketing Director
has confirmed that the past trend of sales is likely to continue unless changes are made to
the selling price of the product. Further analysis of the market for the mobile phone
suggests that demand would be zero if the selling price was raised to $100 or more.

Required:

(i) Calculate the price that TQ should have charged for the phone assuming that it wished
to maximise the contribution from this product.

Note: If price = a bx then marginal revenue = a 2bx


(7 marks)

(ii) Calculate the difference between the contribution that would have been earned at the
optimal price and the actual contribution earned during period 3, assuming the variable
costs per unit were as budgeted.
(3 marks)

24 | P a g e
(b) TQ is currently developing a third generation mobile phone. It is a state of the art
new handheld device that acts as a mobile phone, personal assistant, digital camera
(pictures and video), and music player. The Board of Directors seeks your advice as to
the pricing strategy that it should adopt for such a product. The company has incurred a
significant level of development costs and recognises that the technology for these
products is advancing rapidly and that the life cycle for the product is relatively short.

Required:

Prepare a report, addressed to the Board of Directors that discusses the alternative pricing
strategies available to TQ.
(15 marks)

(Total = 25 marks)

A2 2 QP plc (CIMA P2 Nov 2005)

QP plc is a food processing company that produces pre-prepared meals for sale to
consumers through a number of different supermarkets. The company specialises in three
particular pre-prepared meals and has invested significantly in modern manufacturing
processes to ensure a high quality product. The company is very aware of the importance
of training and retaining high quality staff in all areas of the company and, in order to
ensure their production employees commitment to the company, the employees are
guaranteed a weekly salary that is equivalent to their normal working hours paid at their
normal hourly rate of 7 per hour.

The meals are produced in batches of 100 units. Costs and selling prices per batch are as
follows:

Meal TR PN BE
/batch /batch /batch

Selling Price 340 450 270

Ingredient K (5/kg) 150 120 90


Ingredient L (10/kg) 70 90 40
Ingredient M (15/kg) 30 75 45

Labour (7/hour) 21 28 42

Factory costs absorbed 20 80 40

QP plc has adopted throughput accounting for its short-term decisions.

25 | P a g e
Required:

(a) State the principles of throughput accounting and the effects of using it for short-term
decision making.
(6 marks)

(b) QP plc is preparing its production plans for the next three months and has estimated
the maximum demand from its customers to be as follows:

TR 500 batches
PN 400 batches
BE 350 batches

These demand maximums are amended figures because a customer has just delayed its
request for a large order and QP has unusually got some spare capacity over the next
three months. However, these demand maximums do include a contract for the delivery
of 50 batches of each to an important customer. If this minimum contract is not satisfied
then QP plc will have to pay a substantial financial penalty for non-delivery.

The Production Director is concerned at hearing news that two of the ingredients used are
expected to be in short supply for the next three months. QP plc does not hold inventory
of these ingredients and although there are no supply problems for ingredient K, the
supplies of ingredients L and M are expected to be limited to:

Ingredient L 7,000 kilos


Ingredient M 3,000 kilos

The Production Director has researched the problem and found that ingredient V can be
used as a direct substitute for ingredient M. It also costs the same as ingredient M. There
is an unlimited supply of ingredient V.

Required:

Prepare calculations to determine the production mix that will maximise the profit of QP
plc during the next three months.
(10 marks)

(c) The World Health Organisation has now announced that ingredient V contains
dangerously high levels of a chemical that can cause life-threatening illnesses. As a
consequence it can no longer be used in the production of food.

26 | P a g e
As a result, the production director has determined the optimal solution to the
companysproduction mix problem using linear programming. This is set out below:

Objective function value 110,714


TR value 500
PN value 357
BE value 71
TR slack value 0
PN slack value 43
BE slack value 279
L value 3
M value 28

Required:

Explain the meaning of each of the values contained in the above solution.
(9 marks)

(Total = 25 marks)

A2 3 ZP plc (CIMA P2 Nov 2007)

ZP plc is a marketing consultancy that provides marketing advice and support to small
and medium sized enterprises. ZP plc employs 4 full time marketing consultants who
each expect to deliver 1,500 chargeable hours per year and each receive a salary of
60,000 per year. In addition the company employs 6 marketing support/administration
staff whose combined total salary cost is 120,000 per year.

ZP plc has estimated its other costs for the coming year as follows:
000

Office premises: rent, rates, heating 50

Advertising 5

Travel to clients 15

Accommodation whilst visiting clients 11

Telephone, fax, communications 10

27 | P a g e
ZP plc has been attributing costs to each client (and to the projects undertaken for them)
by recording the chargeable hours spent on each client and using a single cost rate of 75
per chargeable hour. The same basis has been used to estimate the costs of a project when
preparing a quotation for new work.

ZP plc has reviewed its existing client database and determined the following three
average profiles of typical clients:

Client profile D E F
Chargeable hours per client 100 700 300

Distance (miles) to client 50 70 100

Number of visits per client 3 8 3

Number of clients in each profile 10 5 5

The senior consultant has been reviewing the companys costing and pricing procedures.
He suggests that the use of a single cost rate should be abandoned and, where possible,
activities should be costed individually. With this is mind he has obtained the following
further information:

It is ZP plcs policy that where a visit is made to a client and the distance to the
client is more than 50 miles, the consultant will travel the day before the visit and
stay in local accommodation so that the maximum time is available for meeting
the client the following day.

The cost of travel to the client is dependent on the number of miles travelled to
visit the client.

Other costs are facility costs at present the senior consultant cannot identify an
alternative basis to that currently being used to attribute costs to each client.

Required:

(a) Prepare calculations to show the cost attributed to each client group using an
activity based system of attributing costs.
(7 marks)

(b) Discuss the differences between the costs attributed using activity based costing
and those attributed by the current system and adviser whether the senior
consultants suggestion should be adopted.
(9 marks)

28 | P a g e
(c) In a manufacturing environment activity based costing often classifies activities
into those that are: unit; batch; product sustaining; and facility sustaining.
Discuss, giving examples, how similar classifications may be applied to the use of
the technique in consultancy organisations such as ZP plc.
(9 marks)

(Total = 25 marks)

A2 4 AVX plc (CIMA P2 May 2006)

AVX Plc assembles circuit boards for use by high technology audio video companies.
Due to the rapidly advancing technology in this field, AVX Plc is constantly being
challenged to learn new techniques.

AVX Plc uses standard costing to control its costs against targets set by senior managers.
The standard labour cost per batch of one particular type of circuit board (CB45) is set
out below:

Direct labour - 50 hours @ 10 /hour 500

The following labour efficiency variances arose during the first six months of the
assembly of CB45:

Month Number of batches Labour Efficiency


assembled and sold Variance ()
November 1 Nil
December 1 170.00 Favourable
January 2 452.20 Favourable
February 4 1,089.30 Favourable
March 8 1,711.50 Favourable
April 16 3,423.00 Favourable

An investigation has confirmed that all of the costs were as expected except that there
was a learning effect in respect of the direct labour that had not been anticipated when the
standard cost was set.

Required:

(a)
(i) Calculate the monthly rates of learning that applied during the six months;
(ii) Identify when the learning period ended and briefly discuss the implications of
your findings for AVX Plc.
(10 marks)

29 | P a g e
AVX Plc initially priced each batch of CB45 circuit boards on the basis of its standard
cost of 960 plus a mark up of 25%. Recently the company has noticed that, due to
increasing competition, it is having difficulty maintaining its sales volume at this price.

The Finance Director has agreed that the long run unit variable cost of the CB45 circuit
board is 672.72 per batch. She has suggested that the price charged should be based on
an analysis of market demand. She has discovered that at a price of 1,200 the demand is
16 batches per month, for every 20 reduction in selling price there is an increase in
demand of 1 batch of CB45 circuit boards, and for every 20 increase in selling price
there is a reduction in demand of 1 batch.

Required:

(b) Calculate the profit maximising selling price per batch using the data supplied by the
Finance Director
(8 marks)

Note: If Price (P) = a-bx then Marginal Revenue (MR) = a-2bx

The Technical Director cannot understand why there is a need to change the selling price.
He argues that this is a highly advanced technological product and that AVX Plc should
not reduce its price as this reflects badly on the company. If anything is at fault, he
argues, it is the use of Standard Costing and he has asked whether Target Costing should
be used instead.

Required:

(c)
(i) Explain the difference between standard costs and target costs;
(ii) Explain the possible reasons why AVX Plc needs to re-consider its pricing policy
now that the CB45 circuit board has been available in the market for six months.
(7 marks)

(Total = 25 marks)

30 | P a g e
A2 5 GHK (CIMA P2 May 2006)

GHK manufactures four products from different combinations of the same direct
materials and direct labour. An extract from the flexible budgets for next quarter for each
of these products is as follows :

Product G H J K
Units 3,000 5,000 3,000 5,000 3,000 5,000 3,000 5,000
$000 $000 $000 $000 $000 $000 $000 $000
Revenue 30 50 60 100 45 75 90 150
Direct Material A 9 15 12 20 4.5 7.5 18 30
(note 1)
Direct Material B 6 10 6 10 13.5 22.5 36 60
(note 2)
Direct labour 6 10 24 40 22.5 37.5 9 15
(note 3)
Overhead 6 8 13 19 11 17 11 17
(note 4)

Notes

1. Material A was purchased some time ago at a cost of $5 per kg. There are 5,000
kgs in inventory. The costs shown in the flexible budget are based on this
historical cost. The material is in regular use and currently has a replacement cost
of $7 per kg.
2. Material B is purchased as required ; its expected cost is $10 per kg. The costs
shown in the flexible budget are based on this expected cost.
3. Direct labour costs are based on an hourly rate of $10 per hour. Employees work
the number of hours necessary to meet production requirements.
4. Overhead costs of each product include a specific fixed cost of $1,000 per quarter
which would be avoided if the product was to be discontinued. Other fixed
overhead costs are apportioned between the products but are not affected by the
mix of products manufactured.

GHK has been advised by the only supplier of material B that the quantity of material B
that will be available during the next quarter will be limited to 5,000 kgs. Accordingly the
company is being forced to reconsider its production plan for the next quarter. GHK has
already entered into contracts to supply one of its major customers with the following :

500 units of product G


1,600 units of product H
800 units of product J
400 units of product K

31 | P a g e
Apart from this, the demand expected from other customers is expected to be

3,600 units of product G


3,000 units of product H
3,000 units of product J
4,000 units of product K

The major customer will not accept partial delivery of the contract and if the contract
with this major customer is not completed in full, then GHK will have to pay a financial
penalty of $5,000.

Required :

(a) For each of the four products, calculate the relevant contribution per $ of material B
for the next quarter.
(6 marks)

(b) It has been determined that the optimum production plan based on the data above is
to produce 4,100 units of product G, 4600 units of product H, 800 units of product J, and
2,417 units of product K. Determine the amount of financial penalty at which GHK
would be indifferent between meeting the contract or paying the penalty.
(5 marks)

(c) Calculate the relevant contribution to sales ratios for each of the four products.
(2 marks)

(d) Assuming that the limiting factor restrictions no longer apply, prepare a sketch of a
multi product profit volume chart by ranking the products according to your contribution
to sales ratio calculations based on total market demand. Your sketch should plot the
products using the highest contribution to sales ratio first.
(6 marks)

(e) Explain briefly, stating any relevant assumptions and limitations, how the multi
product profit volume chart that you prepared in (d) above may be used by the manager
of GHK to understand the relationships between costs, volume and profit within the
business.
(6 marks)

(Total = 25 marks)

32 | P a g e
A2 6 H (CIMA P2 May 2007)

H, a printing company, uses traditional absorption costing to report its monthly profits. It
is seeking to increase its business by winning work from new customers. It now has
theopportunity to prepare a quotation for a large organisation that currently requires a
new catalogue of its services.

A technical report on the resource requirements for the catalogues has been completed at
a cost of $1,000 and its details are summarised below:

Production period
It is expected that the total time required to print and despatch the catalogue will be one
week.

Material A
10,000 sheets of special printing paper will be required. This is a paper that is in regular
use by H and the company has 3,400 sheets in inventory. These originally cost $1.40 per
sheet but the current market price is $150 per sheet. The resale price of the sheets held in
inventory is $1.20 per sheet.

Material B
This is a special ink that H will need to purchase at a cost of $8 per litre. 200 litres will be
required for this catalogue but the supplier has a minimum order size of 250 litres. H does
not foresee any other use for this ink, but will hold the surplus in inventory. Hs inventory
policy is to review slow moving items regularly. The cost of any inventory item that has
not been used for more than 6 months is accounted for as an expense of the period in
which that review occurs.

Direct labour
Sufficient people are already employed by H to print the catalogue, but some of the
printing will require overtime working due to the availability of a particular machine that
is used on other work. The employees are normally paid $8 per hour, the order will
require 150 hours of work and 50 of these hours will be in excess of the employees
normal working week. A rate of $10 per hour is paid for these overtime hours.
Employees are paid using an hourly rate with a guaranteed minimum wage for their
normal working week.

Supervision
An existing supervisor will take responsibility for the catalogue in addition to her existing
duties. She is not currently fully employed and receives a salary of $500 per week.

33 | P a g e
Machinery
Two different types of machine will be required:

Machine A will print the catalogues. This is expected to take 20 hours of machine time.
The running cost of machine A is $5 per hour. There is currently 30 hours of unused time
on machine A per week that is being sold to other printers for $12 per hour.

Machine B will be used to cut and bind the catalogues. This machine is being used to full
capacity in the normal working week and this is why there is a need to work overtime.
The catalogue will require 25 machine hours and these have a running cost of $4 per
hour.

Despatch
There will be a delivery cost of $400 to transport the catalogues to the customer.

Fixed overhead costs


H uses a traditional absorption costing system to attribute fixed overhead costs to its
work. The absorption rate that it uses is $20 per direct labour hour.

Profit mark-up
H applies a 30% mark-up to its costs to determine its selling prices.

Required:

(a) In order to assist the management of H in preparing its quotation, prepare a schedule
showing the relevant costs for the production of the catalogues. State clearly your reason
for including or excluding each value that has been provided in the above scenario.
(15 marks)

(b) Explain how the use of relevant costs as the basis of setting a selling price may be
appropriate for short-term pricing decisions but may be inappropriatefor long-term
pricing decisions. Your answer should also discuss the conflict between reporting
profitability within a traditional absorption costing system and the use of relevant cost
based pricing.
(10 marks)

(Total = 25 marks)

34 | P a g e
A2 7 DFG (CIMA P2 Nov 2007)

DFG manufactures two products from different combinations of the same resources. Unit
selling prices and unit cost details for each product are as follows:

Product D /unit G /unit


Selling price 115 120

Direct material A (5 per kg) 20 10


Direct material B (3 per kg) 12 24
Skilled labour (7 per hour) 28 21
Variable overhead (2 per machine hour) 14 18
Fixed overhead* 28 36

Profit 13 11

*Fixed overhead is absorbed using an absorption rate per machine hour. It is an


unavoidable central overhead cost that is not affected by the mix or volume of products
produced.

The maximum weekly demand for products D and G is 400 units and 450 units
respectively and this is the normal weekly production volume achieved by DFG.
However, for the next four weeks the achievable production level will be reduced due to
a shortage of available resources. The resources that are expected to be available are as
follows:

Direct material A 1,800kg


Direct material B 3,500kg
Skilled labour 2,500 hours
Machine time 6,500 machine hours

Required:

(a) Using graphical linear programming identify the weekly production schedule for
products D and G that maximises the profits of DFG during the next four weeks.

(15 marks)

(b) The optimal solution to part (a) shows that the shadow prices of Skilled labour and
Direct material A are as follows:

Skilled labour Nil


Direct material A 5.82

Explain the relevance of these values to the management of DFG.


(6 marks)

35 | P a g e
(c) Using the graph you have drawn in part (a) explain how you would calculate by how
much the selling price of Product D could rise before the optimal solution would change.

Note: Assume that demand is not affected by the selling price. You are not required to
perform any calculations.
(4 marks)

(Total = 25 marks)

A2 8 Highly skilled workers (CIMA P2 May 2008)

An engineering company manufactures a number of products and components, using a


team of highly skilled workers and a variety of different metals.

The current supplier has announced that the amount of M1, one of the materials it
currently supplies, will be limited to 1,000 square metres in total for the next three-month
period because there will be insufficient M1 to satisfy demand.

The only items manufactured using M1 and their production costs and selling prices
(where applicable) are shown below:

Product Product Component Component


P4 P6 C3 C5
$/unit $/unit $/unit $/unit
Selling price 125 175 n/a n/a

Direct materials:
M1 * 15 10 5 10
M2 10 20 15 20
Direct labour 20 30 16 10
Variable overhead 10 15 8 5
Fixed overhead ** 20 30 16 10

Total cost 75 105 60 55

* Material M1 is expected to be limited in supply during the next three months. These
costs are based on M1 continuing to be available at a price of $20 per square metre.

** Fixed overhead is absorbed on the basis of direct labour cost.

Products P4 and P6 are sold externally. Components C3 and C5 are used in other
products made by the company. These other products do not require any further amounts
of material M1.

36 | P a g e
The estimated total demand for these products and components during the next three
months is as follows:

P4 2,000 units
P6 1,500 units
C3 500 units
C5 1,000 units

Components C3 and C5 are essential components. They would have to be bought in if


they could not be made internally. They can be purchased from external suppliers for $75
and $95 per unit respectively. The bought in components are of the same quality as those
manufactured by the company. The products they are used in have sufficient margins to
remain financially worthwhile if C3 and C5 are bought in at these prices.

Required:

(a) Prepare calculations to show the most profitable course of action for the company for
the next three months, assuming that there are no other suppliers of material M1, and
advise the company on THREE other factors that it should consider before making its
decision.
(14 marks)

(b) Calculate the maximum prices that the company should pay to obtain further supplies
of material M1 from an alternative supplier, and the quantities of material M1 to which
each of these prices apply.
(6 marks)

The company has now become aware of a contract that it has already accepted, for the
immediate delivery of 500 units of P4 at a selling price of $125 per unit. This contract has
a financial penalty clause for non-delivery. This contract is in addition to the 2,000 units
of estimated demand for P4 stated previously. Assume that there is no alternative supplier
of material M1.

(c) Calculate the minimum financial penalty that would change your recommendation.
(5 marks)

(Total = 25 marks)

A2 9 RT (CIMA P2 May 2010)

RT produces two products from different quantities of the same resources using a just-in-
time (JIT) production system. The selling price and resource requirements of each of the
products are shown below:

37 | P a g e
Product R T

Unit selling price ($) 130 160

Resources per unit:


Direct labour ($8 per hour) 3 hours 5 hours

Material A ($3 per kg) 5 kgs 4 kgs

Material B ($7 per litre) 2 litres 1 litre

Machine hours ($10 per hour) 3 hours 4 hours

Market research shows that the maximum demand for products R and T during June 2010
is 500 units and 800 units respectively. This does not include an order that RT has agreed
with a commercial customer for the supply of 250 units of R and 350 units of T at selling
prices of $100 and $135 per unit respectively. Although the customer will accept part of
the order, failure by RT to deliver the order in full by the end of June will cause RT to
incur a $10,000 financial penalty.

At a recent meeting of the purchasing and production managers to discuss the production
plans of RT for June, the following resource restrictions for June were identified:

Direct labour hours 7,500 hours


Material A 8,500 kgs
Material B 3,000 litres
Machine hours 7,500 hours

Required:

(a) Assuming that RT completes the order with the commercial customer, prepare
calculations to show, from a financial perspective, the optimum production plan for June
2010 and the contribution that would result from adopting this plan.
(6 marks)

(b) Prepare calculations to show, from a financial perspective, whether RT should


complete the order from the commercial customer
(3 marks)

You have now presented your optimum production plan to the purchasing and production
managers of RT. During your presentation it became clear that the predicted resource
restrictions were rather optimistic. In fact the managers agreed that the availability of all
of the resources could be as much as 10% lower than their original predictions.

38 | P a g e
(c) Assuming that RT completes the order with the commercial customer, andusing
graphical linear programming, prepare a graph to show the optimum production plan for
RT for June 2010 on the basis that the availability of all resources is 10% lower than
originally predicted.
(11 marks)

(d) Discuss how the graph in your solution to (c) above can be used to help to determine
the optimum production plan for June 2010 if the actual resource availability lies
somewhere between the managers optimistic and pessimistic predictions.
(5 marks)

(Total = 25 marks)

A2 10 LM (CIMA P2 Nov 2010)

LM produces two products from different quantities of the same resources using a just-in-
time (JIT) production system. The selling price and resource requirements of each of
these two products are as follows:

Product L M
Unit selling price ($) 70 90

Variable costs per unit:


Direct labour ($7 per hour) 28 14
Direct material ($5 per kg) 10 45
Machine hours ($10 per hour) 10 20

Fixed overheads absorbed 12 6

Profit per unit 10 5

Fixed overheads are absorbed at the rate of $3 per direct labour hour.

Market research shows that the maximum demand for products L and M during
December 2010 will be 400 units and 700 units respectively.

At a recent meeting of the purchasing and production managers to discuss the companys
production plans for December 2010, the following resource availability for December
2010 was identified:

Direct labour 3,500 hours


Direct material 6,000 kg
Machine hours 2,000 hours

39 | P a g e
Required:

(a) Prepare calculations to show, from a financial perspective, the optimum production
plan for December 2010 and the contribution that would result from adopting your plan.
(6 marks)

(b) You have now presented your optimum plan to the purchasing and production
managers of LM. During the presentation, the following additional information became
available:

(i) The company has agreed to an order for 250 units of product M for a selling price
of $90 per unit from a new overseas customer. This order is in addition to the
maximum demand that was previously predicted and must be produced and
delivered in December 2010;

(ii) The originally predicted resource restrictions were optimistic. The managers now
agree that the availability of all resources will be 20% lower than their original
predictions.

Required:

Construct the revised resource constraints and the objective function to be used to
identify, given the additional information above, the revised optimum production plan for
December 2010.
(6 marks)

(c) The resource constraints and objective function requested in part (b) above have now
been processed in a simplex linear programming model and the following solution has
been printed:

Product L 400 Product L other value 0


Product M 194 Product M other value 506
Direct labour 312
Direct material ($) 1.22
Machine hours 312
Contribution ($) 10,934.00

Required:

Analyse the meaning of each of the above eight values in the solution to the problem.
Your answer should include a proof of the five individual values highlighted in bold.
(13 marks)

(Total = 25 marks)

40 | P a g e
A2 11 Hotel (CIMA P2 May 2011)

The management of a hotel is planning for the next year. The hotel has 100 bedrooms.
The price of a room night includes breakfast for the guests. Other services (a snack
service and a bar and restaurant) are available but are not included in the price of the
room night. These additional services are provided to hotel guests only.

For planning purposes the hotel divides the year (based on 360 days) into three seasons:
peak, mid and low.

Details of the hotel and its services and forecasts for the next year are given below.

1. Seasons, room charges, room occupancy, guests per room and room revenue
The hotel charges a price per room per night (including breakfast) irrespective of the
number of guests per room. The price charged is different in each of the seasons.

Season Peak Mid Low

Number of days 90 120 150


Price charged per room per night ($) 100.00 80.00 55.00
Hotel room occupancy % 95 75 50
Average number of guests per room 1.8 1.5 1.2
Total room revenue ($) 855,000 720,000 412,500

2. Guest related costs


The hotel incurs some costs, including providing breakfast, that are directly related to the
number of guests in the hotel. These are $12 per guest per night in all seasons.

3. Room related costs


The hotel incurs some costs that are directly related to the number of rooms occupied.
These include cleaning and laundry costs of $5 per occupied room per night regardless of
season. There are also power and lighting costs of $3 in the peak season, $4 in the mid
season and $6 in the low season per occupied room per night.

4. Hot snacks
The hotel offers a 24 hour hot snacks service to the guests. Past records show that this
service has been used by 30% of its guests in the mid and low seasons but only 10% in
the
peak season. It is forecast that the average spend per guest per night will be $10. The
hotel earns a 30% gross contribution from this income.

The hotel employs a cook on a salary of $20,000 per year to provide this service. All of
the costs for the hot snacks service, except for the cooks salary, are variable. The cook
could be made redundant with no redundancy costs.

41 | P a g e
5. Restaurant & Bar
Past records show that the usage of the restaurant and bar is seasonal. The restaurant and
bar are particularly popular with the hotels business guests. The forecast usage is shown
below.

Season Daily demand


Peak 30% of hotel guests spend an average of $15 each
Mid 50% of hotel guests spend an average of $20 each
Low 70% of hotel guests spend an average of $30 each

The hotel earns a 25% gross contribution from this income and employs two chefs on a
combined salary of $54,000 per year to provide this facility. All of the costs in the
restaurant and bar, except for the salaries of the chefs, are variable.

The two chefs could be made redundant with no redundancy costs.

6. General hotel costs.


These include the costs of reception staff, the heating and lighting of the common areas
and other facility related costs. The forecast costs for next year are:

Peak season $300,000


Mid season $400,000
Low season $500,000

These costs could be reduced by 75% if the hotel were to close temporarily for one or
more seasons of the year.

There are also some costs that are incurred by the hotel and can only be avoided by its
permanent closure. These are estimated to $200,000 for next year.

Required:

(a) Prepare, in an appropriate format, a columnar statement that will help the managers of
the hotel to plan for next year. Your statement should show the hotels activities by
season and in total.
(18 marks)

(b)

(i) Identify, based on your statement, the actions that the managers could take to
maximise the profit of the hotel for next year.
(3 marks)

42 | P a g e
(ii) Explain TWO factors that the managers should consider before implementing the
actions you identified in (b)(i).
(4 marks)

(Total = 25 marks)

43 | P a g e
Questions Section A

Part B Cost Planning and Analysis for Competitive Advantage

B1 1 SWAL (CIMA P2 Pilot paper 2005)

SW is a member of the SWAL Group of companies. SW manufactures cleaning liquid


using chemicals that it buys from a number of suppliers. In the past SW has used a
periodic review stock control system with maximum, minimum and re-order levels to
control the purchase of the chemicals and the economic order quantity model to minimise
its costs.

The Managing Director of SW is considering a change by introducing a Just in Time


(JIT) system.

Required:

As Management Accountant, prepare a report to the Managing Director that explains how
a JIT system differs from the system presently being used and the extent to which its
introduction would require a review of SWs quality control procedures.
(10 marks)

B1 2 X group (CIMA P2 May 2005)

The X group is a well-established manufacturing group that operates a number of


companies using similar production and inventory holding policies. All of the companies
are in the same country though there are considerable distances between them.

The group has traditionally operated a constant production system whereby the same
volume of output is produced each week, even though the demand for the groups
products is subject to seasonal fluctuations. As a result there is always finished goods
inventory in the groups warehouses waiting for customer orders. This inventory will
include a safety inventory equal to two weeks production.

Raw material inventories are ordered from suppliers using the Economic Order Quantity
(EOQ) model in conjunction with a computerised inventory control system which
identifies the need to place an order when the re-order level is reached. The purchasing
department is centralised for the group. On receiving a notification from the
computerised inventory control system that an order is to be placed, a series of quotation
enquiries are issued to prospective suppliers so that the best price and delivery terms are
obtained for each order. This practice has resulted in there being a large number of
suppliers to the X group. Each supplier delivers directly to the company that requires the
material.

44 | P a g e
The Managing Director of the X group has recently returned from a conference on World
Class Manufacturing and was particularly interested in the possible use of Just In Time
(JIT) within the X group.

Required:

Write a report, addressed to the Managing Director of the X group that explains how the
adoption of JIT might affect its profitability.
(10 marks)

B1 3 ML (CIMA P2 Nov 2005)

ML is an engineering company that specialises in providing engineering facilities to


businesses that cannot justify operating their own facilities in-house. ML employs a
number of engineers who are skilled in different engineering techniques that enable ML
to provide a full range of engineering facilities to its customers. Most of the work
undertaken by ML is unique to each of its customers, often requiring the manufacture of
spare parts for its customers equipment, or the building of new equipment from customer
drawings. As a result most of MLs work is short-term, with some jobs being completed
within hours while others may take a few days.

To date ML has adopted a cost plus approach to setting its prices. This is based upon an
absorption costing system that uses machine hours as the basis of absorbing overhead
costs into individual job costs. The Managing Director is concerned that over recent
months ML has been unsuccessful when quoting for work with the consequence that
there has been an increase in the level of unused capacity. It has been suggested that ML
should adopt an alternative approach to its pricing based on marginal costing since any
price that exceeds variable costs is better than no work.

Required:

With reference to the above scenario

(i) briefly explain absorption and marginal cost approaches to pricing;

(ii) discuss the validity of the comment any price that exceeds variable costs isbetter
than no work.

(10 marks)

45 | P a g e
B1 4 PK plc (CIMA P2 Nov 2005)

You are the assistant management accountant within PK plc. PK plc manufactures high
quality self-assembly furniture from raw materials utilising highly skilled labour within a
computer-controlled manufacturing facility. The company produces a range of furniture,
and, because of the lead time to receive delivery of its raw materials, has a finished goods
inventory policy of holding an average of two weeks estimated sales in inventory.
Customer demand is seasonal and, as a consequence, this finished goods inventory level
fluctuates throughout the year. The company also holds inventories of raw materials
based upon estimates of its production requirements. An absorption costing system is
used to attribute all manufacturing costs to output.

Increasingly PK plc is facing competition, particularly from overseas manufacturers and


its sales team have to make decisions about the extent to which it can offer price
discounts in order to win customer orders.

Required:

Prepare a report addressed to the Management Team of PK plc that explains the changing
nature of cost structures in the modern manufacturing environment and the implications
for PK plcs

(i) inventory valuation


(ii) short term decision making

(10 marks)

Note: There are 2 marks available for format and presentational style

B1 5 Financial advisors (CIMA P2 May 2006)

A firm of financial advisors has established itself by providing high quality, personalised,
financial strategy advice. The firm promotes itself by sponsoring local events,
advertising, client newsletters, having a flexible attitude towards the times and locations
of meetings with clients and seeking new and innovative ideas to discuss with its clients.

The senior manager of the firm has recently noticed that the firms profitability has
declined, with fewer clients being interested in the firms new investment ideas. Indeed,
many clients have admitted to not reading the firms newsletters.

The senior manager seeks your help in restoring the firms profitability to its former level
and believes that the techniques of Value Analysis and Functional Analysis may be
appropriate.

46 | P a g e
Required:

(a) Explain the meanings of, and the differences between, Value Analysis and Functional
Analysis.
(4 marks)

(b) Briefly explain the series of steps that you would take to implement Value Analysis
for this organisation.
(6 marks)

(Total = 10 marks)

B1 6 Compliance v conformance (CIMA P2 May 2006)

The Managing Director of a manufacturing company based in Eastern Europe has


recently returned from a conference on modern manufacturing. One of the speakers at the
conference presented a paper entitled Compliance versus Conformance the quality
control issue. The Managing Director would like you to explain to her some of the
concepts that she heard about at the conference.

Required:

Prepare a report, addressed to the Managing Director, that discusses quality costs and
theirsignificance for the company. Your report should include examples of the different
quality costs and their classification within a manufacturing environment.

(10 marks)

Note: 2 marks are available for report format

B1 7 AVN (CIMA P2 Nov 2006)

AVN designs and assembles electronic devices to allow transmission of audio / visual
communications between the original source and various other locations within the same
building. Many of these devices require a wired solution but the company is currently
developing a wireless alternative. The company produces a number of different devices
depending on the number of input sources and the number of output locations, but the
technology used within each device is identical. AVN is constantly developing new
devices which improve the quality of the audio / visual communications that are received
at the output locations.

The Managing Director recently attended a conference on world class manufacturing


entitled The extension of the value chain to include suppliers and customers and seeks
your help.

47 | P a g e
Required:

Explain

(i) the components of the extended value chain; and


(3 marks)

(ii) how each of the components may be applied by AVN.


(7 marks)

(Total = 10 marks)

B1 8 W (CIMA P2 Nov 2006)

W has recently completed the development and testing of a new product which has cost
$400,000. It has also bought a machine to produce the new product costing $150,000.
The production machine is capable of producing 1,000 units of the product per month and
is not expected to have a residual value due to its specialised nature.

The company has decided that the unit selling prices it will charge will change with the
cumulative numbers of units sold as follows:

Cumulative sales units Selling price


$ per unit in this band
0 to 2,000 100
2,001 to 7,000 80
7,001 to 14,500 70
14,501 to 54,500 60
54,501 and above 40

Based on these selling prices, it is expected that sales demand will be as shown below:

Months Sales demand per month


(units)
1 10 200
11 20 500
21 30 750
31 70 1,000
71 80 800
81 90 600
91 100 400
101 110 200
Thereafter NIL

48 | P a g e
Unit variable costs are expected to be as follows:

$ per unit
First 2,000 units 50
Next 12,500 units 40
Next 20,000 units 30
Next 20,000 units 25
Thereafter 30

W operates a Just in Time (JIT) purchasing and production system and operates its
business on a cash basis.

A columnar cash flow statement showing the cumulative cash flow of the product after its
Introduction and Growth stages has already been completed and this is set out below:

Introduction Growth
Months 1-10 11-30
Number of units produced and sold 2,000 5,000 7,500
Selling price per unit $100 $80 $70
Unit variable cost $50 $40 $40
Unit contribution $50 $40 $30
Total contribution $100,000 $425,000
Cumulative cash flow ($450,000) ($25,000)

Required:

(a) Complete the cash flow statement for each of the remaining two stages of the
products life cycle. Do not copy the Introduction and Growth stages in your answer.
Ignore the time value of money.
(5 marks)

(b) Explain, using your answer to (a) above and the data provided, the possible reasons
for the changes in costs and selling prices during the life cycle of the product.
(5 marks)

(Total = 10 marks)

B1 9 New product (CIMA P2 May 2007)

A company is planning to launch a new product. It has already carried out market
research at a cost of $50,000 and as a result has discovered that the market price for the
product should be $50 per unit. The company estimates that 80,000 units of the product
could be sold at this price before one of the companys competitors enters the market
with a superior product. At this time any unsold units of the companys product would be
of no value.

49 | P a g e
The company has estimated the costs of the initial batch of the product as follows:

$000
Direct materials 200
Direct labour ($10 per hour) 250
Other direct costs 100

Production was planned to occur in batches of 10,000 units and it was expected that an
80% learning curve would apply to the direct labour until the fourth batch was complete.
Thereafter the direct labour cost per batch was expected to be constant. No changes to the
direct labour rate per hour were expected.

The company introduced the product at the price stated above, with production occurring
in batches of 10,000 units. Direct labour was paid using the expected hourly rate of $10
and the company is now reviewing the profitability of the product. The following
schedule shows the actual direct labour cost recorded:

Cumulative number of batches Actual cumulative direct labour cost


$000
1 280
2 476
4 809
8 1,376

Required:

(i) Calculate the revised expected cumulative direct labour costs for the four levels of
output given the actual cost of $280,000 for the first batch.

(ii) Calculate the actual learning rate exhibited at each level of output.
(iii) Discuss the implications of your answers to (i) and (ii) for the managers of the
company.

(10 marks)

B1 10 New small company (CIMA P2 Nov 2007)

You are the management accountant of a new small company that has developed a new
product using a labour intensive production process. You have recently completed the
budgets for the company for next year and, before they are approved by the Board of
Directors, you have been asked to explain your calculation of the labour time required for
the budgeted output. In your calculations, you anticipated that the time taken for the first
unit would be 40 minutes and that a 75% learning curve would apply for the first 30
units.

50 | P a g e
Required:

(a) Explain the concept of the learning curve and why it may be relevant to the above
company.
(3 marks)

(b) Calculate the expected time for the 6th unit of output.
(3 marks)

(c) Discuss the implications of the learning curve for a company adopting a penetration
pricing policy.
(4 marks)

(Total = 10 marks)

Note: The learning index for a 75% learning curve is -0.415

B1 11 XY (CIMA P2 May 2008)

You are the Management Accountant of XY, an engineering company that assembles
components into engines for sale to the automotive industry. The company is constantly
under pressure from its customers to provide more efficient engines, which are also less
damaging to the environment. The company uses value chain analysis as a tool in the
management of its activities.

The Managing Director of XY has recently been invited to a conference to give a


presentation entitled The concept of the Value Chain and the management of profits
generated throughout the chain in XY.

Required:

Prepare a report for the Managing Director explaining the points that should be covered
in the presentation.
(10 marks)

B1 12 Inventory levels (CIMA P2 May 2008)

A company experiences changing levels of demand, but produces a constant number of


units during each quarter. The company allows inventory levels to rise and fall to satisfy
the differing quarterly demand levels for its product.

51 | P a g e
Required:

(a) Identify and explain the reasons for THREE cost changes that would result if the
company changed to a Just-In-Time production method for 2009. Assume there will be
no inventory at the start and end of the year.
(6 marks)

(b) Briefly discuss the importance of Total Quality Management to a company that
operates a Just-In-Time production method.
(4 marks)

(Total = 10 marks)

B1 13 Workshop (CIMA P2 May 2008)

A company has developed a new product that it will manufacture in its workshop. The
product is highly specialised and initially will be produced to order only. The product
will be manufactured in batches. The estimated labour time required for the first batch is
40 hours, but due to the nature of the product and the manufacturing method to be used, it
is expected that an 80% learning curve will apply.

Required:

(a) Calculate the expected time for the eighth batch.


(3 marks)

(b) When production commenced the first batch took 45 hours. The actual learning rates
observed were as follows:

Month Total batches produced to date Actual learning rate


1 1
2 2 75%
3 4 75%
4 8 90%

For each of months 2 and 4, state possible reasons why the actual learning rates differed
from the expected rates.
(3 marks)

(c) The total time taken to produce the first eight batches was 18225 hours. Calculate the
cumulative learning rate up to the end of Month 4. (Remember that the first batch took 45
hours).
(4 marks)
(Total = 10 marks)

52 | P a g e
B1 14 Out-turn performace report (CIMA P2 May 2010)

The budget for the production cost of a new product was based on the following
assumptions:

(i) Time for the 1st batch of output = 10 hours


(ii) Learning rate = 80%
(iii)Learning will cease after 40 batches, and thereafter the time per batch will be the
same as the time of the final batch during the learning period, i.e. the 40th batch
(iv) Standard direct labour rate per hour = $12.00

An extract from the out-turn performance report based on the above budget is as follows:

Budget Actual Variance

Output (batches) 60 50 10 adverse

Direct labour hours 163.53 93.65 69.88 favourable

Direct labour cost $1,962 $1,146 $816 favourable

Further analysis has shown that, due to similarities between this product and another that
was developed last year, the rate of learning that should have been expected was 70% and
that the learning should have ceased after 30 batches. Other budget assumptions for the
new product remain valid.
Required:

(a) Prepare a revised out-turn performance report for the new product that:
(i) shows the flexed budgeted direct labour hours and direct labour cost based on the
revised learning curve data, and

(ii) shows the variances that reconcile the actual results to your flexed budget in as much
detail as possible.
(7 marks)

(b) Explain why your report is more useful to the production manager than the report
shown above.
(3 marks)

Note: The learning index values for an 80% and a 70% learning curve are -0.3219 and -
0.5146 respectively.

(Total 10 marks)

53 | P a g e
B1 15 PQ (CIMA P2 May 2010)

PQ manufactures and sells consumer electronics. It is constantly working to design the


latest gadgets and must-haves which are unique in the market place at the time they are
launched. The management of PQ are aware of the short product life cycles in this
competitive market and consequently use a market skimming pricing strategy at the
introduction stage.

Required:

Explain the changes that are likely to occur in the following items at the three later stages
in the product life cycle of a typical PQ product.

(i) Selling price


(ii) Production costs
(iii) Selling and marketing costs

(Total = 10 marks)

B1 16 Timber products (CIMA P2 May 2010)

XY, a company that manufactures a range of timber products, is considering changing to


a just-in-time (JIT) production system.

Currently XY employs staff who are contracted to work and be paid for a total of
3,937.75 hours per month. Their labour efficiency ratio is 96% and, as a result, they are
able to produce 3,780 standard hours of output each month in normal working hours.

Overtime working is used to meet additional demand, though the management of XY try
to avoid the need for this because it is paid at a 50% premium to the normal hourly rate of
$10 per hour. Instead, XY plan production so that in months of lower demand inventory
levels increase to enable sales demand to be met in other months. XY has determined that
the cost of holding inventory is $6 per month for each standard hour of output that is held
in inventory.

XY has forecast the demand for its products for the next six months as follows:

Month Demand (Standard hours)


1 3,100
2 3,700
3 4,000
4 3,300
5 3,600
6 4,980

54 | P a g e
You may assume that all production costs (other than labour) are either fixed or are not
driven by labour hours worked, and that there is zero inventory at the start of month 1 and
at the end of month 6. Assume also that production and sales occur evenly during each
month at present, and that the minimum contracted hours will remain the same with the
JIT system.

Required:

(a) With the current production system:

(i) Calculate for each of the six months and the period in total, the total inventory holding
costs.
(ii) Calculate the total production cost savings made by changing to a JIT production
system.
(6 marks)

(b) Explain TWO other factors that should be considered by XY before changing to a JIT
production system.
(4 marks)

(Total = 10 marks)

B1 17 LMN (CIMA P2 May 2010)

LMN comprises three trading divisions plus a Head Office. There is a director for each
trading division and, in addition, there is a Managing Director who is based in Head
Office. Divisional directors are empowered to make decisions concerning the day to day
operations of their division and investment decisions requiring an initial investment up to
$100,000. Investment decisions involving greater initial expenditure must be authorised
by the Managing Director. Inter-divisional trading occurs between all of the trading
divisions. The transfer prices are determined by Head Office. Head Office provides
services and facilities to each of the trading divisions.

At the end of each month, the actual costs of Head Office are apportioned to the trading
divisions. Each Head Office cost is apportioned to the trading divisions using an
appropriate basis. The bases used are: number of employees; value of sales; capital
invested; and standard hours of service delivered.

The Head Office costs, together with the costs and revenues generated at divisional level,
are summarised in a divisional performance statement each month. The divisional
directors are not happy with the present performance statement and how it is used to
appraise their performance.

55 | P a g e
Required:

(a) Explain, using examples from the scenario, three issues that LMN should consider
when designing a new divisional performance statement.
(6 marks)

LMN is thinking of introducing Activity Based Costing at its Head Office to help with
the apportionment of all its costs to the divisions.

(b) Discuss the advantages of applying Activity Based Costing to apportion all of the
Head Office costs.
(4 marks)

(Total = 10 marks)

B1 18 Production manager (CIMA P2 Nov 2010)

The following variances have been calculated in respect of a new product:

Direct labour efficiency variance $14,700 Favourable


Direct labour rate variance $ 5,250 Adverse

The variances were calculated using standard cost data which showed that each unit of
the product was expected to take 8 hours to produce at a cost of $15 per hour. Actual
output of the product was 560 units and actual time worked in the manufacture of the
product totalled 3,500 hours at a cost of $57,750.

However, the production manager now realises that the standard time of 8 hours per unit
was the time taken to produce the first unit and that a learning rate of 90% should have
been anticipated for the first 600 units.

Required:

(a) Calculate planning and operating variances following the recognition of the learning
curve effect.
(6 marks)

(b) Explain the importance of learning curves in the context of Target Costing.
(4 marks)

Note: The learning index for a 90% learning curve is -0.1520

(Total = 10 marks)

56 | P a g e
B1 19 CAL (CIMA P2 Nov 2010)

CAL manufactures and sells solar panels for garden lights. Components are bought in and
assembled into metal frames that are machine manufactured by CAL. There are a number
of alternative suppliers of these solar panels. Some of CALs competitors charge a lower
price, but supply lower quality panels; whereas others supply higher quality panels than
CAL but for a much higher price.

CAL is preparing its budgets for the coming year and has estimated that the market
demand for its type of solar panels will be 100,000 units and that its share will be 20,000
units (i.e. 20% of the available market). The standard cost details of each solar panel are
as follows:
$ per unit
Selling price 60
Bought - in components (1 set) 15
Assembly & machining cost 25
Delivery cost 5 45
Contribution 15

An analysis of CALs recent performance revealed that 2% of the solar panels supplied to
customers were returned for free replacement, because the customer found that they were
faulty. Investigation of these returned items shows that the components had been
damaged when they had been assembled into the metal frame. These returned panels
cannot be repaired and have no scrap value. If the supply of faulty solar panels to
customers could be eliminated then, due to improved customer perception, CALs market
share would increase to 25%.

Required:

(a) Explain, with reference to CAL, quality conformance costs and quality non-
conformance costs and the relationship between them.
(4 marks)

(b) Assuming that CAL continues with its present systems and that the percentage of
quality failings is as stated above:

(i) Calculate, based on the budgeted figures and sales returns rate, the total relevant costs
of quality for the coming year.
(4 marks)

(ii) Calculate the maximum saving that could be made by implementing an inspection
process for the solar panels, immediately before the goods are delivered.
(2 marks)

(Total = 10 marks)

57 | P a g e
B1 20 QW (CIMA P2 Nov 2010)

QW is a company that manufactures machine parts from sheet metal to specific customer
order for industrial customers. QW is considering diversification into the production of
metal ornaments. The ornaments would be produced at a constant rate throughout the
year. It then plans to sell these ornaments from inventory through wholesalers and via
direct mail to consumers.

Presently, each of the machine parts is specific to a customers order. Consequently, the
company does not hold an inventory of finished items but it does hold the equivalent of
one days production of sheet metal so as to reduce the risk of being unable to produce
goods demanded by customers at short notice. There is a one day lead time for delivery
of sheet metal to QW from its main supplier though additional supplies could be obtained
at less competitive prices.

Demand for these industrial goods is such that delivery is required almost immediately
after the receipt of the customer order. QW is aware that if it is unable to meet an order
immediately the industrial customer would seek an alternative supplier, despite QW
having a reputation for high quality machine parts.

The management of QW is not aware of the implications of the diversification for its
production and inventory policies.

Required:

(a) Compare and contrast QWs present production and inventory policy and practices
with a traditional production system that uses constant production levels and holds
inventory to meet peaks of demand.
(5 marks)

(b) Discuss the importance of a Total Quality Management (TQM) system in a just-in-
time (JIT) environment. Use QW to illustrate your discussion.
(5 marks)

(Total = 10 marks)

B1 21 Accountancy services (CIMA P2 Nov 2010)

XY provides accountancy services and has three different categories of client: limited
companies, self employed individuals, and employed individuals requiring taxation
advice. XY currently charges its clients a fee by adding a 20% mark-up to total costs.
Currently the costs are attributed to each client based on the hours spent on preparing
accounts and providing advice.

58 | P a g e
XY is considering changing to an activity based costing system. The annual costs and the
causes of these costs have been analysed as follows:

$
Accounts preparation and advice 580,000
Requesting missing information 30,000
Issuing fee payment reminders 15,000
Holding client meetings 60,000
Travelling to clients 40,000

The following details relate to three of XYs clients and to XY as a whole:


Client XY
A B C
Hours spent on preparing
accounts and providing advice 1,000 250 340 18,000
Requests for missing information 4 10 6 250
Payment reminders sent 2 8 10 400
Client meetings held 4 1 2 250
Miles travelled to meet clients 150 600 0 10,000

Required:

Prepare calculations to show the effect on fees charged to each of these three clients of
changing to the new costing system.
(10 marks)

B1 22 PT (CIMA P2 May 2011)

PT manufactures and sells a number of products. All of its products have a life cycle of
six months or less. PT uses a four stage life cycle model (Introduction; Growth; Maturity;
and Decline) and measures the profits from its products at each stage of their life cycle.

PT has recently developed an innovative product. Since the product is unique it was
decided that it would be launched with a market skimming pricing policy. However PT
expects that other companies will try to enter the market very soon.

This product is generating significant unit profits during the Introduction stage of its life
cycle. However there are concerns that the unit profits will reduce during the other stages
of the products life cycle.

59 | P a g e
Required:
For each of the

(i) Growth; and


(ii) Maturity stages of the new products life cycle

explain the likely changes that will occur in the unit selling prices AND in the unit
production costs, compared to the preceding stage.

(Total = 10 marks)

B1 23 TQM and JIT (CIMA P1 Pilot Paper 2005)

Give FOUR reasons why the adoption of Total Quality Management (TQM) is
particularly important within a Just-in-Time (JIT) production environment.
(5 marks)

B1 24 Standard costing (CIMA P1 May 2006)

Briefly discuss three reasons why standard costing may not be appropriate in a modern
business environment.
(5 Marks)

B1 25 Marginal v throughput (CIMA P1 May 2006)

Compare and contrast marginal costing and throughput accounting.


(5 Marks)

B1 26 MRPS (CIMA P1 May 2007)

Briefly explain the role of a Manufacturing Resource Planning System in supporting a


standard costing system.
(5 Marks)

B1 -27 JIT (CIMA P1 May 2007)

Briefly explain the main differences between the traditional manufacturing environment
and a just-in-time manufacturing environment.
(5 marks)

60 | P a g e
B1 28 Key features of TQM (CIMA P1 May 2008)

Describe THREE key features that are present in any organisation that is successfully
focused on Total Quality Management (TQM).
(5 marks)

61 | P a g e
Questions Section B

Part B Cost Planning and Analysis for Competitive Advantage

B2 -1 The Q organisation (CIMA P2 May 2005)

(a) The Q organisation is a large, worldwide respected manufacturer of consumer


electrical and electronic goods. Q constantly develops new products that are in high
demand as they represent the latest technology and are must haves for those consumers
that want to own the latest consumer gadgets. Recently Q has developed a new handheld
digital DVD recorder and seeks your advice as to the price it should charge for such a
technologically advanced product.

Required:

Explain the relevance of the product life cycle to the consideration of alternative pricing
policies that might be adopted by Q.
(10 marks)

(b) Market research has discovered that the price demand relationship for the item during
the initial launch phase will be as follows:

Price () Demand (units)


100 10,000
80 20,000
69 30,000
62 40,000

Production of the DVD recorder would occur in batches of 10,000 units, and the
production director believes that 50% of the variable manufacturing cost would be
affected by a learning and experience curve. This would apply to each batch produced
and continue at a constant rate of learning up to a production volume of 40,000 units
when the learning would be complete. Thereafter, the unit variable manufacturing cost of
the product would be equal to the unit cost of the fourth batch. The production director
estimates that the unit variable manufacturing cost of the first batch would be 60 (30 of
which is subject to the effect of the learning and experience curve, and 30 of which is
unaffected), whereas the average unit variable manufacturing cost of all four batches
would be 52.71.

There are no non-manufacturing variable costs associated with the DVD recorder.

Required:

(i) Calculate the rate of learning that is expected by the production director.
(4 marks)

62 | P a g e
(ii) Calculate the optimum price at which Q should sell the DVD recorder in order to
maximise its profits during the initial launch phase of the product.
(8 marks)

(iii) Q expects that after the initial launch phase the market price will be 57 per unit.
Estimated product specific fixed costs during this phase of the products life are expected
to be 15,000 per month. During this phase of the product life cycle Q wishes to achieve
a target monthly profit from the product of 30,000.

Calculate the number of units that need to be sold each month during this phase in order
that Q achieves this target monthly profit.
(3 marks)

(Total = 25 marks)

B2 -2 F plc (CIMA P1 May 2005)

F plc supplies pharmaceutical drugs to drug stores. Although the company makes a
satisfactory return, the directors are concerned that some orders are profitable and others
are not. The management has decided to investigate a new budgeting system using
activity based costing principles to ensure that all orders they accept are making a profit.

Each customer order is charged as follows. Customers are charged the list price of the
drugs ordered plus a charge for selling and distribution costs (overheads). A profit margin
is also added, but that does not form part of this analysis.

Currently F plc uses a simple absorption rate to absorb these overheads. The rate is
calculated based on the budgeted annual selling and distribution costs and the budgeted
annual total list price of the drugs ordered.

An analysis of customers has revealed that many customers place frequent small orders
with each order requesting a variety of drugs. The management of F plc has examined
more carefully the nature of its selling and distribution costs, and the following data have
been prepared for the budget for next year:

Total list price of drugs supplied 8m


Number of customer orders 8,000

Selling and Distribution Costs 000 Cost driver


Invoice processing 280 See Note 2
Packing 220 Size of package see Note 3
Delivery 180 Number of deliveries see Note 4
Other overheads 200 Number of orders
Total overheads 880

63 | P a g e
Notes:

1. Each order will be shipped in one package and will result in one delivery to the
customer and one invoice (an order never results in more than one delivery).

2. Each invoice has a different line for each drug ordered. There are 28,000 invoice
lines each year. It is estimated that 25% of invoice processing costs are related to
the number of invoices, and 75% are related to the number of invoice lines.

3. Packing costs are 32 for a large package, and 25 for a small package.

4. The delivery vehicles are always filled to capacity for each journey. The delivery
vehicles can carry either 6 large packages or 12 small packages (or appropriate
combinations of large and small packages). It is estimated that there will be 1,000
delivery journeys each year, and the total delivery mileage that is specific to
particular customers is estimated at 350,000 miles each year. 40,000 of delivery
costs are related to loading the delivery vehicles and the remainder of these costs
are related tospecific delivery distance to customers.

The management has asked for two typical orders to be costed using next years budget
data, using the current method, and the proposed activity-based costing approach. Details
of two typical orders are shown below:

Order A Order B
Lines on invoice 2 8
Package size small large
Specific delivery distance 8 miles 40 miles
List price of drugs supplied 1,200 900

Required:

(a) Calculate the charge for selling and distribution overheads for Order A and Order B
using:

(i) the current system; and


(ii) the activity-based costing approach.
(10 marks)

(b) Write a report to the management of F plc in which you


(i) assess the strengths and weaknesses of the proposed activity-based costing
approach for F plc; and
(5 marks)

64 | P a g e
(ii) recommend actions that the management of F plc might consider in the light of
the data produced using the activity-based-costing approach.
(5 marks)

(Total = 20 marks)

B2 3 KL (CIMA P2 Nov 2006)

KL manufactures three products, W, X and Y. Each product uses the same materials and
the same type of direct labour but in different quantities. The company currently uses a
cost plus basis to determine the selling price of its products. This is based on full cost
using an overhead absorption rate per direct labour hour. However, the Managing
Director is concerned that the company may be losing sales because of its approach to
setting prices. He thinks that a marginal costing approach may be more appropriate,
particularly since the workforce is guaranteed a minimum weekly wage and has a three
month notice period.

Required:

(a) Given the Managing Directors concern about KLs approach to setting selling prices,
discuss the advantages and disadvantages of marginal cost plus pricing AND total cost
plus pricing.
(6 marks)

The direct costs of the three products are shown below:

Product W X Y
Budgeted annual production (units) 15,000 24,000 20,000

$ per unit $ per unit $ per unit


Direct materials 35 45 30
Direct labour ($10 per hour) 40 30 50

In addition to the above direct costs, KL incurs annual indirect production costs of
$1,044,000.

Required:

(b) Calculate the full cost per unit of each product using KLs current method of
absorption costing.
(4 marks)

65 | P a g e
An analysis of the companys indirect production costs shows the following:

$ Cost driver
Material ordering costs 220,000 Number of supplier orders
Machine setup costs 100,000 Number of batches
Machine running costs 400,000 Number of machine hours
General facility costs 324,000 Number of machine hours

The following additional data relate to each product:

Product W X Y
Machine hours per unit 5 8 7
Batch size (units) 500 400 1,000
Supplier orders per batch 4 3 5

Required:

(c) Calculate the full cost per unit of each product using Activity Based Costing.
(8 marks)

(d) Explain how Activity Based Costing could provide information that would be relevant
to the management team when it is making decisions about how to improve KLs
profitability.
(7 marks)

(Total = 25 marks)

B2 4 Retail outlet (CIMA P2 Nov 2007)

A small retail outlet sells four main groups of products: Basic Foods (milk, bread, etc);
Newspapers & Magazines; Frozen Foods; and Canned Foods. A budgeted weekly profit
statement is shown below:

Basic Foods Newspapers Frozen Foods Canned Foods


and Magazines

$ $ $ $
Sales revenue 800 1,000 1,500 2,400
Cost of sales 600 700 550 1,200
Gross margin 200 300 950 1,200
Power for freezers* 100
Overheads** 100 100 200 400
Net margin 100 200 650 800

*The freezers would be emptied and switched off as necessary during redecoration.

66 | P a g e
**Overhead costs comprise general costs of heating and lighting, rent and rates, and other
general overhead costs. These costs are attributed to products in proportion to the floor
area occupied by each product group which is as follows:

Basic Foods Newspapers Frozen Foods Canned foods


and Magazines
Floor area (m2) 50 50 100 200

For each product group, analysis has shown that the sales revenue achieved changes in
direct proportion to the floor space allocated to the product.

The owner of the retail outlet has decided that the premises need to be redecorated but is
undecided as to which of the following two options would be the most profitable.

Option 1
Close the retail outlet completely for four weeks while the redecoration takes place. The
company that is to complete the redecoration would charge $2,500 under this option. It is
expected that following the re-opening of the retail outlet there would be a loss of sales
for the next 12 weeks because customers would have had to find alternative suppliers for
their goods. The reduction in sales due to lost customers has been estimated to be 30% of
the budgeted sales during the first four weeks of reopening; 20% during the next four
weeks; and 10% during the third four weeks. In addition, in order to encourage customers
to return to the retail outlet, there would be a 10% price reduction on all Basic Foods and
Canned Foods for the entire 12 week period.

Option 2
Continue to open the retail outlet while the redecoration takes place but with a reduced
amount of floor area. The useable floor area would be reduced to 40% of that originally
available. After three weeks, the retail outlet would be closed for 05 weeks while the
goods are moved to the newly redecorated area. The retail outlet would then continue to
operate using 40% of its original floor area for a further three weeks before the work was
fully completed. The company that is to complete the redecoration would charge $3,500
under this option, and in addition there would be product movement costs of $1,000. The
owner has determined that in order to avoid losing customers there should be no
reduction in the amount of floor area given to Basic Foods and Newspapers and
Magazines throughout this period. The floor area to be used by Frozen Foods and Canned
Foods should be determined on the basis of their profitability per unit of area. However,
the Frozen Foods are presently kept in four freezers, and therefore any reductions in floor
area must be determined by complete freezer units. It may be assumed that each freezer
unit incurs equal amounts of power costs.

67 | P a g e
Required:

(a) Advise the owner of the retail outlet which option to choose in order to minimise the
losses that will occur as a result of the decision. All workings must be shown.

(15 marks)

(b) Explain how Activity Based Costing may be used in a retail environment to improve
the decision making and profitability of the business.
(10 marks)

(Total = 25 marks)

68 | P a g e
Questions Section A

Part C Budgeting and Management Control

C1 -1 Solicitors firm (CIMA P2 May 2010)

A firm of solicitors is using budgetary control during 2010. The senior partner estimated
the demand for the year for each of the firms four divisions: Civil, Criminal, Corporate,
and Property. A separate partner is responsible for each division.

Each divisional partner then prepared a cost budget based on the senior partners demand
estimate for the division. These budgets were then submitted to the senior partner for his
approval. He then amended them as he thought appropriate before issuing each divisional
partner with the final budget for the division. He did not discuss these amendments with
the respective divisional partners. Actual performance is then measured against the final
budgets for each month and each divisional partners performance is appraised by asking
the divisional partner to explain the reasons for any variances that occur.

The Corporate partner has been asked to explain why her staff costs exceeded the
budgeted costs for last month while the chargeable time was less than budgeted. Her
reply is below:

My own original estimate of staff costs was higher than the final budgeted costs shown
on my divisional performance report. In my own cost budget I allowed for time to be
spent developing new services for the firms corporate clients and improving the clients
access to their own case files. This would improve the quality of our services to clients
and therefore increase client satisfaction. The trouble with our present system is that it
focuses on financial performance and ignores the other performance indicators found in
modern performance management systems.

Required:

(a) Discuss the present budgeting system and its likely effect on divisional partner
motivation.
(6 marks)

(b) Explain two non-financial performance indicators (other than client satisfaction and
service quality) that could be used by the firm.
(4 marks)
(Total = 10 marks)

69 | P a g e
C1 2 DW (CIMA P2 Nov 2010)

DW, a transport company, operates three depots. Each depot has a manager who reports
directly to the Operations Director.

For many years the depot managers have been asked by the Operations Director to
prepare a budget for their depot as part of the companys annual budgeting process. A
new depot manager has been appointed to the Southern region and he has concerns about
the validity of these annual budgets. He argues that they soon become out of date as
operational circumstances change. At a recent managers meeting he said, They are
restrictive. They do not permit the depot managers to make decisions in response to
operational changes, or change working practices for next year until that years budget
has been approved.

Required:
(a) Explain the differences between the above annual budgeting system and a rolling
budget system.
(4 marks)

(b) Discuss how the Southern region depot manager could use a rolling budget system to
address his concerns.
(6 marks)

(Total = 10 marks)

C1 3 JYT (CIMA P2 May 2011)

JYT manufactures and sells a range of products. It is not dominant in the market in which
it operates and, as a result, it has to accept the market price for each of its products. The
company is keen to ensure that it continues to compete and earn satisfactory profit at each
stage throughout a products life cycle.

Required:

Explain how JYT could use Target Costing AND Kaizen Costing to improve itsfuture
performance.

Your answer should include an explanation of the differences between Target Costing
and Kaizen Costing.

(Total = 10 marks)

70 | P a g e
C1 4 DVD (CIMA P2 May 2011)

A company produces and sells DVD players and Blu-ray players.

Extracts from the budget for April are shown in the following table:

Sales Selling price Standard cost


(players) (per player) (per player)
DVD 3,000 $75 $50
Blu-ray 1,000 $200 $105

The Managing Director has sent you a copy of an e-mail she received from the Sales
Manager. The content of the e-mail was as follows:

We have had an excellent month. There was an adverse sales price variance on the DVDs
of $18,000 but I compensated for that by raising the price of Blu-ray players. Unit sales
of DVD players were as expected but sales of the Blu-rays were exceptional and gave a
total sales volume profit variance of $19,000. I think I deserve a bonus!

The Managing Director has asked for your opinion on these figures. You obtained the
following information:

Actual results for April were:

Sales Selling price


(players) (per player)
DVD 3,000 $69
Blu-ray 1,200 $215

The total market demand for DVD players was as budgeted but as a result of distributors
reducing the price of Blu-ray discs the total market for Blu-ray players grew by 50% in
April.

The company had sufficient capacity to meet the revised market demand for 1,500 units
of its Blu-ray players and therefore maintained its market share.

Required:

(a) Calculate the following operational variances based on the revised market details:

(i) The total sales mix profit margin variance


(2 marks)

(ii) The total sales volume profit variance


(2 marks)

71 | P a g e
(b) Explain, using the above scenario, the importance of calculating planning and
operational variances for responsibility centres.
(6 marks)
(Total = 10 marks)

C1 5 SFG (CIMA P2 May 2011)

SFG is a national hotel group that operates more than 100 hotels. The performance of the
manager of each hotel is evaluated using financial measures.

Many of the hotels managers are not happy. They believe that there can be conflict
between good performance and achieving short-term profits. They are also unhappy that
their profit reports include a share of head office costs and other costs that they cannot
control.

Required:

(a) Explain why non-financial performance measures are important in the service sector.
(2 marks)

(b) Recommend, with reasons, TWO non-financial performance measures that SFG could
use to evaluate the performance of the hotel managers.
(4 marks)

(c) Explain why, and how, non-controllable costs should be shown on the profit reports.
(4 marks)

(Total = 10 marks)

C1 6 Feedback and feedforward (CIMA P1 Pilot Paper 2005)

Briefly outline the main features of feedback control, and the feedback loopand
explain how, in practice, the procedures of feedback control can be transformed into
feed-forward control.
(5 marks)

C1 7 Profit centre managers (CIMA P1 Pilot Paper 2005)

(c) Briefly outline the advantages and disadvantages of allowing profit centre managers
to participate actively in the setting of the budget for their units.
(5 marks)

72 | P a g e
C1 8 Balance scorecard (CIMA P1 May 2005)

A general insurance company is about to implement a Balanced Scorecard.

You are required to:

(i) State the four perspectives of a Balanced Scorecard; and

(ii) Recommend one performance measure that would be appropriate for a general
insurance company, for each of the four perspectives, and give a reason to support each
measure. (You must recommend one measure only for each perspective.)
(5 marks)

C1 9 Participation in budgets (CIMA P1 May 2005)

Briefly discuss three different circumstances where participation in setting budgets is


likely to contribute to poor performance from managers.
(5 marks)

C1 10 Beyond budgeting (CIMA P1 May 2005)

W Limited designs and sells computer games. There are many other firms in this
industry. For the last five years the senior management has required detailed budgets to
be produced for each year with slightly less detailed plans for the following two years.
The managing director of W Limited has recently attended a seminar on budgeting and
heard the Beyond Budgeting arguments that have been advanced by Hope and Fraser,
among others.

You are required to:

(i) Briefly describe the Beyond Budgeting approach; and


(2 marks)

(ii) Advise the management of W Limited whether or not it should change its current
budgeting system to a Beyond Budgeting approach.
(3 marks)

(Toal = 10 marks)

73 | P a g e
C1 11 J Limited (CIMA P1 Nov 2005)

J Limited has recently been taken over by a much larger company. For many years the
budgets in J have been set by adding an inflation adjustment to the previous years
budget. The new owners of J are insisting on a zero-base approach when the next
budget is set, as they believe many of the indirect costs in J are much higher than in
othercompanies under their control.

(i) Explain the main features of zero-based budgeting.


(2 marks)

(ii) Discuss the problems that might arise when implementing this approach in J Limited.
(3 marks)

(Total = 5 marks)

C1 12 ST plc (CIMA P1 Nov 2005)

ST plc is a medium-sized engineering company using advanced technology. It has just


implemented an integrated enterprise resource planning (ERP) system in place of an old
MRP (manufacturing resource planning) system.

Discuss the changes that are likely to be seen after the implementation of the ERP system
in:

(i) the budget-setting process; and


(ii) the budgetary control process
(5 marks)

C1 13 W Limited (CIMA P1 Nov 2005)

W Limited has conducted a review of its budget-setting procedures. The review


coordinator frequently heard the following comment from staff interviewed:

Its impossible to make this system work because senior managers want budgets to be a
challenging target whereas the finance department require an accurate forecast.

Discuss the issues raised in this comment, and advise the review coordinator on practical
action that could be taken to alleviate the situation described.
(5 marks)

74 | P a g e
C1 14 T plc (CIMA P1 May 2006)

T plc is a large insurance company. The Claims Department deals with claims from
policy holders who have suffered a loss that is covered by their insurance policy. Policy
holders could claim, for example, for damage to property, or for household items stolen
in a burglary. The Claims Department staff investigate each claim and determine what, if
any, payment should be made to the claimant.

The manager of the Claims Department has decided to benchmark the performance of the
department and has chosen two areas to benchmark:

the detection of false claims


the speed of processing claims

For each of the above two areas:


(i) state and justify a performance measure
(ii) explain how relevant benchmarking data could be gathered.
(5 marks)

C1 15 Product M (CIMA P1 May 2007)

A company uses variance analysis to monitor the performance of the team of workers
which assembles Product M. Details of the budgeted and actual performance of the team
for last period were as follows:

Budget Actual
Output of product M 600 units 680 units
Wage rate 30 per hour 32 per hour
Labour hours 900 hours 1,070 hours

It has now been established that the standard wage rate should have been 31.20 per
hour.

Required:

(i) Calculate the labour rate planning variance and calculate the operational labour
efficiency variance.

(ii) Explain the major benefit of analysing variances into planning and operational
components.
(5 marks)

75 | P a g e
C1 16 QBD (CIMA P1 Nov 2007)

QBQ produces one type of product. Details of the budgeted sales and production are
given below.

Selling Price and Costs per unit


Selling price 40
Material FX: 1.5kg @ 6 per kg 9
Conversion costs (variable) 8
Fixed production overheads 15

The fixed production overhead absorption rate is based on annual production overheads
of 720,000 and budgeted annual output of 48,000 units. The fixed overheads will be
incurred evenly throughout the year.

The company also incurs fixed costs for administration of 200,000 per year.

Budgeted Sales

Quarter Units
1 10,000
2 12,000
3 14,000
4 12,000

Inventory

It has been decided that inventory levels are to be reduced. Details are as follows:

Finished goods: 5,500 units are currently held but it has been decided that the closing
inventories for Quarters 1, 2 and 3 will be 45%, 40% and 35% of the following quarters
sales respectively.

Raw materials: 4,500 kg are currently held but it has been decided that the closing
inventories for Quarters 1 and 2 will be 25% and 20% of the followingquarters
production requirements respectively.

Required:

(a) Prepare a materials purchase budget for Quarter 1.


(5 Marks)

76 | P a g e
C1 17 Budgetary planning and control (CIMA P1 Nov 2007)

Briefly explain three reasons why budgetary planning and control might be inappropriate
in a rapidly changing business environment.
(5 Marks)

C1 18 JIT systems (CIMA P1 Nov 2007)


Briefly explain Just-in-Time (JIT) and two major requirements for the successful
operation of a JIT system.
(5 Marks)

C1 19 Feedback and forward (CIMA P1 Nov 2007)

Explain, giving examples, how budgets can be used for feedback control and feed-
forward control.
(5 Marks)

C1 20 Nursing homes (CIMA P1 Nov 2007)

A nursing home uses incremental budgeting. The previous periods budget is adjusted by
reference to a set of indices. It is adjusted firstly for volume changes and then for
changes in the cost of resources. The indices are referenced to the previous periods
budget by using that budget as the base index number of 100. The index numbers to be
used to prepare Period 3s budget from that of Period 2 are as follows:

Index
Patient days 90
House-keeping costs 106
Nursing costs 105
Administration costs 104

The budget for Period 2 was:



House-keeping costs (all variable) 125,000
Nursing costs (see below) 324,000
Administration costs (all fixed) 100,000

Nursing costs are semi-variable. The nursing costs for Period 2 were adjusted from the
total nursing costs of 280,000 for Period 1 by using a Patient days index of 125 and a
Nursing costs index of 108.

77 | P a g e
Required:

Prepare the budget for Period 3.


(5 marks)

C1 21 Participative budgeting (CIMA P1 May 2008)

Explain THREE behavioural consequences that may result after the introduction of
participative budgeting.
(5 marks)

C1 - 22 Rolling budgets (CIMA P1 May 2008)

Discuss the advantages and disadvantages of rolling budgets.


(5 marks)

78 | P a g e
Questions Section B

Part C Budgeting and Management Control

C2 -1 M plc (CIMA P1 May 2006)

M plc designs, manufactures and assembles furniture. The furniture is for home use and
therefore varies considerably in size, complexity and value. One of the departments in the
company is the Assembly Department. This department is labour intensive; the workers
travel to various locations to assemble and fit the furniture using the packs of finished
timbers that have been sent to them.

Budgets are set centrally and they are then given to the managers of the various
departments who then have the responsibility of achieving their respective targets. Actual
costs are compared against the budgets and the managers are then asked to comment on
the budgetary control statement. The statement for April for the Assembly Department is
shown below.

Budget Actual Variance


Assembly labour hours 6,400 7,140
$ $ $
Assembly labour 51,970 58,227 6,257 Adverse
Furniture packs 224,000 205,000 19,000 Favourable
Other materials 23,040 24,100 1,060 Adverse
Overheads 62,060 112,340 50,280 Adverse
Total 361,070 399,667 38,597 Adverse

Note: the costs shown are for assembling and fitting the furniture (they do not include
time spent travelling to jobs and the related costs). The hours worked by the Manager are
not included in the figure given for the assembly labour hours.

The Manager of the Assembly Department is new to the job and has very little previous
experience of working with budgets but he does have many years experience as a
supervisor in assembly departments. Based on that experience he was sure that the
department had performed well. He has asked for your help in replying to a memo he has
just received asking him to explain the serious overspending in his department. He has
sent you some additional information about the budget:

1. The budgeted and actual assembly labour costs include the fixed salary of $2,050
for the Manager of the Assembly Department. All of the other labour is paid for
the hours they work.

2. The cost of furniture packs and other materials is assumed by the central finance
office of M plc to vary in proportion to the number of assembly labour hours
worked.

79 | P a g e
3. The budgeted overhead costs are made up of three elements: a fixed cost of
$9,000 for services from central headquarters, a stepped fixed cost which changes
when the assembly hours exceed 7,000 hours, and some variable overheads. The
variable overheads are assumed to vary in proportion to the number of assembly
labour hours. Working papers for the budget showed the impact on the overhead
costs of differing amounts of assembly labour hours:

Assembly labour hours 5,000 7,500 10,000


Overhead costs $54,500 $76,500 $90,000

The actual fixed costs for April were as budgeted.

Required:

(a) Prepare, using the additional information that the Manager of the Assembly
Department has given you, a budgetary control statement that would be more helpful to
him.
(7 marks)

(b)

(i) Discuss the differences between the format of the statement that you have produced
and that supplied by M plc.
(4 marks)

(ii) Discuss the assumption made by the central office of M plc that costs vary in
proportion to assembly labour hours.
(3 marks)

Discuss whether M plc should change to a system of participative budgeting.


(6 marks)

(Total = 20 marks)

C2 2 RF Ltd (CIMA P1 May 2007)

RF Ltd is a new company which plans to manufacture a specialist electrical component.


The company founders will invest 16,250 on the first day of operations, that is, Month
1. They will also transfer fixed capital assets to the company.

The following information is available:

80 | P a g e
Sales
The forecast sales for the first four months are as follows:

Month Number of
components
1 1,500
2 1,750
3 2,000
4 2,100
The selling price has been set at 10 per component in the first four months.

Sales receipts
Time of payment % of customers
Month of sale 20*
One month later 45
Two months later 25
Three months later 5

The balance represents anticipated bad debts.

*A 2% discount is given to customers for payment received in the month of sale.

Production
There will be no opening inventory of finished goods in Month 1 but after that it will be
policy for the closing inventory to be equal to 20% of the following months forecast
sales.

Variable production cost


The variable production cost is expected to be 6.40 per component.

Time of payment
Direct materials 1.90
Direct wages 3.30
Variable production overheads 1.20
Total variable cost 6.40

Notes:

Direct materials: 100% of the materials required for production will be purchased in the
month of production. No inventory of materials will be held. Direct materials will be paid
for in the month following purchase.

Direct wages will be paid in the month in which production occurs.

81 | P a g e
Variable production overheads: 60% will be paid in the month in which production
occurs and the remainder will be paid one month later.

Fixed overhead costs


Fixed overhead costs are estimated at 75,000 per annum and are expected to be incurred
in
equal amounts each month. 60% of the fixed overhead costs will be paid in the month in
which they are incurred and 30% in the following month. The balance represents
depreciation of fixed assets.

Calculations are to be made to the nearest 1.

Ignore VAT and Tax.

Required:

(a) Prepare a cash budget for each of the first three months and in total.
(15 marks)

(b) There is some uncertainty about the direct material cost. It is thought that the direct
material cost per component could range between 1.50 and 2.20.

Calculate the budgeted total net cash flow for the three month period if the cost of the
direct material is:

(i) 1.50 per component; or


(ii) 2.20 per component.
(6 marks)

(c) Using your answers to part (a) and (b) above, prepare a report to the management of
RF Ltd that discusses the benefits or otherwise of performing what if analysis when
preparing cash budgets.
(9 marks)

(Total = 30 marks)

C2 3 Trackit (CIMA P1 May 2008)

Q, a new company, is being established to manufacture and sell an electronic tracking


device: the Trackit. The owners are excited about the future profits that the business will
generate. They have forecast that sales will grow to 2,600 Trackits per month within five
months and will be at that level for the remainder of the first year.

The owners will invest a total of $250,000 in cash on the first day of operations (that is
the first day of Month 1). They will also transfer non-current assets into the company.

82 | P a g e
Extracts from the companys business plan are shown below.

Sales
The forecast sales for the first five months are:

Month Trackits
(units)
1 1,000
2 1,500
3 2,000
4 2,400
5 2,600

The selling price has been set at $140 per Trackit.

Sales receipts
Sales will be mainly through large retail outlets. The pattern for the receipt of payment is
expected to be as follows:

Time of payment % of sales value


Immediately 15 *
One month later 25
Two months later 40
Three months later 15

The balance represents anticipated bad debts.

* A 4% discount will be given for immediate payment.

Production
The budget production volumes in units are:

Month 1 Month 2 Month 3 Month 4


1,450 1,650 2,120 2,460

Variable production cost


The budgeted variable production cost is $90 per unit, comprising:
$
Direct materials 60
Direct wages 10
Variable production overheads 20
Total variable cost 90

83 | P a g e
Direct materials: Payment for purchases will be made in the month following receipt.
There will be no opening inventory of materials in Month 1. It will be company policy to
hold inventory at the end of each month equal to 20% at of the following months
production requirements. The direct materials cost includes the cost of an essential
component that will be bought in from a specialist manufacturer.

Direct wages will be paid in the month in which the production occurs.

Variable production overheads: 65% will be paid in the month in which production
occurs and the remainder will be paid one month later.

Fixed overhead costs


Fixed overheads are estimated at $840,000 per annum and are expected to be incurred in
equal amounts each month. 60% of the fixed overhead costs will be paid in the month in
which they are incurred and 15% in the following month. The balance represents
depreciation of non-current assets.

Ignore VAT and Tax

Required:

(a) Prepare a cash budget for each of the first three months and for that three-month
period in total.
(14 marks)

(b) There is some uncertainty about the cost of the specialist component (this is included
in the direct material cost). It is thought that the cost of the component could range
between $32 and $50 per Trackit. It is currently included in the cost estimates at $40 per
Trackit.

Calculate the budgeted total net cash flow for the three-month period in total if the cost of
the component was:

(i) $32
(ii) $50

(6 marks)

(c) Prepare a report for the owners of Q that offers advice about the profitability of their
business and the situation revealed by the extracts from the business plan and your
answers to (a) and (b) above.
(10 marks)

(Total = 30 marks)

84 | P a g e
C2 4 X plc (CIMA P1 Nov 2006)

X Plc manufactures specialist insulating products that are used in both residential and
commercial buildings. One of the products, Product W, is made using two different raw
materials and two types of labour. The company operates a standard absorption costing
system and is now preparing its budgets for the next four quarters. The following
information has been identified for Product W:

Sales
Selling price is 220 per unit

Sales demand
Quarter 1 2,250 units
Quarter 2 2,050 units
Quarter 3 1,650 units
Quarter 4 2,050 units
Quarter 5 1,250 units
Quarter 6 2,050 units

Costs
Material A is 5 kgs per unit @ 4 per kg
Material B is 3 kgs per unit @ 7 per kg

Labour skilled is 4 hours per unit @ 15 per hour


Labour semi-skilled is 6 hours per unit @ 9 per hour

Annual overheads 280,000 40% of these overheads are fixed and the remainder varies
with total labour hours. Fixed overheads are absorbed on a unit basis.

Inventory holding policy


Closing inventory of finished goods is 30% of the following quarters sales demand.
Closing inventory of materials is 45% of the following quarters materials usage.

The management team are concerned that X Plc has recently faced increasing
competition in the market place for Product W. As a consequence there have been issues
concerning the availability and costs of the specialised materials and employees needed
to manufacture Product W, and there is concern that these might cause problems in the
current budget setting process.

Required:

(a) Prepare the following budgets for each quarter for X Plc:
(i) Production budget in units;
(ii) Raw material purchases budget in kgs and value for Material B.
(5 marks)

85 | P a g e
(b) X Plc has just been informed that Material A may be in short supply during the year
for which it is preparing budgets. Discuss the impact this will have on budget preparation
and other areas of X Plc.
(5 marks)

(c) Assuming that the budgeted production of Product W was 7,700 units and that the
following actual results were incurred for labour and overheads in the year:

Actual production 7,250 units

Actual overheads
Variable 185,000
Fixed 105,000

Actual labour costs


Skilled - 16.25 per hour 568,750
Semi-skilled - 8 per hour 332,400

Prepare a flexible budget statement for X Plc showing the total variances that have
occurred for the above four costs only.
(5 marks)

(d) X Plc currently uses incremental budgeting. Explain how Zero Based Budgeting
could overcome the problems that might be faced as a result of the continued use of the
current system.
(5 marks)

(e) Explain how rolling budgets are used and why they would be suitable for X Plc.
(5 marks)

(Total = 25 marks)

86 | P a g e
Questions Section A

Part D Control and Performance Measurement of Responsibilty Centres

D1 -1 EVA and RI (CIMA P2 Pilot paper 2005)

Explain and discuss the similarities and differences between Residual Income and
Economic Value Added as methods for assessing the performance of divisions.
(5 marks)

D1 2 Controllability principle (CIMA P1 Pilot Paper 2005)

Define the controllability principle and give arguments for and against its
implementation in determining performance measures.
(5 marks)

D1 3 Transfer pricing (CIMA P1 Pilot Paper 2005)

Discuss the problems that arise specifically when determining transfer prices where
divisions are located in different countries.
(5 marks)

D1 4 EVA (CIMA P1 May 2005)

(i) Briefly explain the main features of Economic Value Added (EVA) as it would be
used to assess the performance of divisions.
(2 marks)

(ii) Briefly explain how the use of EVA to assess divisional performance might affect
the behaviour of divisional senior executives.
(3 marks)

D1 5 WD, PD & TD (CIMA P1 May 2005)

C plc is a large company that manufactures and sells wooden garden furniture. It has
three divisions:

The Wood Division (WD) purchases logs and produces finished timber as planks or
beams. Approximately two-thirds of its output is sold to the Products Division, with the
remainder sold on the open market.

87 | P a g e
The Products Division (PD) manufactures wooden garden furniture. The policy of C plc
is that the PD must buy all its timber from the WD and sell all its output to the Trading
Division.

The Trading Division (TD) sells wooden garden furniture to garden centres, large
supermarkets, and similar outlets. It only sells items purchased from PD.

The current position is that all three divisions are profit centres and C plc uses Return on
Investment (ROI) measures as the primary means to assess divisional performance. Each
division adopts a cost-plus pricing policy for external sales and for internal transfers
between divisions. The senior management of C plc has stated that the divisions should
consider themselves to be independent businesses as far as possible.

(i) For each division suggest, with reasons, the behavioural consequences that might arise
as a result of the current policy for the structure and performance evaluation of the
divisions.
(5 marks)

(ii) The senior management of C plc has requested a review of the cost-plus transfer
pricing policy that is currently used.

Suggest with reasons, an appropriate transfer pricing policy that could be used for
transfers from PD to TD, indicating any problems that may arise as a consequence of the
policy you suggest.
(5 marks)

D1 6 G group (CIMA P1 May 2007)

G Group consists of several autonomous divisions. Two of the divisions supply


components and services to other divisions within the group as well as to external clients.
The management of G Group is considering the introduction of a bonus scheme for
managers that will be based on the profit generated by each division.

Required:

Briefly explain the factors that should be considered by the management of G


Groupwhen designing the bonus scheme for divisional managers.
(5 marks)

88 | P a g e
D1 7 Digital equipment (CIMA P1 May 2008)

A multi-national company manufactures and sells a wide range of digital equipment. The
company is structured into three Divisions: Computers, Audio-visual and Photographic.
The Divisions operate as investment centres and the performance of the Divisional
Managers is evaluated by using Return on Investment (ROI).

The Manager of the Photographic Division was concerned that the Division was falling
behind its competitors in terms of financial returns and market share, and has
implemented strategies to improve the situation. An external benchmarking exercise was
undertaken to try to establish the position of the Division in relation to its competitors in
a number of key areas. It has now been suggested that the Division should also carry out
an internal benchmarking exercise.

The manager of the Photographic Division is considering introducing a Balanced


Scorecard to measure the success of the strategies. He has identified two perspectives and
two associated goals. They are:

Perspective Goal
Innovation Technology Leadership
Customer Support

Required:

(a) For the Innovation Perspective of the Division, recommend a performance measure
and briefly explain how the measure will reflect the achievement of the stated goal.
(3 marks)

(b) For the Customer Perspective of the Division, state which data should be collected
and explain how this could be used to ensure the goal of support is met.
(2 marks)

(c) Explain THREE reasons why internal benchmarking may provide information that is
more useful to the Manager of the Photographic Division, in terms of monitoring and
improving performance, than that provided by external benchmarking.
(5 marks)

(d) Explain THREE reasons why ROI may not be a good performance measure.
(5 marks)

(Total = 15 marks)

89 | P a g e
Questions Section B

Part D Control and Performance Measurement of Responsibilty Centres

D2 -1 Y and Z (CIMA P1 Nov 2005)

Y and Z are two divisions of a large company that operate in similar markets. The
divisions are treated as investment centres and every month they each prepare an
operating statement to be submitted to the parent company. Operating statements for
these two divisions for October are shown below:

Operating Statements for October


Y Z
000 000
Sales revenue 900 555
Less variable costs 345 312
Contribution 555 243
Less controllable fixed costs 95 42
(includes depreciation on divisional assets)
Controllable income 460 201
Less apportioned central costs 338 180
Net income before tax 122 21

Total divisional net assets 976m 126m

The company currently has a target return on capital of 12% per annum. However, the
company believes its cost of capital is likely to rise and is considering increasing the
target return on capital. At present the performance of each division and the divisional
management are assessed primarily on the basis of Return on Investment (ROI).

Required:

(a) Calculate the annualised Return on Investment (ROI) for divisions Y and Z, and
discuss the relative performance of the two divisions using the ROI data and other
information given above.
(9 marks)

(b) Calculate the annualised Residual Income (RI) for divisions Y and Z, and explain
the implications of this information for the evaluation of the divisions
performance.
(6 marks)

90 | P a g e
(c) Briefly discuss the strengths and weaknesses of ROI and RI as methods of
assessing the performance of divisions. Explain two further methods of
assessment of divisional performance that could be used in addition to ROI or RI.
(5 marks)

(Total = 20 marks)

D2 2 FP (CIMA P1 May 2006)

FP sells and repairs photocopiers. The company has operated for many years with two
departments, the Sales Department and the Service Department, but the departments had
no autonomy. The company is now thinking of restructuring so that the two departments
will become profit centres.

The Sales Department


This department sells new photocopiers. The department sells 2,000 copiers per year.
Included in the selling price is 60 for a one year guarantee. All customers pay this fee.
This means that during the first year of ownership if the photocopier needs to be repaired
then the repair costs are not charged to the customer. On average 500 photocopiers per
year need to be repaired under the guarantee. The repair work is carried out by the
Service Department who, under the proposed changes, would charge the Sales
Department for doing the repairs. It is estimated that on average the repairs will take 3
hours each and that the charge by the Service Department will be 136,500 for the 500
repairs.

The Service Department


This department has two sources of work: the work needed to satisfy the guarantees for
the Sales Department and repair work for external customers. Customers are charged at
full cost plus 40%. The details of the budget for the next year for the Service Department
revealed standard costs of:

Parts at cost
Labour 15 per hour
Variable overheads 10 per labour hour
Fixed overheads 22 per labour hour

The calculation of these standards is based on the estimated maximum market demand
and includes the expected 500 repairs for the Sales Department. The average cost of the
parts needed for a repair is 54. This means that the charge to the Sales Department for
the repair work, including the 40% mark-up, will be 136,500.

91 | P a g e
Proposed Change
It has now been suggested that FP should be structured so that the two departments
become profit centres and that the managers of the Departments are given autonomy. The
individual salaries of the managers would be linked to the profits of their respective
departments. Budgets have been produced for each department on the assumption that the
Service Department will repair 500 photocopiers for the Sales Department and that the
transfer price for this work will be calculated in the same way as the price charged to
external customers. However the manager of the Sales Department has now stated that he
intends to have the repairs done by another company, RS, because they have offered to
carry out the work for a fixed fee of 180 per repair and this is less than the price that the
Sales Department would charge.

Required:

(a) Calculate the individual profits of the Sales Department and the Service Department,
and of FP as a whole from the guarantee scheme if:

(i) The repairs are carried out by the Service Department and are charged at full cost
plus 40%;

(ii) The repairs are carried out by the Service department and are charged at marginal
cost;

(iii)The repairs are carried out by RS.


(8 marks)

(b)
(i) Explain, with reasons, why a full cost plus transfer pricing model may not be
appropriate for FP.
(3 marks)

(ii) Comment on other issues that the managers of FP should consider if they decide
to allow RS to carry out the repairs.
(4 marks)

(c) Briefly explain the advantages and disadvantages of structuring the departments as
profit centres.
(5 marks)

(Total = 20 marks)

92 | P a g e
D2 3 ZZ group (CIMA P1 Nov 2006)

The ZZ Group has two divisions, X and Y. Each division produces only one type of
product: X produces a component I and Y produces a finished product (FP). Each FP
needs one C. It is the current policy of the group for C to be transferred to Division Y at
the marginal cost of 10 per component and that Y must buy all the components it needs
from X. The markets for the component and the finished product are competitive and
price sensitive. Component C is produced by many other companies but it is thought that
the external demand for the next year could increase to 1,000 units more than the sales
volume shown in the current budget for Division X. Budgeted data, taken from the ZZ
Group Internal Information System, for the divisions for the next year is as follows:

Division X
Income statement
Sales 70,000
Cost of sales
Variable costs 50,000
Contribution 20,000
Fixed costs (controllable) 15,000
Profit 5,000

Production/Sales (units) 5,000 (3,000 of which are transferred to Division Y)


External demand (units) 3,000 (Only 2,000 of which can be currently satisfied)
Capacity (units) 5,000
External market price per unit 20

Balance sheet extract


Capital employed 60,000

Other information
Cost of capital charge 10%

Division Y
Income statement
Sales 270,000
Cost of sales
Variable costs 114,000
Contribution 156,000
Fixed costs (controllable) 100,000
Profit 56,000

Production/Sales (units) 3,000


Capacity (units) 7,000
Market price per unit 90

93 | P a g e
Division Y
Balance sheet extract
Capital employed 110,000

Other information
Cost of capital charge 10%

Four measures are used to evaluate the performance of the Divisional Managers. Based
on the data above, the budgeted performance measures for the two divisions are as
follows:

Division X Division Y
Residual income (1,000) 45,000
Return on capital employed 833% 5091%
Operating profit margin 714% 2074%
Asset turnover 117 246

Current policy

It is the current policy of the group for C to be transferred to Division Y at the marginal
cost of 10 per component and that Y must buy all the components that it needs from X.

Proposed policy

ZZ Group is thinking of giving the Divisional Managers the freedom to set their own
transfer price and to buy the components from external suppliers but there are concerns
about problems that could arise by granting such autonomy.

Required:

(a) If the transfer price of the component is set by the Manager of Division X at the
current market price (20 per component), recalculate the budgeted performance
measures for each division.
(8 marks)

(b) Discuss the changes to the performance measures of the divisions that would arise
as a result of altering the transfer price to 20 per component.
(6 marks)

(c)

(i) Explain the problems that could arise for each of the Divisional Managers and for
ZZ Group as a whole as a result of giving full autonomy to the Divisional
Managers.

94 | P a g e
(ii) Discuss how the problems you have explained could be resolved without resorting
to a policy of imposed transfer prices.
(6 marks)

(Total = 20 marks)

D2 4 Computer manufacturer (CIMA P1 Nov 2007)

A multinational computer manufacturer has a number of autonomous subsidiaries


throughout the world. Two of the groups subsidiaries are in America and Europe. The
American subsidiary assembles computers using chips that it purchases from local
companies. The European subsidiary manufactures exactly the same chips that are used
by the American subsidiary but currently only sells them to numerous external companies
throughout Europe.

Details of the two subsidiaries are given below.

America
The American subsidiary buys the chips that it needs from a local supplier. It has
negotiated a price of $90 per chip. The production budget shows that 300,000 chips will
be needed next year.

Europe
The chip production subsidiary in Europe has a capacity of 800,000 chips per year.
Details of the budget for the forthcoming year are as follows:

Sales 600,000 chips


$ per chip
Selling price 105
Variable costs 60

The fixed costs of the subsidiary at the budgeted output of 600,000 chips are $20 million
per year but they would rise to $26 million if output exceeds 625,000 chips.

Note: The maximum external demand is 600,000 chips per year and the subsidiary has no
other uses for the current spare capacity.

Group Directive
The Managing Director of the group has reviewed the budgets of the subsidiaries and has
decided that in order to improve the profitability of the group the European subsidiary
should supply chips to the American subsidiary. She is also thinking of linking the
salaries of the subsidiary managers to the performance of their subsidiaries but is unsure
which performance measure to use.

95 | P a g e
Two measures that she is considering are profit and the return on assets consumed
(where the annual fixed costs would be used as the assets consumed).

The Manager of the European subsidiary has offered to supply the chips at a price of $95
each. He has offered this price because it would earn the same contribution per chip that
would be earned on external sales (this is after adjusting for increased distribution costs
and reduced customer servicing costs).

Required:

(a) Assume that the 300,000 chips are supplied by the European subsidiary at a transfer
price of $95 per chip. Calculate the impact of the profits on each of the subsidiaries and
the group. (5 marks)

(b) Calculate the minimum unit price at which the European subsidiary would be willing
to transfer the 300,000 chips to the American subsidiary if the performance and salary of
the Manager of the subsidiary is to be based on

(i) the profit of the subsidiary (currently $7 million)


(ii) the return on assets consumed by the subsidiary (currently 35%).
(9 marks)

(c) Write a report to the Managing Director of the group that discusses issues raised by
the directive and the introduction of performance measures. (You should use your
answers to parts (a) and (b), where appropriate, to illustrate points in your report).
(10 marks)

(d) Briefly explain how multi-national companies can use transfer pricing to reduce their
overall tax charge and the steps that national tax authorities have taken to discourage the
manipulation of transfer prices.
(6 marks)

(Total = 30 marks)

D2 5 Perfumes and cosmetics (CIMA P2 May 2010)

H manufactures perfumes and cosmetics by mixing various ingredients in different


processes, before the items are packaged and sold to wholesalers. H uses a divisional
structure with each process being regarded as a separate division with its own manager
who is set performance targets at the start of each financial year which begins on 1
January. Performance is measured using Return on Investment (ROI) based on net book
value of capital equipment at the start of the year. The company depreciates its capital
equipment at the rate of 20% per annum on a reducing balance basis.

96 | P a g e
The annual depreciation is calculated at the start of the financial year and one-twelfth of
this annual amount is included as monthly depreciation in the fixed overhead costs of
each process. Output transferred from one process to another is valued using transfer
prices based on the total budgeted costs of the process plus a mark-up of 15%.

Process B
This is the first process. Raw materials are blended to produce three different outputs,
two of which are transferred to Processes C and D respectively. The third output is
accounted for as a by-product and sold in the external market without further processing.
The equipment used to operate this process originally cost $800,000 on 1 January 2005.

The Process B account for April 2010 was as follows:

Litres $ Litres $
Opening WIP NIL NIL Normal Loss 3,000 3,000
Material W 10,000 25,000 By-product 5,000 5,000
Material X 5,000 10,000 Output to C 9,000 82,800
Material Y 12,000 24,000 Output to D 10,000 92,000
Direct labour 30,000 Closing WIP NIL NIL
Overhead 75,000
Profit & Loss 18,800
Totals 27,000 182,800 Totals 27,000 182,800

The material costs are variable per unit of input and direct labour costs are fixed in the
short term because employees contracts provide them with a six month notice period.
Overhead costs include a share of Head Office costs, and of the remaining overhead costs
some vary with the input volume of the process. The level of activity in April 2010 was
typical of the monthly volumes processed by the company.

Process C
This process receives input from Process B to which is added further materials to produce
a finished product that is sold in the external market at the budgeted selling price of $20
per litre. The equipment used to operate this process originally cost $500,000 on 1
January 2008.

The Process C account for April 2010 was as follows:

Litres $ Litres $
Opening WIP 1,000 11,200 Normal Loss 3,000 1,500
Input from B 9,000 82,800 Abnormal Loss 1,500 750
Material Z 3,000 15,000 Output 7,500 150,000
Direct labour 20,000 Closing WIP 1,000 11,200
Overhead 50,000
Profit & Loss 15,550
Totals 13,000 179,000 Totals 13,000 179,000

97 | P a g e
The material costs are variable per unit of input and direct labour costs are fixed in the
short term because employees contracts provide them with a six month notice period.
Overhead costs include a share of Head Office costs, and of the remaining overhead costs
some vary with the input volume of the process. The level of activity that occurred in
April 2010 was typical of the monthly volumes processed by the company, and the
opening and closing work in process are identical in every respect. The process is
regarded as an investment centre and completed output and losses are valued at their
selling prices. The manager of Process C is concerned at the level of output achieved
from the input volume and is considering investing in new equipment that should
eliminate the abnormal loss. This would involve investing $1,000,000 in new processing
equipment on 1 January 2011; the existing equipment would be sold on the same date at a
price equal to its net book value.

Process D
This process receives input from Process B which is further processed to produce a
finished product that is sold in the external market at the budgeted selling price of $16 per
litre. The equipment used to operate this process originally cost $300,000 on 1 January
2000.

The Process D account for April 2010 was as follows:

Litres $ Litres $
Opening WIP 1,000 5,500 Normal Loss 1,000 3,000
Input from B 10,000 92,000 Output 9,000 144,000
Direct labour 30,000 Closing WIP 1,000 5,500
Overhead 30,000 Profit & Loss 5,000
Totals 11,000 157,500 Totals 11,000 157,500

Direct labour costs are fixed in the short term because employees contracts provide them
with a six month notice period. Overhead costs include a share of Head Office costs, and
of the remaining overhead costs some vary with the input volume of the process. The
level of activity in April 2010 was typical of the monthly volumes processed by the
company, and the opening and closing work in process are identical in every respect. The
process is regarded as an investment centre and completed output and losses are valued at
their selling prices. The manager of Process D believes that the transfer price from
Process B is unfair because the equivalent material could be purchased in the open
market at a cost of $7.50 per litre.

Required:

(a)
(i) Calculate the annualised Return on Investment (ROI) achieved by each of the process
divisions during April 2010.
(4 marks)

98 | P a g e
(ii) Discuss the suitability of this performance measure in the context of the data provided
for each process division.
(4 marks)

(b)
(i) Calculate the effect on the annualised Return on Investment in 2011 of Process
Division C investing in new capital equipment.
(4 marks)

(ii) Discuss the conflict that may arise between the use of NPV and ROI in this
investment decision.
(4 marks)

(c) Discuss the transfer pricing policy being used by H from the viewpoints of the
managers of Process Division B and Process Division D.
(9 marks)

(Total = 25 marks)

D2 6 SWZ (CIMA P2 Nov 2010)

SWZ is a manufacturing company that has many trading divisions. Return on Investment
(ROI) is the main measure of each divisions performance. Each divisional managers
salary is linked only to their divisions ROI.

The following information summarises the financial performance of the S division of


SWZ over the last three years:

Year ending 31 October 2008 2009 2010


$000 $000 $000
Turnover 400 400 400
Cost of sales 240 240 240
Gross profit 160 160 160
Other operating costs 120 104 98
Pre-tax operating profit 40 56 62

Capital invested as at the end of the year 400 320 256

Other operating costs include asset depreciation calculated at the rate of 20% per annum
on a reducing balance basis.

The figures shown in the above table for the capital invested as at the end of the year is
the net book value of the divisions fixed assets.

99 | P a g e
All of the above values have been adjusted to remove the effects of inflation. There have
been no additions or disposals of fixed assets within the S division during this period.

Required:

(a) Discuss the performance of the S division over the three year period.
(9 marks)

The manager of the S division is now considering investing in a replacement machine.


The machine that would be replaced would be sold for its net book value which was
$40,000 at 31 October 2010 and the new machine would cost $100,000. The new
machine would have an expected life of five years and would be depreciated using the
same depreciation rates as the existing machinery. The new machine would reduce the
divisions cost of sales by 10%. At the end of five years it would be sold for its net book
value.

The divisional cost of capital is 8% per annum. The company has evaluated the
investment and correctly determined that it has a positive Net Present Value (NPV) of
$24,536.

Required:

(b) Prepare calculations to show why the manager of the S division is unlikely to go
ahead with the investment. Ignore taxation.
(11 marks)

(c) Prepare calculations to show how the use of Residual Income (RI) as the performance
measure would have led to a goal congruent decision by the manager of the S division in
relation to the purchase of the replacement machine. Ignore taxation.
(5 marks)

(Total = 25 marks)

D2 7 DE company (CIMA P2 May 2011)

The DE Company has two divisions. The following statement shows the performance of
each division for the year ended 30 April 2011:

D E
$000 $000
Sales 500,200 201,600
Variable cost 380,400 140,000
Contribution 119,800 61,600
Fixed costs 30,000 20,000
Operating profit 89,800 41,600

100 | P a g e
Division E manufactures just one type of component. It sells the components to external
customers and also to Division D. During the year to 30 April 2011, Division E operated
at its full capacity of 140,000 units. The transfer of 70,000 units to Division D satisfied
that divisions total demand for that type of component. However the external demand
was not satisfied. A further 42,000 components could have been sold to external
customers by Division E at the current price of $1,550.

The current policy of the DE Company is that internal sales should be transferred at their
opportunity cost. Consequently during the year, some components were transferred to
Division D at the market price and some were transferred at variable cost.

Required:

(a) Prepare an analysis of the sales made by Division E that shows clearly, in units and
in $, the internal and external sales made during the year.
(3 marks)

(b) Discuss the effect of possible changes in external demand on the profits of Division
E, assuming the current transfer pricing policy continues.
(6 marks)

Division E is considering investing in new equipment which would reduce its unit
variable costs by 20% and increase its capacity by 10% for each of the next five years.
The capital cost of the investment is $120m and the equipment would have no value after
five years. The DE company and its divisional managers evaluate investments using net
present value (NPV) with an 8% cost of capital.

External annual demand for the next five years will continue to be 112,000 components
at $1,550 each but the DE Company will insist that the internal annual demand for 70,000
components must be satisfied.

Required:

(c) Assuming that the current transfer pricing policy continues:


(i) Evaluate the investment from the perspective of the manager of Division E.
(6 marks)

(ii) Evaluate the investment from the perspective of the DE Company.


(4 marks)

Note: Ignore inflation and taxation.

101 | P a g e
(d) Explain TWO factors that should be considered when designing divisional
performance measures.
(6 marks)

(Total = 25 marks)

102 | P a g e
Solutions Section A

Part A Pricing and Product Decisions

A1 1 EXE (CIMA P2 Pilot Paper 2005)

Relevant costs

Workings $
Steel W1 55.00
Brass fittings W2 20.00
Skilled labour W3 300.00
Semi skilled labour W4 Nil
Overheads W5 7.50
Estimating time W6 Nil
382.50
Administration O/H W7 Nil
Profit W8 Nil
Selling price 382.50

Therefore the lowest quote is $382.50

Workings

W1 Steel
It is a direct material which is regularly used and therefore we would need to use the
replacement cost to value it for the quote. 10m x $5.50 = $55.00

W2 Brass fittings
We need the brass fittings specifically for the job and therefore should be included.

W3 Skilled labour
Skilled labour can either be obtained by asking staff to do overtime or stop production of
another product, because there is no skilled labour available.

Cost of overtime = 25hrs x $8p/h x 1.5 = $300

Stop existing production


= labour costs + lost contribution
= (25hrs x $8p/h) + (25hrs x $13p/h) = $525

Choose the cheapest option which would be paying staff overtime, therefore $300.

103 | P a g e
W4 Semi-skilled labour
The cost for semi-skilled labour should be zero as we currently have surplus staff that we
can use on this job.

W5 Overheads
The overheads are fixed and therefore ignored, but the machine power costs will be
incurred as a result of this job, and so therefore we should include these costs. $0.75 x
10hrs = $7.50.

W6 Estimating time
The cost of the estimating has already been spent and so should not be included. It is a
sunk cost.

W7 Administration overheads
These costs are ignored as they are not incurred as result of this job.

W8 Profit
The profit mark up should be ignored as we are working out the lowest possible quote for
this job.

A1 2 MNP (CIMA P2 May 2005)

Part (a)

Joint products are when two or more products are produced from the same process.
Shared costs at the split-off point are called common costs.

The basis used to apportion the common costs between M, N and P is litres produced.
This can be seen by the fact that we are left with the same value per litre of $5.68 when
we divide the respective values and litres of products M, N and P.

It is fair to use this method as outputs are in litres and the results will be truly reflective
of costs apportioned.

Costs of the complete process are apportioned between the joint products only (never by-
products) for stock valuation, pricing or profitability purposes.

Part (b)

Viability means whether or not it is profitable, if it is viable it means that it is profitable


and if it is not viable it means that it is not profitable.

Sales from M, N and P


= ($6.25 x 25,000) + ($5.20 x 15,000) + ($6.80 x 45,000) = $540,250

104 | P a g e
Common costs for M, N and P
= $141,875 + $85,125 + $255,375 = $482,375

Profit from M, N and P = 540,250 - $482,375 = $57,875

The common process is viable.

To work out the optimal processing plan we need to also consider the extra benefit
compared to the extra cost involved in processing M, N and P further.

Products Extra revenue Extra cost Net benefit


M $8.40 - $6.25 = $2.15 $1.75 $0.40
N $6.45 $5.20 = $1.25 $0.95 $0.30
P $7.45 - $6.80 = $$0.65 $0.85 ($0.20)

The optimal processing plan is to make products M and N further as they yield further
profits, but not product P as it yields a further loss. Product P should be sold just after the
common process.

A1 3 VBJ (CIMA P2 May 2005)

Relevant costs

Workings $
Coach W1 360
Fuel W2 1,500
Driver W3 4,000 or 800
Hotel W4 5,000
Cost of quote W5 Nil
General overheads W6 Nil
Total 10,860 or 7,660

Workings

W1 Coach
If we were to go ahead with this contract we would need to obtain a replacement coach to
cover existing obligations. If this is ignored then we lose contribution and incur
significant penalties on existing obligations.

Replacement coach costs = $180 x 2 days = $360 or if we dont honour our current
obligations then we would lose contribution of $250 x 2 days = $500.
It is cheaper to hire replacement coach for $360.

105 | P a g e
W2 Fuel
We need to spend $1,500 on fuel costs for this new contract.

W3 Driver
We need to hire a replacement driver for the 10 days of the new contract to cover current
obligations. Therefore $400 x 10 days = $4,000.

An alternativie way of looking at this is that the coach in any event will not be used for 8
days and so therefore ther is no need to hire a driver for 10 days as he will not be used.
Instead hire a driver for 2 days when he will be used for the coach. Therefore $400 x 2
days = $800.

W4 Hotel
This a specific cost for the new contract of $5,000.

W5 Cost of quote
The cost for preparing the quote of $250 has already been spent and so should not be
included.

W6 General overheads
These are to be ignored as they are not relevant to the quote.

A1 4 QXY plc (CIMA P2 Nov 2006)

Explanation of sales revenue line

The sales revenue line shows the amount of sales earned throughout the different level of
activities. It can be seen that between zero and somewhere between activity B and C there
is a constant rise or straight line increase in sales revenue. This means that the unit price
charged is the same and volume sold has been increasing at a constant rate.

Beyond the point between B and C sales revenue increases at much slower rate, this
indicates that selling price per unit is too high and in order to achieve previous rates of
growth there has to be a reduction in unit price.

Explanation of the fixed cost line

A fixed cost is a cost which cannot be easily identified or related to a cost per unit or
activity of any kind e.g. a cost which remains constant when the production of a good or
service within the organisation rises or falls.

Fixed cost over the long-term will normally display the characteristics of stepped cost
behaviour. That is the cost remains constant but only within a certain range of production.
Once this range of production is exceeded the fixed cost will rise.

106 | P a g e
In the diagram we can see that the fixed cost line is stepped. Between the zero activity
and up to activity level B fixed costs are constant. At zero activity fixed costs need to be
spent such as machinery and buildings in order to manufacture products.

If we were to increase our level of activity beyond level B there needs to be an increase in
fixed costs and then the costs are constant up to activity level D. There is another increase
in fixed costs at activity D when looking beyond this point and then the costs are constant
again.

These sudden stepped increases in fixed costs could be due to the factory reaching full
capacity and then extra leasehold expenses will need to be incurred in order to obtain
more buildings, if production is to increase or expand further.

Another example is supervisors salaries, they could be paid fixed salaries, but
supervision is limited to how many workers that can be supervised. Once the size of the
workforce exceeds a certain range another supervisor will need to be employed.

Explanation of total cost line

Total costs include both fixed costs and variable costs. Variable costs are costs that can
be easily identified or related to a cost per unit or activity level of some kind e.g. a
cost which rises or falls directly with the production/provision of a good or service within
an organisation. Examples could include labour piece work schemes e.g. a factory worker
that gets paid for each unit they make or the cost of material/components for the
production or assembly of a product.

All variable cost starts from the origin of the graph indicating the cost is nil if the activity
level is zero. Variable cost does not necessarily behave in a linear manner e.g. a constant
amount incurred for each unit of activity. It can behave in a curvilinear (non-linear)
manner as well, in which case the variable cost line would be curved not straight.

In the diagram it can be seen that at activity zero total cost is equal to fixed costs. At this
point there are no variable costs as there is no activity only fixed costs. Between activity
levels zero and B we can see that total cost line is increasing at constant level. The
constant increase is due to variable costs being incurred as a result of increasing activity.

Total costs increases in a stepped fashion at activity levels B and D because of the costs
behaviour of fixed costs as mentioned previously. It can be seen that variable costs are
increasing at the same constant rate within total costs up to activity level D, after this
point it can be seen that the total costs line increases more steeply. This is due to
increased variable costs per unit.

107 | P a g e
Issues to consider when making decisions

At activity level A it can be seen from the diagram the sales revenue line intersects the
total cost line indicating that this is the point when the company makes no loss or profit
i.e. breakeven. Any activity beyond this point sales revenue will exceed total costs
causing the company to make a profit, and anything below this activity, total costs will
exceed sales revenue causing the company to make losses.

After activity level B fixed costs will increases sharply because of perhaps new
investment required in the manufacturing process and profits will be reduced compared to
just before activity level B. The operational mangers needs to consider whether the sales
revenue forecast is likely to hold true, if not then profits can be reduced significantly as a
result of this investment.

Between activity levels B and C the sales revenue line has a much higher gradient line
than total costs and the company is earning greater profits as it increases its activity.
Profits are maximised just before point C when beyond this point the sales revenue line is
increasing at a slower rate when compared to total costs.

At activity level D there is another sharp increase in fixed costs and also variable costs
are rising at steeper gradient to sales revenue. The operational manger should recommend
to the company to continue to produce activity as long as the extra revenue is greater than
the extra cost or variable cost.

A1 5 RST (CIMA P2 May 2007)

Part (a)

We need to work out the extra benefit of making RZ, SZ and TZ and then compare these
to the extra cost involved. We should make those which give a net extra benefit.

Product Extra benefit Extra variable Extra fixed cost Net


cost
RZ $6.00 - $3.00 = $3.00 $1.40 Nil $1.60
SZ $5.75 - $5.00 = $0.75 $0.90 Nil ($0.15)
TZ $6.75 - $3.50 = $3.25 1.00 $600 / 1200 kg = $0.50 $1.75

Products R and T should be further processed to produce products RZ and TZ


respectively as they provide and extra net benefit of $1.60 and $1.75 per kg respectively.

We have assumed that future production levels of product TZ would be based on current
levels and so therefore the extra fixed costs per unit is based on 1,200 kg.

Product S should not be further processed to make product SZ as there is net cost of
$0.15 per kg every time an SZ is produced.

108 | P a g e
These recommendations are based purely on financial grounds and the company should
also look at qualitative factors as well before making their final decisions.

Part (b) (i)


Product Selling price Kg Total sales value
R $3 800 $2,400
S $5 2,000 $10,000
T $3.50 1,200 $4,200
$16,600

The common costs = $17,500. Therefore there would be a net loss of $900 if the
products R, S and T were sold to the external market. It is not financially viable.

Part (b) (ii)

If there is no external market for R, S and T then we must further process all these
products to produce RZ, SZ and TZ which can be sold.

Product Net benefit per kg Kg Total ($)


RZ $1.60 800 1,280
SZ ($0.15) 2,000 (300)
TZ $1.75 1,200 2,100
3,080
Loss from the common process (900)
Net benefit 2,180

After further processing the products it now is financially viable as there is a net
benefit of $2,180.

A1 6 HS (CIMA P2 Nov 2007)

Part (a)

1. Determine the price function or demand function

The price or demand function formula is:

P = a - bq

P = Price
a = Price at which demand would be zero (i.e. the p when q=0)
b = The gradient of the demand curve
q = Quantity sold at that price (P)

109 | P a g e
P = 1,350
q = 8,000
b = 50 /100 = 0.05
a=?

In order to determine the price function we need to first find the value of a. Substitute
all known values into the price function formula to determine a.

1,350 = a 0.05 (8,000)


1,350 = a 400
1,350 + 400 = a
1,750 = a

Now we can construct the price function:

P = 1,750 0.05q

2. Determine the marginal revenue function (MR)

The MR function is the price function itself, but it will have twice the value of whatever
the b value is, within the price function.

Therefore:

MR = 1,750 2(0.05)q
MR = 1,750 0.1q

3. Determine the marginal cost function (MC)

Marginal cost is $270 being the direct material cost; however this does not include any
variable costs from labour and conversion costs. We need to use the high low method to
work this out.

Units $000
High 9,400 7,000
Low 7,300 5,446
Difference 2,100 1,554

Variable cost = 1,554,000 / 2,100 = $740 per unit

Therefore total marginal cost = $270 + $740 = $1,010

110 | P a g e
4. Equate MR = MC to obtain the units sold to maximise profits

MR = MC

1,750 0.1q = 1,010


-740 = -0.1q
740 = 0.1q
7,400 = q

Therefore we sell 7,400 units when we maximise our profits.

5. Use the price function to determine the selling price that would maximise profits

We can substitute in 7,400 units into the price function we created before to determine
the selling price that will maximise profits.

P = 1,750 0.05q
P = 1,750 0.05 (7,400)
P = 1,750 -370
P = 1,380

The selling price that will maximise profits is $1,380.

Part (b)

Exam tip: Please note that the question only asks for two reasons however we have
provided a selection of possible reasons. Any two would suffice.

The pricing model assumes that there is a linear or straight line relationship between price
and demand in reality this may not be true.
It is difficult to determine the price at which demand would be zero in other the
maximum price that can be charged.

The theory assumes that price is the only factor which will influence or change demand
but in reality there are other factors such fashion, the availability of substitutes and the
cost of complimentary products.

It is difficult to ascertain the variable costs accurately without detailed knowledge of cost
curves this may not be available as HS is a new company.

111 | P a g e
A1 7 Bank charges (CIMA P2 May 2008)

Part (a) (i)

Existing bank account structure

The amount of business customers will fall by 20% if no action is taken.


Therefore the number of customers going forward = 1,000,000 x 80% = 800,000
Number of transactions per customer per year = 800,000 x 1,000 = 800,000,000
Transaction charges = 0.60 x 800,000,000 = 480,000,000

Number of customers with a positive balance = 45% of 800,000 = 360,000


Interest paid to positive balance customers
= 0.1% of 2,000 x 360,000 = 720,000

Number of customers with a negative balance = 55% of 800,000 = 440,000


Interest received from negative balance customers
= 20% of 4,000 x 440,000 = 352,000,000

Investment income is 3% of net position of customer balances


= 3% of ((440,000 x 4,000) (360,000 x 2,000)) = 31,200,000

Net income for the bank


= 480,000,000 - 720,000 + 352,000,000 + 31,200,000 = 862,480,000

Part (a) (ii)

Option 1

The amount of business customers will increase by 5%.


Therefore the number of customers going forward = 1,000,000 x 1.05 = 1,050,000
Monthly fixed charges = 10 x 12mths x 1,050,000 = 126,000,000

Number of customers with a positive balance = 45% of 1,050,000 = 472,500


Interest paid to positive balance customers
= 0.5% of 2,000 x 472,500 = 4,725,000

Number of customers with a negative balance = 55% of 1,050,000 = 577,500


Interest received from negative balance customers
= 20% of 4,000 x 577,500 = 462,000,000

Investment income is 3% of net position of customer balances


= 3% of ((577,500 x 4,000) (472,500 x 2,000)) = 40,950,000

Net income for the bank


= 126,000,000 - 4,725,000 + 462,000,000 + 40,950,000 = 624,225,000

112 | P a g e
Option 2

The amount of business customers will increase by 1%.


Therefore the number of customers going forward = 1,000,000 x 1.10 = 1,100,000

Number of customers with a positive balance = 45% of 1,100,000 = 495,000


Interest paid to positive balance customers is nil

Number of customers with a negative balance = 55% of 1,100,000 = 605,000


Interest received from negative balance customers
= 20% of 4,000 x 605,000 = 484,000,000

Investment income is 3% of net position of customer balances


= 3% of ((605,000 x 4,000) (495,000 x 2,000)) = 40,900,000

Net income for the bank


= 484,000,000 + 40,900,000 = 524,900,000

Conclusion

The recommended course of action should be to continue with the excising bank account
structure.

A1 8 WX (CIMA P2 May 2011)

Part (a) (i)

The direct material and labour costs are completely variable. This can be determined by
dividing the combined costs of labour and material costs by the respective activity level
in the forecast, which will result in the same cost per unit at all activity levels.

For example at activity level 100,000 units material and labour costs added together are
$800,000, and therefore cost per unit = $8 per unit. This is the same rate at activity levels
160,000 and 200,000 units if you compare them with their respective combined material
and labour costs.

Overhead costs however are not completely variable and must be analysed between fixed
overheads and variable overheads.

We need to use the high-low method to find the variable overheads.

Units Overhead cost ($)


200,000 1,460,000
100,000 800,000
100,000 580,000

113 | P a g e
$580,000 / 100,000 = $5.80.
Total variable cost per unit = $8 + $5.80 = $13.80.

Part (a) (ii)

1. Determine the price function or demand function

The price or demand function formula is:

P = a - bx

P = Price
a = Price at which demand would be zero (i.e. the p when x=0)
b = The gradient of the demand curve
x = Quantity sold at that price (P)

P = 25
b = 1 /25,000 = 0.00004
x = 150,000
a=?

In order to determine the price function we need to first find the value of a. Substitute
all known values into the price function formula to determine a.

25 = a 0.00004 (150,000)
25 = a 6
25 + 6 = a
31 = a

Now we can construct the price function:


P = 31 0.00004x
2. Determine the marginal revenue function (MR)

The MR function is the price function itself, but it will have twice the value of whatever
the b value is, within the price function.

Therefore:
MR = 31 2(0.00004)x
MR = 31 0.00008x

3. Determine the marginal cost function (MC)

Marginal cost is given in the question as $13.80.

114 | P a g e
4. Equate MR = MC to obtain the units sold to maximise profits

MR = MC

31 0.00008x =13.80
-0.00008x = 13.80 - 31
-0.00008x = -17.20
x = -17.20 / -0.00008
x = 215,000

Therefore we sell 215,000 units when we maximise our profits.

5. Use the price function to determine the selling price that would maximise profits

We can substitute in 215,000 units into the price function we created before to determine
the selling price that will maximise profits.

P = 31 0.00004x
P = 31 0.00004 (215,000)
P = 31 8.6
P = 22.4

The selling price that will maximise profits is $22.40.

Part (b)

Exam tip: Please note that the question only asks for two reasons however we have
provided a selection of possible reasons. Any two would suffice.

The optimum selling process is derived from the price function or demand function.

The main problem with using the price function is that the quality of the market research
to determine the demand function has to be very good, for it to have any real value;
otherwise it may give unrealistic predictions.

It also assumes that price and quantity are the only factors in determining demand, but we
must be mindful of other factors which are just as important such as quality, advertising,
the availability of substitutes, brand loyalty, fashion, and the cost of complimentary
products. It is difficult to estimate the demand curve.

The price function assumes that there is a linear or straight line relationship between price
and demand in reality this may not be true.

It is difficult to determine the price at which demand would be zero in other words the
maximum price that can be charged.

115 | P a g e
It is difficult to ascertain the variable costs accurately without detailed knowledge of cost
curves this may not be available.

116 | P a g e
Solutions Section B

Part A Pricing and Product Decisions

A2 -1 TQ (CIMA P2 Pilot paper 2005)

Part (a) (i)

In order to work out the price when profits are maximised we need to construct the
demand function, from this we can derive the marginal revenue (MR) curve and equate
this to the marginal cost curve (MC) to obtain the profit max position. MR=MC.

We are given in the question that when x = 0 then P = $100 which is the maximum price.
If this is the case then we can work out a by using the demand function formula.

Therefore:

P = a bx
100 = a b(0)
100 = a 0
100 = a

We need to obtain another set of price and quantity to obtain the gradient or b value of
P = a bx, as the above set does not enable us to find it.

We are told that the selling price is $60 but not the quantity sold, but we are given fixed
overhead volume variances at this price. We can use these to obtain a quantity for this
price.

Fixed overhead volume variance


= (Actual production Budgeted production) x F/OH per unit

For each period we shall work out the actual units produced using the above formula.
This is to determine whether we have different actual units for each period and therefore
we would have a different demand function for each period as well.

Period 1

-1200 = (Actual production 520) x 10


-1,200 / 10 = Actual production 520
-120 + 520 = Actual production
400 = Actual production

117 | P a g e
Period 2

-1900 = (Actual production 590) x 10


-1900 / 10 = Actual production 590
-190 + 590 = Actual production
400 = Actual production

Period 3

-2600 = (Actual production 660) x 10


-2600 / 10 = Actual production 660
-260 + 660 = Actual production
400 = Actual production

We have the same actual units in each period of 400 and at price $60, this is quantity we
will sell.

We can now work out b.

P = a bx
60 = 100 b (400)
60 - 100 = -b (400)
-40 / 400 = -b
-0.1 = -b
0.1 = b

Therefore the demand function is P = 100 0.1x

MR = 100 2(0.1)x
MR = 100 0.2x

MC = budgeted variable costs = $25

Therefore profit max is when MR = MC


100 0.2x = 25
-0.2x = 25 100
-0.2x = -75
x = -75 / -0.2
x = 375
The quantity we would sell to maximize profits is 375 units.

The price for this quantity is P = 100 0.1 (375) = $62.50

118 | P a g e
Part (a) (ii)

Actual contribution in period 3


400 units x ($60 per unit - $25 per unit) = $14,000

We would have earned if the optimal price was used in period 3


375 units x ($62.50 per unit - $25 per unit) = $14,062.50

Difference = $62.50

Part (b)

This question had at least one mark for writing your answer in a report format!

REPORT
To: Board of Directors of TQ
From: Management Accountant
Subject: Alternative pricing strategies
Date: 19th September 2003

1. Introduction

This report is designed to discuss the alternative pricing strategies available to TQ.

2. Market skimming

TQ can initially charge a high price, achieve low volume but earn a larger profit per unit
sold. This is known as market skimming. It is a strategy which exploits a price insensitive
market or an inelastic demand for a product.

TQ is selling a third generation mobile phone, it is a high technology product and as a


result there maybe an expectation by customers to pay a premium for these products.
TQs third generation phone is the first of its kind and customers will want to be the first
to enjoy it.

TQ could also use this approach as it is good for innovative high quality products where
little competition exists initially. Therefore a premium can be charged without reducing
demand for it too much.

There is always a choice of lowering the price later on when competition comes it to the
market with cheaper substitutes.

The market skimming approach also allows TQ to recover the significant levels of
development costs it had to spend on developing this phone and the fact that they have a
very short life cycle.

119 | P a g e
3. Market penetration

TQ can charge a very low price in order to capture a larger market share quicker, this is
known as market penetration.

This approach will aim to achieve a wide customer base as quickly as possible by setting
the price very low. It means that because if its affordability a lot of people will be able to
purchase it, as a result the first 2 stages are quickly achieved in the product life cycle and
also achieve economies of scale far quicker.

It will also be more difficult for other companies to enter the market as they would have
to compete on this low price and may struggle to make a profit.

4. Demand based approach

TQ could create a demand function much like that created from the second generation
phones in part (a). This would represent a straight line relationship between price charged
and quantity sold (P = a - bx) of the third generation phones, and allow us to find that
price that we can charge that would maximise profits (MR = MC).

The main problem with this is that the quality of the market research to determine the
demand function has to be very good, for it to have any real value.

It also assumes that price and quantity are the only factors in determining demand, but
TQ must be mindful of other factors which are just as important such as quality,
advertising, substitutes and brand loyalty. It is difficult to estimate the demand curve.

The demand function approach does give us a useful insight into the relationship between
price and quantity and a basis to predict what strategy to take. It makes an attempt to
incorporate demand into its calculations and is concerned with the marginal costs of
making decision ignoring irrelevant costs.

The demand function also allows us to see at what price and quantity we would maximise
profits and we can predict quantity sold given any selling price and vice versa.

I hope you have found this report useful but should you require any further assistance or
have any questions please do not hesitate to contact me.

Signed : Management Accountant.

120 | P a g e
A2 2 QP plc (CIMA P2 Nov 2005)

Part (a)

The theory of constraints is where a bottleneck exists (or limiting factor in production);
this therefore is a constraint on throughput. Throughput is the rate of production.

A managers aim in this situation would be to operate the bottleneck resource at 100%
capacity, whilst running non-bottleneck resources at a speed that matches this (which
may not be at 100%). This would be efficient use of resources, but also would avoid the
pile up of unnecessary work-in-progress in a JIT environment.

Throughput accounting aims to maximise contribution whilst minimising conversion


(labour and overhead) cost.

Throughput contribution = Sales - material cost

It is assumed only materials and components are the variable cost.

Conversion cost = all other fixed cost

Return per factory hour is similar to the concept of contribution maximisation; you
should notice the following calculation is similar to the contribution per unit of a limiting
factor used in short-term decision-making.

Return per factory hour = Sales less material cost


Usage (in hours) of the bottleneck resource

TA ratio = Contribution (sales less material) per hour


Conversion cost per hour (or cost per factory hour)

The above TA ratio can be used to assess the manager in terms of how well they have
done maximising contribution, whilst eliminating the build up of stock.

Such a technique is similar to limiting factor analysis, as managers try to improve their
TA Ratio they must concentrate on producing those products that generate the highest
contribution per bottleneck resource (or limiting factor) in order to optimise contribution
and therefore profit.

Traditionally when absorption costing was used, managers could improve results by
setting production levels higher than sales (therefore the carry forward of fixed overhead
this period to the next in the valuation of closing stock), however stock piling is not a
good thing and such stock piling will not improve performance if you use the TA Ratio.

121 | P a g e
Part (b)

We need to first identify the limiting factor and from the question it would seem it is only
ingredient L. Ingredient M may be limited but it has a substitute being V which has
unlimited supply. Having identified the limiting factor we must now work out the
contribution we earn from each of the meals, then the contribution per kg of L, and then
rank in order of production.

TR PN BE

Selling price 340 450 270
Ingredient K (150) (120) (90)
Ingredient L (70) (90) (40)
Ingredient M (30) (75) (45)
Throughput contribution 90 165 95

Kg of L used 7kg 9kg 4kg

Contribution per kg of L 90/7kg = 12.86 165/9kg = 18.33 95/4kg = 23.75

Rank in order of production 3 2 1

Total ingredient L available is 7,000kg, however we must make a minimum order first of
50 batches for each type of meal. The ingredients used for this must be deducted before
working out the optimum production plan.

Minimum contract Kg of L per batch Ingredient L used


TR 50 7 350kg
PN 50 9 450kg
BE 50 4 200kg
Total 1,000kg

Amount of ingredient L available after the minimum contract is 6,000kg.

L used
BE 300 batches x 4kg 1,200
PN 350 batches x 9kg 3,150
TR 6,000kg 1,200kg 3,150kg = 1,650kg 1,650kg / 7kg = 236 batches 1,650
6,000

Optimum production plan

Minimum contract Production Total


PN 50 350 400
BE 50 300 350
TR 50 236 286

122 | P a g e
Part (c)

We need to interpret the linear programming solution that the computer has produced.

Objective function

This is 110,714 and is the maximum contribution that can be earned given the
constraints.

TR, PN and BE values

These are the amounts we should produce in order to maximise contribution. Therefore
we should make 500 batches of TR, 357 batches of PN and 71 batches of BE.

TR slack value

This is zero and represents the fact that we have produced up to the maximum demand of
TR, this being 500 batches. There is no shortfall in demand.

PN and BE slack values

These values represent the fact that we have not met our maximum demands for PN and
BE by 43 batches and 279 batches respectively. We are only producing 357 PNs and 71
BEs where our maximum demand is 400 and 350 batches respectively.

L and M values

These values represent shadow prices for ingredients L and M as they are both scarce
resources. Ingredient M is now also scarce because it says in the question that substitute
ingredient V is poisonous and therefore cannot be used.

The values show that for each kg of scarce resource we obtain at normal cost we can
expect to earn 3 of contribution for each kg of L and 28 contribution for each kg of M.

Ingredient K does not have a shadow price because it is not a scarce resource; we have
enough of K to produce our batches.

123 | P a g e
A2 3 ZP plc (CIMA P2 Nov 2005)

Part (a)

Workings D E F Total
Consultants salary W1 40,000 140,000 60,000 240,000
Travel W2 4,000 7,000 4,000 15,000
Accommodation W3 nil 8,000 3,000 11,000
44,000 155,000 67,000 266,000

Workings

W1 - Consultants salary are direct costs for each client group, driven by chargeable
hours.

Total cost = 4 x 60,000 = 240,000

Chargeable hours = (100 x 10) + (700 x 5) + (300 x 5) = 6,000 hours

Rate per hour = 240,000 / 6,000 hours = 40 per hour

Cost for D = 40 x 100 x 10 = 40,000


Cost for E = 40 x 700 x 5 = 140,000
Cost for F = 40 x 300 x 5 = 60,000

W2 Travel cost driven by number of miles.

Number of total miles = (50 x 3 x 10) + (70 x 8 x 5) + (100 x 3 x 5) = 5,800 miles

Cost per mile = 15,000 / 5,800 miles = 2.59 per mile

Cost for D = 2.59 x 50 x 3 x 10 = 3,885


Cost for E = 2.59 x 70 x 8 x 5 = 7,252
Cost for F = 2.59 x 100 x 3 x 5 = 3,885

W3 Accommodation costs are driven by visits to clients in excess of 50 miles.


Therefore not applicable to client D as distance to their clients is less than 50 miles.

Number of qualifying visits = (8 x 5) + (3 x 5) = 55

Cost per visit = 11,000 / 55 = 200

Cost for E = 200 x 8 x 5 = 8,000


Cost for F = 200 x 3 x 5 = 3,000

124 | P a g e
Other costs

Office premises 50,000
Advertising 5,000
Telephone 10,000
Support staff 120,000
185,000

There is no obvious activity which directly drives these costs and therefore we must leave
them out of our calculations as they would not add any further benefit if they were
included.

Part (b)

Comparing the current system with the proposed ABC system

D E F Total
Chargeable hours 1,000 3,500 1,500 6,000

Current system (75p/hr) 75,000 263,000 113,000 451,000
ABC 44,000 155,000 67,000 266,000
Difference 31,000 108,000 46,000 185,000

The difference is due to the unattributable costs of 185,000 as shown in part (a). We
need to obtain a cost driver for these other costs to make the above comparison more
meaningful.

We need to look at whether the same or similar figures can be arrived at by doing a rate
per chargeable hour on the total costs for ABC.

Blanket rate on chargeable hours = 266,000 / 6,000 hrs = 44.33p/hr

D E F Total
Chargeable hours 1,000 3,500 1,500 6,000

Blanket (44.33p/hr) 44,333 155,155 66,500 265,988
ABC 44,000 155,000 67,000 266,000
Difference 333 155 (500) (12)

It seems that ABC does not add much more value than the current system, as the current
system gives similar figures.

In conclusion ABC is theoretically superior, but in this case perhaps not appropriate
unless further costs drivers reveal different figures.

125 | P a g e
Part (c)

Unit level costs

These are driven by the quantity of items produced. The more produced the lower the unit
cost.

Example:
Consultants produce chargeable hours in exchange for their salary at work. We can work
out a unit cost per chargeable hour. Cost per hour = 240,000 / 6,000 hours = 40 per
hour

Batch level costs

These are costs incurred for every time a batch is processed for example administration
costs maybe incurred when a batch of mugs are made in a factory.

Examples:
The cost incurred in visiting every client once.
The cost incurred in billing every client.
The cost incurred in chasing clients for outstanding monies.
The cost incurred in negotiating the fees on a contract with a client.

Product sustaining

These are costs needed to continue the product in the future, for example client care
costs.

Examples:
The cost to employ administrative staff to organise, plan for clients service requirements.

The cost to entertain clients such as buying them lunch, evening meals and drinks or even
tickets to see sports events.

Facility sustaining

These are fixed costs spent for the use of the company as a whole, for example
warehouse or machinery costs.

Examples:
Telephone systems, fax machines, photocopiers and the canteen.

126 | P a g e
A2 4 AVX plc (CIMA P2 May 2006)

Part (a) (i) (ii)

November

1 batch made which took 50 hours which took as long as the standard and therefore no
learning effect as the labour efficiency variance was nil.

December

1 batch was made which cost 500 170 = 330


Actual spend of standard cost is 330 / 500 x 100% = 66%

Actual hours taken = 66% x 50hrs = 33hrs

Average hours per batch = (50hrs +33hrs) / 2 = 41.5hrs

Learning curve effect = 41.5 hrs / 50hrs = 83%

January

2 batches were made which cost 1,000 452.20 = 547.80


Actual spend of standard cost is 547.80 / 1,000 x 100% = 54.78%

Actual hours taken = 54.78% x 100hrs = 54.78hrs

Average hours per batch taken to date = (50hrs + 33hrs + 54.78hrs) / 4 = 34.45hrs

Learning curve effect = 34.45hrs / 41.5hrs = 83%

February

4 batches were made which cost 2,000 1,089.30 = 910.70


Actual spend of standard cost is 910.70 / 2,000 x 100% = 45.53%

Actual hours taken = 45.53% x 200hrs = 91.06hrs

Average hours per batch taken to date = (50hrs + 33hrs + 54.78hrs + 91.06hrs) / 8 =
28.61hrs

Learning curve effect = 28.61hrs / 34.45hrs = 83%

127 | P a g e
March

8 batches were made which cost 4,000 1,711.50 = 2,288.50


Actual spend of standard cost is 2,288.50 / 4,000 x 100% = 57.21%

Actual hours taken = 57.21% x 400hrs = 228.84hrs

Average hours per batch taken to date = (50hrs + 33hrs + 54.78hrs + 91.06hrs +
228.84hrs) / 16 = 28.61hrs

Learning curve effect = 28.61hrs / 28.61hrs = 100%

April

16 batches were made which cost 8,000 3,423 = 4,577


Actual spend of standard cost is (4,577 / 8,000) x 100% = 57.21%

Actual hours taken = 57.21% x 800hrs = 457.68hrs

Average hours per batch taken to date = (50hrs + 33hrs + 54.78hrs + 91.06hrs +
228.84hrs + 457.68hrs) / 32 = 28.61hrs

Learning curve effect = 28.61hrs / 28.61hrs = 100%

The learning curve is 83% and ceases in March, when batches take 28.61 hours to make
on average. The implications are that there are no further labour efficiencies to be had
after the curve ceases and selling price and costs should be based on a batch taking 28.61
hrs.

Part (b)

Note: If Price (P) = a bx then Marginal Revenue (MR) = a 2bx

1. Determine the price function or demand function

The price or demand function formula is:

P = a - bQ

P = Price
a = Price at which demand would be zero (i.e. the p when Q=0)
b = The gradient of the demand curve
Q = Quantity sold at that price (P)

128 | P a g e
P = 1,200
b = 20
Q = 16
a=?

In order to determine the price function we need to first find the value of a. Substitute
all known values into the price function formula to determine a.

1,200 = a 20 (16)
1,200 = a 320
1,200 + 320 = a
1,520 = a

Now we can construct the price function:

P = 1,520 20Q

2. Determine the marginal revenue function (MR)

The MR function is the price function itself, but it will have twice the value of whatever
the b value itself is, within the price function.

Therefore:

MR = 1,520 2 (20) Q
MR = 1,520 - 40Q

3. Determine the marginal cost function (MC)

Marginal cost is given in the question as 672.72.

4. Equate MR = MC to obtain the units sold to maximise profits

MR = MC
1,520 40Q = 672.72
-40Q = 672.72 1,520
-40Q = -847.28
Q = 21.182

Therefore we sell 21.182 units when we maximise our profits.

5. Use the price function to determine the selling price that would maximise profits

We can substitute in 21.182 units into the price function we created before to determine
the selling price that will maximise profits.

129 | P a g e
P = 1,520 20Q
P = 1,520 20 (21.182)
P = 1,520 423.64
P = 1,096.36

The selling price that will maximise profits is 1,096.36.

Part (c) (i)

Standard costing

A standard cost is a planned or forecast unit cost for a product or service, which is
assumed to hold good given expected efficiency and cost levels within an organisation. It
represents a target cost and is useful for planning, controlling and motivating within an
organisation.

Under a standard costing system an organisation can value stock at standard cost,
incorporating this within the ledger or cost accounts of the organisation, the budget or
forecasts being a memorandum kept outside the ledger accounts.

Target costing

Market price to achieve desired market share XX

TARGET COST (Balance) (XX)

Desired profit XX

Used by Nissan, Sony and Toyota and many other Japanese companies, who sought not
what a product does cost (which is what most UK companies used as the method of
pricing) but rather what it should cost.

The idea is that the product price is determined by the market place, a desired level profit
is decided upon and the balance is the costs to manufacture the products through
improved processes to be able to reduce costs to sell at the market price. Target costing
seeks to continually improve the manufacturing process by the use of JIT, TQM, cost
reduction, value analysis and benchmarking.

Standard costing develops a product, determines its cost and adds a mark up to determine
a price. This was very much an internal process ignoring competition or demand. Target
costing starts with the external market price and works backwards to deicide on levels of
cost and profit.

130 | P a g e
Part (c) (ii)

AVX Plc sells the CB45 circuit board which is a high technology product and has been
able to charge a premium price for these products.

AVX Plc has used this approach and has enjoyed good product volume as little
competition exists initially, however it seems that competitors have now managed to
create a close substitute to this circuit board and so customers are choosing the cheaper
alternative.

This indicates that AVX Plc must look to reducing the price of the circuit board in order
to sustain the volume of sales they expect, and use economies of scale to reduce costs. If
a price reduction is not made then we could find continuation of falling volumes sold and
the CB45 falling out of favour with customers who have bought the cheaper alternative.

It also shows that these circuit boards have a short lifecycle as in six months it seems that
the CB45 is already into its maturity stage and perhaps heading towards decline. AVX
Plc can no longer set its own price and must accept the market price.

If AVX Plc wanted to sustain the same price for these circuit boards then they to be
modified to provide additional functions that customers value which the competitors
circuit boards do not provide.

A2 5 GHK (CIMA P2 May 2006)

Part (a)

Workings

W1 Direct Material A

We know that they used $5 per kg in the budgets. This is in correct as they should have
used $7 per kg as the material is in regular use and is not scarce.

Material A cost percentage change = ($7 per kg / $5 per kg) x 100% = 140%

Therefore increase the budgets by 140% to obtain a basis on $7 per kg, and then divide
by the number of units budgeted for to obtain a cost per unit.

Product Relevant cost Budgeted units Cost per unit


G $9,000 x 140% = $12,600 3,000 $12,600 / 3,000 = $4.20
H $12,000 x 140% = $16,800 3,000 $16,800 / 3,000 = $5.60
J $4,500 x 140% = $6,300 3,000 $6,300 / 3,000 = $2.10
K $18,000 x 140% = $25,200 3,000 $25,200 / 3,000 = $8.40

131 | P a g e
W2 Direct Material B

This is used when required and the cost of $10 per kg is used in the budgets.

Therefore on a per unit basis:

G: $6,000 / 3,000 units = $2


H: $6,000 / 3,000 units = $2
J: $13,500 / 3,000 units = $4.50
K: $36,000 / 3,000 units = $12

W3 Direct Labour

This is used when required and the cost of $10 per hour is used in the budgets.

Therefore on a per unit basis:

G: $6,000 / 3,000 units = $2


H: $24,000 / 3,000 units = $8
J: $22,500 / 3,000 units = $7.50
K: $9,000 / 3,000 units = $3

W4 Overhead

We need to look at the change in overhead costs between different levels of units. This
will be our relevant cost for overheads.

5,000 Variable Variable overheads


3,000 units
units overheads per unit
Product G $6,000 $8,000 $2,000 $2,000 / 2,000 units = $1
Product H $13,000 $19,000 $6,000 $6,000 / 2,000 units = $3
Product J $11,000 $17,000 $6,000 $6,000 / 2,000 units = $3
Product K $11,000 $17,000 $6,000 $6,000 / 2,000 units = $3

Product G H J K
$ $ $ $
Selling price 10 20 15 30
Direct Material A (W1) (4.20) (5.60) (2.10) (8.40)
Direct Material B (W2) (2) (2) (4.50) (12)
Direct Labour (W3) (2) (8) (7.50) (3)
Overhead (W4) (1) (3) (3) (3)
Contribution 0.80 1.40 (2.10) 3.60
0.80 / 2 1.40 / 2 (2.10) / 4.50 3.60 / 12
Contribution per $ of Material B
= 0.40 = 0.70 = 0.47 = 0.30

132 | P a g e
Part (b)

We need to work out how much extra contribution we can make if we did not supply the
major customer.

All resources are available except material B. Lets find out how much of the scarce
resource material B will be released if the major customer was not supplied.

Material B Material B Material B


Product Units in contract Total kg
cost per unit cost per kg per unit in kg
G 500 $2 $10 $2 / $10 = 0.2 500 x 0.2 = 100
H 1,600 $2 $10 $2 / $10 = 0.2 1,600 x 0.2 = 320
J 800 $4.50 $10 $4.50 / $10 = 0.45 800 x 0.45 = 360
K 400 $12 $10 $12 / $10 = 1.2 400 x 1.2 = 480

Total material B released = 100 + 320 + 360 + 480 = 1,260 kg

Our optimum production plan as described in the requirement is greater than normal
customer demand for products G and H, but for product J and K they are lower. We
should not use the released material B to produce any more of G or H as there is no
further demand for them nor should we look to produce J as it has a negative
contribution. Therefore we should make product K with the released material B.

We should only produce another 4,000 2,017 = 1,983 units of K as demand would be
satisfied then. With 1,260 kg of material B we can make another 1,260 kg / 1.20 kg per
unit = 1,050 units of K.

The extra contribution from K = 1,050 units x $3.60 per unit = $3,780. As a result of
stopping the contract to the major customer it means that we will be stopping the
production of J which is loss making, and therefore we will be saving on its the specific
fixed costs of $1,000. Therefore the extra contribution will be from K = 4,780.

Product Units Contribution per unit Total contribution


G 500 $0.80 $400
H 1,600 $1.40 $2,240
J 800 ($2.10) ($1,680)
K 400 $3.60 $1,440
Total $2,400

More contribution is earned if the contract is ignored. The extra amount of contribution
that is earned is $4,780 - $2,400 = $2,380. GHK would be indifferent meeting the
contract or paying the penalty if the penalty was $2,380.

133 | P a g e
Part (c)

Product Selling Price per unit Contribution per unit C/S ratio
G $10 $0.80 0.80/10 = 0.08
H $20 $1.40 1.40/20 = 0.07
J $15 ($2.10) (2.10)/15 = 0.14
K $30 $3.60 3.60/30 = 0.21

Part (d)

We need to see how fixed costs are covered by the sales of each of these products first, if
we are to sketch a profit volume chart. Specific fixed costs can be avoided by not
undertaking a product but non-specific or general fixed costs are unavoidable. We need to
work out how much non-specific fixed costs we have.

Workings

Product G for 5,000 units

Overhead costs = $8,000


We know that the variable element of this is $1 per unit x 5,000 units = $5,000
We also know that product specific fixed cost is $1,000
Therefore the non-specific fixed cost = $8,000 - $5,000 - $1,000 = $2,000

Product H for 5,000 units

Overhead costs = $19,000


We know that the variable element of this is $3 per unit x 5,000 units = $15,000
We also know that product specific fixed cost is $1,000
Therefore the non-specific fixed cost = $19,000 - $15,000 - $1,000 = $3,000

Product J for 5,000 units

Overhead costs = $17,000


We know that the variable element of this is $3 per unit x 5,000 units = $15,000
We also know that product specific fixed cost is $1,000
Therefore the non-specific fixed cost = $17,000 - $15,000 - $1,000 = $1,000

Product K for 5,000 units

Overhead costs = $17,000


We know that the variable element of this is $3 per unit x 5,000 units = $15,000
We also know that product specific fixed cost is $1,000
Therefore the non-specific fixed cost = $17,000 - $15,000 - $1,000 = $1,000

134 | P a g e
Product $
G 2,000
H 3,000
J 1,000
K 1,000
7,000
Specific fixed costs for all four products 4,000
Total 11,000

We need to now introduce each of the products and noting the cumulative profit or loss
figure. We would obviously make the product which gives the most contribution first.
Remember the requirement says that we have no longer any restrictions on resources and
make as many as we desire, and we should make up to the demand we have which would
of course include the one off contact to a major customer.

Cumulative Profit Cumulative Sales


No products sold
-$7,000 Nil
(only non-specific costs)
-$7,000 - $1,000
Specific fixed costs of K
= -$8,000
-$8,000 + (4,400 x $30 x 0.12) 4,400 x $30
Make K
= $7,840 = $132,000
-$7,840 - $1,000
Specific fixed costs of G
= $6,840
$132,000
$6,840 + (4,100 x $10 x 0.08)
Make G + (4,100 x $10)
= $10,120
= $173,000
$10,120 - $1,000
Specific fixed costs of H
= $9,120
$173,000
$9,120 + (4,600 x $20 x 0.07)
Make H + (4,600 x $20)
= $15,560
=$265,000
$15,560 - $1,000
Specific fixed costs of J
= $14,560
$265,000
$14,560 + (3,800 x $15 x -0.14)
Make J + (3,800 x $15)
= $6,580
=$322,000

135 | P a g e
Profit volume chart for products G, H, J, and K
20,000

15,000
H
J
Profit/Loss ($000)

10,000 G

5,000
K

0
50 100 150 200 250 300 350

5,000
Sales ($000)

10,000

Part (e)

The chart illustrates the profit that could be earned by the 4 products. It assumes that we
will earn profits in line with contribution and assumes that the products are manufactured
in the order of highest contribution earners first.

It also shows which products are earning more contribution as the more steep the line is
the more contribution is earned, clearly here we can see that K is the most steep and earns
the most contribution then G and then H. J slopes downwards and shows that it is making
a negative contribution per unit. J should not be manufactured on these grounds.

We can also see the specific fixed costs for each product on the chart by the initial
reduction of profit by 1,000 at the start of each separate product line. The specific fixed
would not be spent if the product was not made.

136 | P a g e
A2 6 H (CIMA P2 May 2007)

Part (a)

Relevant costs

Workings $
Technical report W1 0
Material A W2 15,000
Material B W3 2,000
Direct labour W4 500
Supervision W5 0
Machine A W6 240
Machine B W7 100
Despatch W8 400
Fixed overhead costs W9 0
Profit mark-up W10 0
Total costs 18,240

Workings

W1 Technical report
The cost of the technical report has already been spent and so should not be included. It is
a sunk cost.

W2 Material A
It is a direct material which is regularly used and therefore we would need to use the
replacement cost to value it. 10,000 sheets x $1.50 = $15,000.

W3 Material B
We need 200 litres of ink specifically for the job but we can only buy in order sizes of
250 litres. There is also no certainty of any value for the remaining ink therefore the full
amount should be included. $8 x 250 litres = $2,000.

W4 Direct labour
The cost for direct labour is the overtime = 50hrs x $10p/h = $500.

W5 Supervision
The cost for the supervisor should be zero as currently she is able to include the
additional duties within her current hours.

W6 Machine A
Lost contribution of machine A hours = 20 hrs x $12per hr = $240.

137 | P a g e
W7 Machine B
These are the extra running costs = 25 machine hrs x $4 per hr = $100.

W8 Despatch
Delivery cost necessary for the catalogues of $400.

W9 Fixed overhead costs


These costs are ignored as they are not incurred as result of this job.

W10 Profit
The profit mark up should be ignored as we are working out the lowest possible quote for
this job.

Part (b)

In the case of short-term pricing it is appropriate to look at relevant costing. This is


because relevant costing takes into account the true costs and benefits that would occur as
a result of selling at a price today. It ignores those costs that have already been spent such
as fixed overheads as these would not be incurred now if the product was sold.

In the case of long-term pricing it is appropriate to include these fixed overheads as they
have not being occurred yet and would be as result of the manufacture of the product in
question. As a result of this the long-term price maybe higher than the short-term price,
and may mean that the price is no longer competitive.

Traditional absorption costing systems tries to include an element of fixed overhead costs
in the unit cost of products; this would be based on an activity basis that best represents
how the fixed overheads will be used. However fixed overheads do not increase or
decrease on a per unit basis they change when capacity is reached for a facility in the
organisation. This makes the inclusion of fixed overheads arbitrary and the true
profitability of the different products distorted.

Relevant costing would ignore the fixed overheads when trying to understand the
profitability of the different products because it would show the true contribution to fixed
overheads without any arbitrary distortions.

138 | P a g e
A2 7 DFG (CIMA P2 Nov 2007)

Part (a)

Define the key variables - this is the assigning of letters to the products and services that
are needed to be made and then using these letters to represent the amount that should be
made at the optimum point.

Let: D = number of D units produced


G = number of G units produced

Construct the objective function this is looking at identifying the main objective that
is trying to be achieved. Here we are trying to maximise the contribution for D and G, so
we need to work this out first and then construct the objective function.

D per unit ($) G per unit ($)


Selling price 115 120
Direct material A 20 10
Direct material B 12 24
Skilled labour 28 21
Variable overhead 14 18
Contribution 41 47

Objective function:

C = 41D + 47G

Set up the constraints these show the limits of resources available to us to try and
meet the conditions of the objective function and they are usually described as linear
equations.
D per unit G per unit
Usage of material A 20 / 5 = 4kg 10 / 5 = 2kg
Usage of material B 12 / 3 = 4kg 24 / 3 = 8kg
Usage of skilled labour 28 / 7 = 4hrs 21 / 7 = 3hrs
Usage of machine 14 / 2 = 7hrs 18 / 2 = 9hrs

4D + 2G 1,800 (Material A constraint)


4D + 8G 3,500 (Material B constraint)
4D + 3G 2,500 (Skilled labour hours constraint)
7D + 9G 6,500 (Machine hours constraint)
D 400, G 400 (Maximum demand constraints)

139 | P a g e
Logic or non-negativity constraints these are constraints which will ensure that the
answer obtained in the solution is sensible in that only zero or positive values are in the
answer.

D 0, G 0 (Non-negativity constraints)

All constraints are plotted on to a graph and then moving away from the origin a solution
is sought where all constraint conditions are met and maximises the objective function.

Material A (4D + 2G = 1,800)

If D = 0 then:

4(0) + 2G = 1,800
2G = 1,800
G = 1,800 / 2
G = 900

If G = 0 then:

4D + 2(0) = 1,800
4D = 1,800
D = 1,800 / 4
D = 450

Material B (4D + 8G = 3,500)

If D = 0 then:

4(0) + 8G = 3,500
8G = 3,500
G = 3,500 / 8
G = 438

If G = 0 then:

4D + 8(0) = 3,500
4D = 3,500
D = 3,500 / 4
D = 875

140 | P a g e
Skilled labour (4D + 3G = 2,500)

If D = 0 then:

4(0) + 3G = 2,500
3G = 2,500
G = 2,500 / 3
G = 833

If G = 0 then:

4D + 3(0) = 2,500
4D = 2,500
D = 2,500 / 4
D = 625

Machine hours (7D + 9G = 6,500)

If D = 0 then:

7(0) + 9G = 6,500
9G = 6,500
G = 6,500 / 9
G = 722

If G = 0 then:

7D + 9(0) = 6,500
7D = 6,500
D = 6,500 / 7
D = 929

141 | P a g e
G
900 Material A (4D + 2G = 1,800)

800
Skilled labour (4D + 3G = 2,500)

700

Machine hours (7D + 9G = 6,500)


600
Max sales D = 400
500

Max sales G = 450


400

300
Material B (4D + 8G = 3,500)

200 Feasible
Region
100

100 200 300 400 500 600 700 800 900


D

Through observing the graph the solution appears to be to make 290 units of G and 315
units of D. This however is only as accurate as the graph drawn. The solution can also be
derived through simultaneous equations which is far more accurate than using the graph
or graphical method.

We know that our solution is where material A constraint intersects with the material
B constraint.

Material A 4D + 2G = 1,800 Equation 1


Material B 4D + 8G = 3,500 Equation 2

We can use the subtraction method or substitution method to solve. We will use the
subtraction method.

Therefore subtract equation 2 from equation 1

4D + 2G = 1,800 Equation 1
4D + 8G = 3,500 Equation 2
-6G = -1,700

142 | P a g e
6G = 1,700
G = 1,700 / 6
G = 283

Substitute G =283 into equation 1

4D + 2(283) = 1,800
4D + 566 = 1,800
4D = 1,800 566
4D = 1,234
D = 1,234 / 4
D = 308.5

Therefore to maximise contribution we should make:

308 units of D and 283 units of G.

Part (b)

The shadow price is the extra contribution earned if one more unit of the scarce resource
was made available. A shadow price only exists for scarce resource. Skilled labour has a
nil shadow price because at the optimal solution it is not fully utilised as there are some
skilled labour hours left. This can be seen in the graph in that the optimal solution lies
below the skilled labour constraint indicating that there are excess skilled labour hours
available.

Direct material A is a scarce resource because at the optimal solution in the graph we are
on the constraint itself and so we have fully utilised all this resource. This is further
indicated by the value of 5.82 above. If we were to obtain another unit of direct material
A then we would earn a contribution of 5.82. Also as a result of this our optimal
solution would change.

Part (c)

If the selling price of product D increased then this would have direct impact on the
objective function or iso-contribution line. It would change how the objective function
sloped.

In order to calculate how much the selling price would have to rise before the optimal
solution would change we would have to identify the extreme points of the feasible
region and the relative unit contributions of D and G that would cause a change in choice
of optimal solution in the objective function.

143 | P a g e
A2 8 Highly skilled workers (CIMA P2 May 2008)

Part (a)

The limiting factor is material M1 and the most profitable course action would be to
make those products that will maximise the contribution earned per square metre of M1.

P4 P6 C3 C5
$ $ $ $
Selling price
125 175 75 95
or opportunity cost

Variable cost (55) (75) (44) (45)


Contribution 70 100 31 50

M1 used at
0.75 0.50 0.25 0.50
$20 per square metre

Contribution 70/0.75 100/0.50 31/0.25 50/0.50


per square metre = $93.33 = $200 = $124 = $100

Rank in order
4 1 2 3
of production

Amount of M1 available is 1,000 square metres.

M1 used
P6 1,500 units x 0.50 750
C3 500 units x 0.25 125
C5 1,000 750 125 = 125 square metres left 125 / 0.50 = 250 units 125
1,000

Optimum production plan

Make (units)
P6 1,500
C3 500
C5 250
Other factors:

Exam tip: The examiner is only after three other factors, however we have given you
other factors that could be equally valid to discuss.

The non-production of P4 may have a detrimental impact on customers. They


may go to other suppliers who provide both P6 and P4 and not just P6.

144 | P a g e
P6 and P4 maybe complimentary products and so the non- production of P4 may
reduce the sales of P6.

The fixed overheads may have a variable element varying with production; if this
is the case then contribution will change for some of the products and therefore
the optimum production plan.

The stability of the cost of material M1 as if this increases then it may not be
beneficial to make C3 and C5 but to buy them in.

The future market of the products if they are going to continue to be popular with
customers or whether they are starting to fall out of favour because of better
products coming on to the markets to replace them.

The elasticity of demand of the products and in particular how sensitive demand
would be to changes in prices.

Part (b)

If we had more material M1 available then we would make a further 750 units of C5 and
then 2,000 units of P4.

The amount of M1 needed to make 750 units of C5:

750 units x 0.50 = 375 square metres

This would save $100 per square metre of M1 and so therefore the maximum price would
be $120 including the cost of M1.

The amount of M1 needed to make 2,000 units of P4:

2,000 units x 0.75 = 1,500 square metres

This would give contribution of $93.33 per square metres of M1 and so therefore the
maximum price would be $113.33 including the cost of M1.

After satisfying these requirements there is no further need for M1 and so therefore the
maximum price to be paid for M1 is zero.

Part (c)

If we were to supply this contract and we have no more M1 then we have to stop the
production of some or all of C5, C3 and P6. We should stop the production of those
products which give us the least amount of contribution per square metre of M1 first.

500 units of P4 would need 500 x 0.75 = 375 square metres of M1.

145 | P a g e
Therefore take material M1 as follows to fulfil the new order:

125 square metres from C5


125 square metres from C3
125 square metres from P6

Lost contribution:
C5 125 x $100 $12,500
C3 125 x $124 $15,500
P6 125 x $200 $25,000
Total $53,000

Contribution from the 500 units = 500 x $70 = $35,000

Net benefit from doing the 500 units = $53,000 - $35,000 = $18,000

Therefore the minimum financial penalty must be $18,000.

A2 9 RT (CIMA P2 May 2010)

Part (a)

In order to find the optimum production plan we must first establish what the scarce
resource is that is restricting production to meet all demand. We will work out the total
amount of resources needed to meet maximum demand and then compare this to the
resources that we have available to us to determine any scarce resources.

R T Total
Total demand 750 1,150
Direct labour (hours) 2,250 5,750 8,000
Material A (kg) 3,750 4,600 8,350
Material B (litre) 1,500 1,150 2,650
Machine hours 2,250 4,600 6,850

Direct labour hours is the scarce resource or limiting factor as we only have available to
us 7,500 hours and we need 8,000 hours.

146 | P a g e
Product R T
$ per unit $ per unit
Selling price 130 160
Direct labour ($8 per hour) (24) (40)
Material A ($3 per kg) (15) (12)
Material B ($7 per litre) (14) (7)
Machine hours ($10 per hour) (30) (40)
Contribution 47 61

Contribution per labour hour 47/3 = 15.67 61/5 = 12.20

Rank in order of production 1 2

Total labour hours available is 7,500 hrs, however we must fulfil a commercial customer
order first of 250 Rs and 350 Ts before working out the optimum production plan.

Commercial Labour hours per Labour hours


contract unit used
R 250 3 750
T 350 5 1,750
Total 2,500

Amount of labour hours available after commercial contract is 5,000 hrs.

Labour
hrs used
R 500 units x 3 hrs 1,500
T 5,000 hrs 1,500 hrs = 3,500 hrs 3,500 hrs / 5 hrs = 700 units 3,500
5,000

Optimum production plan

Commercial contract Production Total


R 250 500 750
T 350 700 1,050

Contribution per unit from commercial contract

Product R T
$ per unit $ per unit
Selling price 100 135
Direct labour ($8 per hour) (24) (40)
Material A ($3 per kg) (15) (12)
Material B ($7 per litre) (14) (7)
Machine hours ($10 per hour) (30) (40)
Contribution 17 36

147 | P a g e
Contribution from production plan

Contract Market
R 250 x $17 = $4,250 500 x $47 = $23,500 $27,750
T 350 x $36 = $12,600 700 x $61 = $42,700 $55,300
Contribution $83,050

Part (b)

By carrying out the commercial contract it means that we fall short of meeting market
demand of T by 100 units. The lost contribution of this is 100 units x $61 per unit =
$6,100. This is less than the financial penalty of $10,000 if the commercial contract was
not met, so therefore it is better at least from a financial perspective to fulfil the
commercial contract.

Part (c)

Revised resource Commercial Resources


availability contract remaining
Labour (hrs) 7,500 x 0.9 = 6,750 2,500 4,250
Material A (kg) 8,500 x 0.9 = 7,650 2,650 5,000
Material B (kg) 3,000 x 0.9 = 2,700 850 1,850
Machine hours 7,500 x 0.9 = 6,750 2,150 4,600

Objective function = 47R + 61T

Constraints:

3R + 5T 4,250 (Labour)
5R + 4T 5,000 (Material A)
2R + T 1,850 (Material B)
3R + 4T 4,600 (Machine hours)
R 500, T 800 (Maximum demand)

Logic or non-negativity constraints these are constraints which will ensure that the
answer obtained in the solution is sensible in that only zero or positive values are in the
answer.

R 0, T 0 (Non-negativity or logic)

All constraints are plotted on to a graph and then moving away from the origin a solution
is sought where all constraint conditions are met and maximises the objective function.

148 | P a g e
(Labour) 3R + 5T = 4,250

If R = 0 then:

3(0) + 5T = 4,250
5T = 4,250
T = 4,250 / 5
T = 850

If T = 0 then:

3R + 5(0) = 4,250
3R = 4,250
R = 4,250 / 3
R = 1,417

(Material A) 5R + 4T = 5,000

If R = 0 then:

5(0) + 4T = 5,000
4T = 5,000
T = 5,000 / 4
T = 1,250

If T = 0 then:

5R + 4(0) = 5,000
5R = 5,000
R = 5,000 / 5
R = 1,000

(Material B) 2R + T = 1,850

If R = 0 then:

2(0) + T = 1,850
T = 1,850

If T = 0 then:

2R + 0 = 1,850
2R = 1,850
R = 1,850 / 2
R = 925

149 | P a g e
(Machine hours) 3R + 4T = 4,600

If R = 0 then:

3(0) + 4T = 4,600
4T = 4,600
T = 4,600 / 4
T = 1,150

If T = 0 then:

3R + 4(0) = 4,600
3R = 4,600
R = 4,600 / 3
R = 1,533

T
2,000
Material B (2R + T = 1,850)
1,600
Skilled labour (3R + 5T = 4,250)
1,400

1,200 Machine hours (3R + 4T = 4,600)


Max sales R = 500
1,000

Max sales T = 800


800

600
Material A (5R + 4T = 5,000)

400 Feasible
Region
200

500 1,000 1,500


R

The optimal solution is the furthest point away from the origin within the feasible region;
therefore the optimal production plan is make 500 units of R and 550 units of T in
addition to the contract.

150 | P a g e
Part (d)

If there is an increase in optimism by mangers then the constraints would all move to the
right illustrating the availability of more resources and production would increase.
Currently the most constraining lines are labour hours and maximum demand for R.

If labour hours were increased then there would be further production of T as the labour
constraint would move to the right. This would continue up until the labour constraint
intersects with the material A constraint and maximum demand for R, it is at this point
material A also becomes a constraining resource and would need to be increased if
production were to further increase.

Clearly it can be said that in any event labour hours needs to increase if production were
to be expanded, other resources if increased would have no immediate effect as there are
resources in excess available.

A2 10 LM (CIMA P2 Nov 2010)

Part (a)

In order to find the optimum production plan we must first establish what the scarce
resource is that is restricting production to meet all demand. We will work out the total
amount of resources needed to meet maximum demand and then compare this to the
resources that we have available to us to determine any scarce resources.

L M Total
Total demand 400 700
Direct labour (hours) 1,600 1,400 3,000
Direct material (kg) 800 6,300 7,100
Machine hours 400 1,400 1,800

Direct material is the scarce resource or limiting factor as we only have available to us
6,000 kg and we need 7,100 kg.

Product L M
$ per unit $ per unit
Selling price 70 90
Direct labour ($7 per hour) (28) (14)
Direct material ($5 per kg) (10) (45)
Machine hours ($10 per hour) (10) (20)
Contribution 22 11

Contribution per kg 22/2 = 11 11/9 = 1.22

Rank in order of production 1 2

151 | P a g e
Amount of material available is 6,000 kg.

Kg used
L 400 units x 2 kg 800
M 6,000 kg 800 kg = 5,200 kg 5,200 kg / 9 kg = 577 units 5,193
5,993

Optimum production plan

Production
L 400
M 577

Contribution from production plan

L 400 x $22 = $8,800


M 577 x $11 = $6,347
Contribution $15,147

Part (b)

The agreed order of 250 units of product M should be worked out separately for the
resources needed to complete it first as this is a requirement, then we can see what
resources are left over to formulate our new resource constraints.

250 units of product M would need:


Direct labour = 2 hrs per unit x 250 units = 500 hrs
Direct material = 9kg per unit x 250 units = 2,250 kg
Machine hours = 2 hrs per unit x 250 units = 500 hrs

Resources have also been overestimated by 20% and need to be reduced before deduction
of resource usage by the agreed order of 250 units of M, therefore:

Direct labour = (3,500 hrs x 80%) - 500 hrs = 2,300 hrs


Direct material = (6,000 kg x 80%) - 2,250 kg = 2,550 kg
Machine hours = (2,000 hrs x 80%) 500 hrs = 1,100 hrs

Revised resource constrains:

Direct labour 4L + 2M 2,300


Direct material 2L + 9M 2,550
Machine hours 1L + 2M 1,100

Objective function:

C = 22L + 11M

152 | P a g e
Part (c)

Product L 400 and product L other value 0

The value 400 represents the amount of L we should produce in order to maximise
contribution given the resource constraints. The other value 0 means that we have no
further units of L to make as we have reached the maximum market demand for L. units.
There is no shortfall in demand.

Product M 194 and product M other value 506

The value 194 represents the amount of M we should produce in order to maximise
contribution given the resource constraints. The other value 506 means that we have not
met our maximum market demand of M by 506. We are only producing 194 units of M
where as the maximum market demand is 700. There is a shortfall in demand of 700 units
194 units = 506 units.

Machine hours 312

This value represents the number of unused machine hours left at the optimal production
point where contribution is maximised given current resource constraints.

We can proof this amount of unused machine hours by comparing what has been used to
the total amount of machine hours available.

L units produced = 400 x 1 hr per unit = 400 hrs


M units produced = 194 x 2 hrs per unit = 388 hrs
Total hours used = 400 + 388 = 788 hrs
Hours available = 1,100 hrs
Hours unused = 1,100 hrs 788 hrs = 312 hrs

Direct material $1.22

This is the shadow price for direct materials as it is a scarce resource at the optimal
production point where contribution is maximised given current resource constrains. The
shadow price is maximum price you should pay above the original cost for one more
extra unit of the scarce resource, in this case being one more kg of direct material.

The proof is that if we were given 1 more kg of direct material we would use it to
increase output of product M, because still has unfulfilled demand. Each unit of M
requires 9 kg, therefore 0.11 units of M could be produced from 1 kg of material. Each
unit of M yields a contribution of $11 and therefore 0.11 units yield $1.22 contribution.

153 | P a g e
Labour hours 312

This value represents the number of unused labour hours left at the optimal production
point where contribution is maximised given current resource constraints.

Contribution $10,934

This is the maximum contribution that can be earned given the constraints at the optimal
production point.

Proof:
Contribution from L units = 400 x $22 = $8,800
Contribution from M units = 194 x $11 = $2,134
Total contribution = $8,800 + $2,134 = $10,934

A2 11 Hotel (CIMA P2 May 2011)

Part (a)

Peak Mid Low Total


$ $ $ $ $ $ $ $
Room revenue 855,000 720,000 412,500 1,987,500
Guest related costs (W1) (184,680) (162,000) (108,000) (454,680)
Room costs (W2) (68,400) (81,000) (82,500) (231,900)
Avoidable general costs (W3) (225,000) (300,000) (375,000) (900,000)
Room / Guest contribution 376,920 177,000 (153,000) 400,920
Snacks
Gross contribution (W4) 4,617 12,150 8,150 24,867
Cook costs (5,000) (6,667) (8,133) (20,000)
(383) 5,483 (233) 4,867
Restaurant
Gross contribution (W5) 17,313.75 33,750 47,250 98,313.75
Staff costs (13,500) (18,000) (22,500) (54,000)
3,813.75 15,750 24,750 44,313.75
Total contribution 380,350.75 198,233 (128,483) 450,100.75
Non-avoidable general costs (75,000) (100,000) (125,000) (300,000)
Net contribution 305,350.75 98,233 (253,483) 150,100.75
Hotel annual fixed costs (200,000)
Hotel annual profit (49,899,25)

Workings

W1 Guest related costs

Season Days Rooms Occupants Guests Guest related costs


(D) (R) (O) (D x R x O) ($12 per guest per night)
Peak 90 95 1.8 15,390 184,680
Mid 120 75 1.5 13,500 162,000
Low 150 50 1.2 9,000 108,000
154 | P a g e
W2 Room costs

Peak = ($5 + $3) x 90 days x 95 rooms = $68,400


Mid = ($5 + $4) x 120 days x 75 rooms = $81,000
Low = ($5 + $6) x 150 days x 50 rooms = $82,500

W3 Avoidable general costs

Peak = $300,000 x 75% = $225,000


Mid = $400,000 x 75% = $300,000
Low = $500,000 x 75% = $375,000

W4 Snacks gross contribution

Peak = 15,390 guests x 10% x $10 x 30% = $4,617


Mid = 13,500 guests x 30% x $10 x 30% = $12,150
Low = 9,000 guests x 30% x $10 x 30% = $8,100

W5 Restaurant gross contribution

Peak = 15,390 guests x 30% x $15 x 25% = $17,313.75


Mid = 13,500 guests x 50% x $20 x 25% = $33,750
Low = 9,000 guests x 70% x $30 x 25% = $47,250

Part (b) (i)

Overall from our statement it is obvious that the hotel is making losses of nearly $50,000.
Actions management could take to maximise profits:

During the low season they could shut down the hotel as it is making the main
losses in this part of the year.

Close down the snack service during the peak and the low season as it makes
losses in these seasons but makes profits during the mid season when it should
remain open.

Part (b) (ii)

Other factors that the managers should consider:

It should be investigated as to how easily the snack service can be shut down and re-
opened only for the mid season.

If the snack service was closed entirely whether or not this would encourage guests to use
the restaurant.

155 | P a g e
If the hotel was closed during the low season whether or not this would still retain the
regular guests to come in the peak and mid season or whether guests will view this as
being an obvious move to focus on profits rather than customer service.

156 | P a g e
Solutions Section A

Part B Cost Planning and Analysis for Competitive Advantage

B1 1 SWAL (CIMA P2 Pilot Paper 2005)

This question had at least one mark for writing your answer in a report format!

REPORT
To: Managing Director of the SWAL group
From: Management Accountant
Subject: JIT differences and review of current quality control systems
Date: 19th September 2001

1. Introduction

This report is designed to explain how a just-in-time (JIT) system differs from the system
presently being used and the extent to which its introduction would require a review of
SWs quality control procedures.

2. JIT purchasing and production

The JIT philosophy states that products should only be produced if there is an internal or
external customer waiting for them. Traditionally manufacturers stockpiled. JIT aims
ideally for zero stock e.g. raw materials delivered immediately at the time they are
needed, no build up of work-in-progress, finished goods only produced if there is a
customer waiting for them.

1. Closer relationships with suppliers required, fewer and more frequent items, in
return for this the supplier would get long term steady purchase orders.
2. Smaller more frequent deliveries need to be managed in order to produce.
3. Higher quality machines perhaps multi-purpose with regular maintenance.
4. Involvement and training of staff to maintain flexibility.

3. The current system compared to JIT

The current system aims to procure chemicals based on holding a minimum quantity this
is very different to JIT. JIT would aim to always hold a zero or near zero stock level as
chemicals should be bought in when there is a demand for them not to hold a minimum
level.

To secure regular small amounts of chemicals bought in under JIT, SWAL group would
have to enter into long term contracts, and therefore order costs may increase using such
as stock control system.

157 | P a g e
However holding costs such as damage, deterioration, administration, security and
interest costs will all be significantly reduced under JIT. The EOQ will not be needed in
this situation because holding costs will be near zero and there would not be any
significant amounts of chemicals held.

4. JIT and quality control procedures

If SWAL adopts JIT in managing its stock levels then there maybe a lot of reliance on the
one supplier delivering on time and ensuring the chemicals are of satisfactory quality.
There is no room for error and it would mean production would have to stop as SWAL
would not carry any emergency stocks.

SWAL would need to build good working relationships with suppliers. One way of
achieving this is to have one supplier with whom SWAL has a long-term contract
guaranteeing many purchases or exclusivity to that supplier.

Quality control checks should be carried out on materials by supplier before it arrives at
SWAL to ensure quality of product. SWAL will not have the time to carry out detailed
checks as the chemicals would be needed straight away in production. If the chemicals
were not satisfactory then the supplier could face penalties, cancellation fees and perhaps
the termination of long-term contract.

Quality controls should be put into place in the production process to eliminate scrap, re-
works and defects.

There should be proper and regular maintenance of the machines used in production by
the staff using them, this will help to ensure an unbroken production line.

Staff should receive the necessary training in how to use the machines and maintain
them.

Quality circles, where a group of managers, normally working in the same area, volunteer
to meet on a regular basis, can be used to:

Identify areas for improvement.


Analyse work related problems in order to find alternative solutions.
Brainstorm for new ideas or alternatives.
Investigate report and discuss findings.

There is much that is needed to be changed an improved if we are to adopt a JIT approach
and should be planned carefully, and the benefits of higher quality and cheaper
production will only be seen in the long term. I would suggest many consultations with
myself and other experts in this field before embarking on such a stock control system.

158 | P a g e
I hope you have found this report useful but should you require any further assistance or
have any questions please do not hesitate to contact me.

Signed : Management Accountant.

B1 2 X group (CIMA P2 May 2005)

This question had two marks for explaining and defining JIT and the rest of the eight
marks were for profitability would be affected.

REPORT
To: Managing Director of the X group
From: Assistant Management Accountant
Subject: Adoption of JIT and its impact on profitability
Date: 19th September 2005

1. Introduction

This report is designed to explain how the adoption of JIT might affect X groups
profitability.

2. JIT purchasing and production

The JIT philosophy states that products should only be produced if there is an internal or
external customer waiting for them. Traditionally manufacturers stockpiled. JIT aims
ideally for zero stock e.g. raw materials delivered immediately at the time they are
needed, no build up of work-in-progress, finished goods only produced if there is a
customer waiting for them.

Closer relationships with suppliers required, fewer and more frequent items, in
return for this the supplier would get long term steady purchase orders.
Smaller more frequent deliveries need to be managed in order to produce.
Higher quality machines perhaps multi-purpose with regular maintenance.
Involvement and training of staff to maintain flexibility.

3. The impact on profitability

By the introduction of JIT, X group should see the following effects on profits:

1. Reduction in stock holding costs for raw materials, finished goods and work in
progress.
2. Reduction in number of suppliers used to try to ensure quality and as a result may
pay slight premium.

159 | P a g e
3. Increase in price of raw materials from suppliers for regular small amounts
ordered at short notice, but maybe reduced through the introduction of long-term
contracts with suppliers.
4. Additional costs incurred in improved quality control procedures put in the
production process at X group.
5. Staff training costs to ensure that they are able to carry out control procedures and
use the material efficiently.
6. Additional planning is required by management to ensure that there are no stock
outs, and therefore addition administration costs.

I hope you have found this report useful but should you require any further assistance or
have any questions please do not hesitate to contact me.

Signed : Assistant Management Accountant.

B1 3 ML (CIMA P2 Nov 2005)

Part (i)

Traditional absorption costing takes the total budgeted fixed overhead for the period and
divides by a budgeted (or normal) activity level in order to find the overhead absorption
rate

The overhead absorption rate is then used to absorb fixed overheads into the range of
products we provide. Once all costs have been allocated then a price is obtained by
adding a % mark up on the costs. The mark up would represent the profit earned on the
product.

This approach is used by companies that have a single or small range of products, fixed
overhead are a small percentage of total cost, and mass production techniques are used.

ML engineering uses an absorption costing approach based on machine hours. This is


particularly useful in this circumstance as it means that every product will always recover
a portion of the production fixed costs. However it may not be a fair method as not every
product will consume the fixed overheads based on the single activity being machine
hours. Some products may not use machine hours but still consume fixed production
overheads.

The main consequence of this is that prices charged on the various products may not be
competitive, due to the fact the cost plus approach depends on the total costs allocated
to that product. As a result of this some products make losses because fixed production
costs being unfairly allocated to it and therefore discontinued when in fact it maybe
making profit.

160 | P a g e
The alternative being a marginal cost approach would seek to ignore the fixed production
overheads. The difference between absorption costing and marginal costing organisations
is that the marginal costing organisation makes no attempt to absorb production
overhead into a cost unit.

It treats production overhead as a period cost only and does not absorb overhead, but
rather charges it entirely to the profit and loss account for each period. It is equally
important to remember that marginal costing organisations would also value stock at
variable production cost only not full production cost, as does an absorption costing
company.

This means that pricing is more competitive as costs would be lower and it means that
unused capacity in ML could be used as more contracts would be secured. The main
problem with this is that ML still may not be competitive as the mark up maybe too high.

Part (ii)

Quite clearly the Managing Director is suggesting a marginal costing approach to price
by making this comment. A marginal costing approach is very good approach in but only
in the short term, this is because fixed costs have already been spent or committed to and
by ignoring these and concentrating on those costs that do change in the short term it
means that we are covering all the costs that we would incur, but only in the short term.

Therefore any price that exceeds variable costs would be making some contribution to
fixed costs, but this may not be sustainable over the long term and fixed costs need to be
recovered at some point and enough profit needs to be made for the products to be
continued to be made.

By ignoring fixed production costs there is no attempt to understand and control them to
ensure that they are costs that should be spent and used effectively. Using a marginal
costing approach there is danger that these costs could spiral out of control over the long
term.

In addition by charging a lower price initially it maybe difficult to justify to customers


price increases especially if the market is competitive.

B1 4 PK plc (CIMA P2 Nov 2005)

REPORT
To: Management Team of PK plc
From: Assistant Management Accountant
Subject: Changing nature of cost structures in the modern environment
Date: 12th December 2005

161 | P a g e
1. Introduction

This report is designed to explain the changing nature of cost structures in the modern
manufacturing environment and the implications to PK plcs inventory valuation and
short term decision making.

2. Cost structures in the modern manufacturing environment

Absorption costing system used by PK plc takes the total budgeted fixed overhead for the
period and divides by a budgeted (or normal) activity level in order to find the overhead
absorption rate.

This approach is used by companies that have a single or small range of products, fixed
overhead are a small percentage of total cost, and mass production techniques are used.

It is a system that has been used for over a hundred years now and is perhaps seen as
inappropriate today because of increased competition from overseas companies,
economies of scale enjoyed by many companies, trade barriers and barriers to entry into
many markets being reduced significantly or abolished, improvements in technology and
access that customers have around the world to other manufacturers has made the global
economy into a local economy.

PK plc is in an industry producing high quality pieces of furniture and where labour skills
are very important. High quality furniture manufacture tends to have a lot of different
machines which have specific in built programs. This means there are a range of different
machines and different types of fixed production overheads not just one type these days.
Therefore we need to understand how these are driven by relating these to activities
which cause their expenditure and as a result control the costs far better.

So unlike traditional absorption costing where you have one or two costs drivers we are
now seeking several cost drivers relating to different segments of the total fixed
production costs and relating this to how products consume them. This is a modern
technique known as activity based costing (ABC).

3. Inventory valuation

We can obtain an inventory valuation using the absorption costing method which
includes a portion of fixed production costs in the valuation. Inventory valuation is
important because it is a reporting requirement but also gives you an understanding as to
how much money is being tied up in these products that are being manufactured and how
much profit is being made once the items have been sold.

Financial reporting requirements state that stock valuation should include all costs
bringing the item to its present location. This shows that stock valuation is not based on
change of activity but on a blanket absorption rate and therefore not giving a true
reflection of cost of the product and therefore profitability.

162 | P a g e
4. Short-term decision making

When making decision in the short term then we must include those cost and revenues
which change as a result of our decision, and ignore those that dont. This normally
means that we are interested in variable costs only as fixed costs are normally historical
or committed. Fixed production costs cannot be changed in the short term.

If this is the case indeed PK plc should focus on making decisions looking at how much
contribution (sales less variable costs) they can earn rather than profit (sales less all
costs). This will make them more competitive against their rivals, being able to charge a
lower price and still earning contribution to fixed costs.

But of course over the long term they cannot take a marginal costing approach because
they would need to recover the fixed production costs as well.

ABC has many uses and there are very good benefits to an organisation when formulating
their long term strategic decisions, such as product pricing, mix of products,
discontinuance, launch or promotion of existing products and the launch of new products.

ABCs strength lies in the fact it allows accuracy over costs and drivers for products and
as a result a sensible pricing strategy is achieved. It more specifically gives a good long
term understanding of the variable costs being very relevant for decision making.

However ABC information must be put into perspective as these are historic costs and
cannot be used alone to predict future costs. They should be used as a starting point and
other internal and external information should be used to determine future costs. All costs
are variable in the long term and subject to change.

I hope you have found this report useful but should you require any further assistance or
have any questions please do not hesitate to contact me.

Signed : Assistant Management Accountant.

B1 5 Financial advisors (CIMA P2 May 2006)

Part (a)

Value Analysis

Value analysis focuses on removing those costs during production which do not add
value to the product because the customer does not actually want them as they are not
viewed by the customer as adding any value.

163 | P a g e
Value engineering is the activity which looks at how to achieve the same quality product
for customers at the lowest costs possible. It focuses before on costs before production
has begun whereas value analysis focuses at costs during production.
Although value analysis is another form of cost reduction it is different in that it seeks not
just to reduce costs as far as possible but takes into account what is important to
customers.

Four aspects of value:


Cost value cost of production.
Exchange value sales price.
Utility value what its functionality is.
Esteem value prestige that it is perceived hold.

Value analysis seeks to reduce cost value while maintaining the esteem value of a
product. It normally is easier reduce the cost value on items which have a utility
value as opposed to an esteem value.

Functional analysis

This looks at the functions that a product has and their perceived value to customers
versus the cost of providing these. There are nine basic steps involved in functional
analysis:

1. Choosing the object of analysis product, service or overhead area where there is
high cost, complex, low demand.

2. Select the members of the team different departments.

3. Gather information internal and external.

4. Define the functions of the object list all the functions of product (e.g. functions
of a pen?).

5. Draw a functional family tree showing relationship of function to part product


and its cost.

6. Evaluate the function arrive at a total target cost and compare to customers
expectations.

7. Suggest alternatives and compare these with the current or target (expected) cost
new methods and materials.

8. Choose the alternative operations.

9. Review the actual results.

164 | P a g e
Part (b)

Value analysis method

1. Obtain the precise requirements of the customer by considering utility, esteem and
exchange value.

2. Obtain alternative ways of manufacturing the requirements of the customers


product. For example, the newsletter that the firm produces is not read by most
clients, perhaps looking at alternative ways to convey this information to clients
such as email, meetings, free one off business advice days.

3. Authorise the suggested alternative methods in achieving the customer desires by


senior management, before production begins.

4. Produce and manufacture product.

5. Evaluate changes and response from customers. Benefits of this maybe positive
feedback from customers, improved sales and profitability, eliminate wastage,
attract more staff and better morale within the company.

B1 6 Compliance v conformance (CIMA P2 May 2006)

REPORT
To: Managing Director
From: Management Accountant
Subject: Quality costs
Date: 12th February 2006

1. Introduction

This report is designed to discuss quality costs and their significance for the company,
and in particular costs of conformance and costs of non-conformance.

2. Costs of conformance

Total quality management (TQM) is the process of adopting a quality conscious


philosophy within staff culture, as well as adopting standards and procedures e.g. such as
ISO 9000 certification to monitor and control quality. Benchmarking, quality circles and
investment in training and development of employees often achieve this.

Conformance costs are those costs which are spent to try to achieve a standard or target,
such regulations or functional specifications of a product for a customer.

165 | P a g e
There are two main types of conformance costs:

Prevention costs these are spent to try and ensure the product is made to detailed
specifications and regulations. For example training staff, customer surveys,
supplier reviews and investment in machines.
Appraisal costs these are spent to understand how well a process has performed
and corrective action is taken if needed subsequently. For example measuring
equipment, inspections and tests, product quality audits.

3. Costs of non-conformance

Non-conformance costs are quality failure costs that are needed to correct products, as
they did not meet expectations or target.

There are two main types of non-conformance costs:

Internal failure costs these are quality failure costs before the products or
services have been transferred to the customer. For example re-inspection of
goods, losses or scrapping of materials and finished goods, additional
administrative costs.
External failure costs - these are quality failure costs before the products or
services have been transferred to the customer. External costs should be avoided
as they expose poor manufacturing abilities to customers. Examples are
administration of customer complaints, administration of customer services,
product liability claims, repairs and replacements, lost good will and reputation.

4. Importance of quality costs

In the modern manufacturing environment customers are demanding better quality


products at cheaper prices and on demand. This has been due to increased competition
through global networks, deregulation and privatization of industries, increased
innovation and use of automation, shorter product lifecycles and more efficient
manufacturers.

This has meant manufacturers these days must consider spending more and more of their
money on producing better quality items, through improved processes and work with
high quality materials.

Companies these days must consider what their target audience is and what is important
to them be it quality, cost, service or speed of delivery. Then consider what they need to
spend on conformance costs and non-conformance costs.

Companies must decide whether to undertake a check of all products or inspection by


random sample of the products before they leave for customers. They should consider
which is appropriate for their customers needs.

166 | P a g e
Although conformance costs are high costs in the short term, they would yield greater
benefits in the long term in the form of greater profitability and longevity for the
business.

Manufacturing companies could use Kaizen costing as way of improving the long-term
quality of their products. It is the process of continuous improvement by small
incremental rather than transformational changes. Its also believes strongly in
empowerment of employees to enable them to improve operations. Kaizen costing
focuses on reducing variable costs of future periods below that of prior periods. Similar
in concept to total quality management philosophy, Japanese idea and again works hand
in hand with quality circles, benchmarking or quality assurance.

Human resources are your most valued asset and they should be involved in the
search for perfection. They have superior knowledge as they are in the operation.
This is the opposite of traditional systems where the managers develop standards.
Incremental or gradual improvement.
Perfection should be sought all the time and so there is always room for
improvement.
Cost reduction targets more frequent rather than traditional annual standard costs.
Variance analysis used to help with Kaizen costing.
Investigations carried out if targets not met even if improvements have been
made.

I hope you have found this report useful but should you require any further assistance or
have any questions please do not hesitate to contact me.

Signed : Assistant Management Accountant.

B1 7 AVN (CIMA P2 Nov 2006)

Question 2

Part (i)
Porter grouped the various activities of an organisation into what he called the value
chain; he divided the organizations activities into nine types, classified as either primary
or secondary activities. These activities incur costs, but in combination with other
activities provide customer satisfaction and therefore add value.

The main components are as follows:

Technology - R & D, product design and testing, process design and testing.

Inbound Logistics - Receiving, handling and storing inputs (warehousing, stock


control, inbound transport).

167 | P a g e
Operations - Conversion of inputs (people, materials, machines etc) into the final
product or service.

Marketing & Sales - Informing the customer (Marketing mix).

Outbound logistics - Delivery of the actual product or service (storage, outbound


transport to customer).

Service - Customer service, repairs, installations, supply of parts/consumables.

The extended value chain is including external factors into the chain such as suppliers
and customers creating a value system, an example of additional components are as
below:

Supplier
value
chain
Distributor value
chains
Supplier
value
chain
Retailer
Wholesaler
value
value chain
chains
Supplier
value
chain

Part (ii)

AVN manufactures electronic devices which appear to be quite sophisticated. This is a


fast moving, volatile and competitive market and therefore AVN needs to ensure that
they are supporting each of the main components of the value chain in order to be
successful.

Each of the main components may be applied to AVN as follows:

Technology AVN must make sure that their R&D is directed towards the needs of the
customer as there is no point developing a product which consumers do not want to
purchase. Functional analysis would be a useful way of achieving this. Once the
prototype has been developed AVN must make sure that it is designed and tested to make
it easy for the consumer to operate and be durable and robust enough for its purpose.

168 | P a g e
The product must also be cost effective and so target costing should be used here to
determine the market price and profit desired and then the balancing figure being the
budget for costs, which must be realistic.

Inbound logistics The material required for this product must be sourced from
reputable and reliable suppliers. They must be checked for quality assurance and this
would reduce level of assurance needed in AVN. Re-works and customers returns are in
the long run more costly to rectify than making sure it was correct the first time it was
manufactured. Cost should also be considered to make sure that the service is delivered at
rate where AVN can make profits. Storage should also be suitable for these materials and
be secure with additional capacity if needed.

Operations Commercial production must be cost effective and efficient. There should
be coordination between inbound logistics and production determining the level of
materials required. There should also be an understanding of what types of products
should be made and the mix of labour required to achieve this. There should also be
investment in quality machinery and quality controls to prevent internal and external
failure costs.

Marketing and sales It is important that AVN informs the right target consumers of
the new product that is being launched and its main benefits over older models or other
substitutes supplied by other competitors.

Outbound logistics This is the delivery of the products to consumers. It is important


that AVN deliver these items in perfect condition, on time and the right products. AVN
must employ suitably trained sales team and distribution team to meet consumer needs,
and also consider whether this will be cost effective.

Service It is vital for AVN to receive continual feedback on their products from the
consumers to ensure that their needs are being fulfilled exactly, and if needs are changing
these are also met by creating revised models. AVN should be looking to deliver a high
quality after sales service to consumers in the way of dedicated support lines,
replacement of parts and installations if required, all of which should be cost effective.

169 | P a g e
B1 8 W (CIMA P2 Nov 2006)

Part (a)

Maturity Decline
Months 31 - 70 71 - 110
No. of units produced and sold 20,000 20,000 20,000
SP per unit $60 $60 $40
VC per unit $30 $25 $30
Contribution per unit $30 $35 $10
Total contribution $600,000 $700,000 $200,000
Cumulative cash flow $1,275,000 $1,475,000

Part (b)

Changes in costs

There has been a continued reduction in unit variable costs of this product from its
introduction stage to its maturity stage. It can be seen from the data given that as we
produce more of the product unit variable cost falls from $50 to $25 per unit. This can be
attributable to economies of scale being enjoyed by the company through mass
production techniques, such as below:

Learning curve effects enjoyed by the workforce if the production is labour


intensive, repetitive and most of the staff is retained over the long term.

Technical efficiencies learned by all departments in the manufacture of the


product, especially the production department through improved understanding of
how material and machines can be used more efficiently to maximise production.

Stock control systems maybe more efficient as the data states the company
employs a JIT approach to stock management. This reduces stock holding costs to
nil and material is only used when and bought if there is a demand.

Discounts would have been received as the company would buy more and more
bigger batches of the raw material throughout the products life cycle.

Total quality management techniques maybe used which means that the product is
made to a very high standard reducing internal failure costs such as the inspection
and scrap material and also reduction in external failure costs such as repairs and
replacement.

170 | P a g e
The unit variable cost has also increased at the end of the product life cycle from $25 to
$30 in its decline stage. This is expected as the demand for this product would be reduced
significantly as it would have fallen out of favour with customers, and therefore less is
being made and as a result fewer materials needed. The lower prices on raw materials
cannot be enjoyed because of the reduced quantities bought by the company and hence
the increase in unit variable costs.

Changes in selling prices

W has managed to initially charge a high price, achieve low volume but earned a larger
profit per unit sold. This is known as market skimming. It is a strategy which exploits a
price insensitive market or an inelastic demand for a product.

It would seem that Ws product has been a product that customers have been willing to
pay a premium for but this not sustainable as competitors enter the market. This can be
seen during the growth stage as competition begins to enter the market W has had to
reduce its selling price from $100 per unit to $80 and then $70 per unit. W will still
continue to increase contribution through economies of scale.

During the maturity and decline phase W has had to continue to reducing the selling price
of their product in order to stay competitive as cheaper substitutes have entered the
market produced by other manufactures.

This approach will aim to sustain its wide customer base and W should continue to
produce the product as long the selling price is greater than the marginal cost of
manufacturing the product. We can see from our answer in (a) that this is still the case for
W in the decline phase. Production should cease when this is not the case as the there
would be no extra benefit for the company, at this point the product has fallen out of
favour with customers.

B1 9 New product (CIMA P2 May 2007)

Part (i)

The company did originally estimate the labour costs for the first batch to be $250,000,
however the actual cost for the first batch was $280,000. There is a learning curve rate of
80% which we will apply to the actual cost to obtain the revised expected cumulative
labour costs.

Number of cumulative batches Average labour cost Total labour costs


(000s) (000s)
1 $280 $280
2 $280 x 0.8 = $224 224 x 2 = $448
4 $224 x 0.8 = $179.20 179.20 x 4 = $716.80
8 $179.20 x 0.8 = $143.36 143.36 x 8 = $1,146.88

171 | P a g e
Part (ii)

Number of cumulative batches Average labour cost Learning curve rate


(000s)
1 $280
2 $476 / 2 = $238 $238 / $280 = 0.85
4 $809 / 4 = $202.25 $202.25 / $238 = 0.85
8 $1,376 / 8 = $172 $172 / $202.25 = 0.85

Learning curve rate at each output is 85%.

Part (iii)

The labour costs estimated were different to the actual labour costs. The actual initial
batch labour cost was more than the estimated cost by $30 but the actual learning rate
was 5% better than the estimated. This will have an impact on the labour costs involved
and may have an overall reduction in costs. This would lead to a more competitive
pricing strategy, increased sales and profits, if the price could be reduced.

B1 10 New small company (CIMA P2 Nov 2007)

Part (a)

Exam tip: You need only mention the first 2 paragraphs below to obtain the 3 marks
for this part. The further paragraphs subsequent are to show other relevant things that
are just as valid.

Learning curve theory is the theory that as output doubles the average time per unit (total
time for all units divided by the units produced) drops by a fixed percentage each time
this happens. The same theory can be applied to find the average cost per unit as well as
the average time.

Labour intensive production processes like the one used in the company would mean that
the humans used in this process will learn to do their job more efficiently and so over
time the labour time needed would fall. The learning curve theory can calculate this fall
in labour time required.

Being a new small company it therefore has new products and so would be in the early
stages of the product life cycle and production, therefore there would be room for
improvement in terms of labour needs.

If the industry that the new small company is in is very competitive then pricing on long
run average costs can consider the extent to which costs will fall because of the learning
curve effect.

172 | P a g e
Realistic standard costs could be achieved if learning curve theory is used to judge labour
performance and would reflect anticipated learning benefits.

Budgeting for manpower needs would be far more accurate if the learning curve concept
is adopted and so therefore would safeguard against over manning the production
process.

Part (b)

Y = aXb

Y = average time for that (X) number of units or the average cost per unit
a = time for the first unit or the cost for the first unit
X = the number of units you want to calculate an average time or cost for
b = the index of learning (log r/log 2)

a = 40 mins, b = -0.415

To work out the time taken for the sixth unit of output:

Time for the first 6 units = 40 x (6 to the power of 0.415) x 6 = 114.1 mins
Time for the first 5 units = 40 x (5 to the power of 0.415) x 5 = 102.6 mins
Time for the 6th unit = 114.1 mins 102.6 mins = 11.5 mins

Part (c)

Market penetration or a penetration pricing policy is the decision to charge a very low
price in order to capture a larger market share quicker. This strategy is used when
introducing a new product into an established and competitive market.

This policy also has the following implications:

The economies of scale for the product should be achieved quicker as market
share is obtained rapidly.
Competition may be discouraged and leave the market due to the low price being
charged.
This kind of pricing policy is good for inferior products as they are cheap to
manufacture and so costs are low and therefore the price charged can be kept low.

The company should also seek to ensure that all costs are recovered in the long run or
over the life of the product. There are two ways of achieving this:

Lifecycle costing recording all costs and revenues separately and so being able
to monitor if costs are being recovered.

173 | P a g e
Experience curves show how costs decline over the life of the product because
of the learning curve effect.

B1 11 XY (CIMA P2 May 2008)

Exam tip: The examiner very often gives between 1 and 3 marks for presenting your
answer in the report format as requested in the question. So you must do this and not lose
these easy marks!

REPORT
To: Managing Director of XY
From: Management Accountant of XY
Subject: The concept of the Value Chain
Date: 21st May 2008

1. Introduction

This report is designed to explain the points that should be covered in the presentation on
Value Chain and the management of profits generated throughout the chain in XY.

2. The concept of Value Chain


Porter grouped the various activities of an organisation into what he called the value
chain; he divided the organizations activities into nine types, classified as either primary
or secondary activities. These activities incur costs, but in combination with other
activities provide customer satisfaction and therefore add value.

The main components are as follows:

Technology - R & D, product design and testing, process design and testing.

Inbound Logistics - Receiving, handling and storing inputs (warehousing, stock


control, inbound transport).

Operations - Conversion of inputs (people, materials, machines etc) into the final
product or service.

Marketing & Sales - Informing the customer (Marketing mix).

Outbound logistics - Delivery of the actual product or service (storage, outbound


transport to customer).

Service - Customer service, repairs, installations, supply of parts/consumables.

174 | P a g e
Value chain analysis is underpinned by the concept of quality which involves the
following:

Commitment to developing processes that achieve high product quality and


customer satisfaction
Commitment to continuous improvement
Involvement of the entire workforce
Quality assurance through statistical method a key component
The meaning of quality can be subjective but generally it can mean non-inferiority,
superiority or usefulness of the product or service the customer is buying. A common
interpretation of quality is "fitness for its use or purpose".

TQM is the process of embracing a quality conscious philosophy or culture, as well as


adopting quality standards and procedures within an organisation, aiming towards
perfection and continuous improvement.

Kaizen, the Japanese equivalent to TQM means continuous improvement by small


incremental steps.

I hope you have found this report useful but should you require any further assistance or
have any questions please do not hesitate to contact me.

Signed : Management Accountant.

B1 12 Inventory levels (CIMA P2 May 2008)

Part (a)

Exam tip: The examiner is only after three cost changes being explained, however we
have given you other cost changes that could be equally valid to discuss.

If Just-In-Time (JIT) were introduced the following cost changes would result:

Reduction in stock holding costs to nil as material is only used and bought if there
is a demand being the philosophy of JIT.

Reduction in stock holding costs for finished goods and work in progress as all
items will be sold to waiting customers.

Reduction in the number of suppliers used to ensure quality and consistency as


material would go straight into production and there would be no time for quality
control. This would increase the cost of materials however there would equally be
a reduction to nil of internal quality control procedure costs.

175 | P a g e
Increase in price of raw materials from suppliers for regular small amounts
ordered at short notice, but maybe reduced through the introduction of long-term
exclusive contracts with suppliers.

Quality control costs will have to be put into place in the production process to
eliminate scrap, re-works and defects which would delay despatch of goods to
customers.

Staff training costs to ensure that they are able to use the new machinery
efficiently and effectively and be able to maintain them.

Additional planning is required by management to ensure that there are no stock


outs as JIT is based on the idea of holding zero stock, and therefore addition
planning costs are needed.

Part (b)

Total quality management (TQM) is important to a company that operates a JIT


production method because TQM means that the product is made to a very high standard
reducing internal failure costs such as the inspection and scrap material and also
reduction in external failure costs such as repairs and replacement.

JIT requires a TQM approach in order to operate holding zero stock levels and only
producing if there is demand, being the philosophy of JIT. This would mean whatever is
made must be to a high level of quality the first time and every time as there is no spare
material to re-make a product that is defectively made. If defective products were made
then production would have to stop while more material is sourced and the product re-
made, this may cause a loss of sales and customer goodwill as we would not be able to
deliver the product on time.

If the defective product were to be sold on to the customer this would cause the product
to be sent back for repairs and customer confidence and goodwill being lost.

If TQM were adopted then it would encourage a quality conscious philosophy within
staff culture, as well as adopting standards and procedures.

176 | P a g e
B1 13 Workshop (CIMA P2 May 2008)

Part (a)

Y = aXb

Y = average time for that (X) number of units or the average cost per unit
a = time for the first unit or the cost for the first unit
X = the number of units you want to calculate an average time or cost for
b = the index of learning (log r/log 2)

a = 40 hours, b = log 0.8 / log 2 = -0.3219

To work out the time taken for the 8th batch:

Time for the first 8 batches 40 x (8 to the power of 0.3219) x 8 = 163.85 hours
Time for the first 7 batches 40 x (7 to the power of 0.3219) x 7 = 149.67 hours
Time for the 8th batch = 163.85 hours 149.67 hours = 14.18 hours

Part (b)

The learning rate has reduced between months 2 and 4 from 75% to 90%. This
maybe due to a number of reasons:

An increase in staff turnover and so new staff have to learn how to make these
products efficiently and effectively and so require more time than the experienced
staff who have left.

A gradual reduction in how much more that can be learned by the existing work
force in the manufacturing of the product.

Inferior materials being introduced and so have led to more re-works and longer
production times.

Production is no longer in its early stages and so great improvements have already
been had.

Motivation and enthusiasm of the existing staff has fallen due to lack of
incentives in perhaps skills or pay, and so therefore taking longer to produce the
items.

177 | P a g e
Part (c)

a = 45 hrs, y = 182.25 hrs / 8 batches = 22.78 hrs on average for 8 batches

If we assume that r represents the learning rate, then:

Batches Average hours per unit


1 45
2 45 x r
4 45 x r x r
8 45 x r x r x r

Therefore for 8 batches:


45r = 22.78
r = 22.78 / 45
r = 0.506
r = (0.506) to the power of 1/3
r = 0.797

Therefore learning rate is 80%

B1 14 Out-turn performace report (CIMA P2 May 2010)

Part (a) (i) (ii)

Flexed budget Actual Variance


Output (batches) 50 50
Direct labour hours 68.91 hours (W1) 93.65 hours 24.74 hours (A)
Direct labour cost ($) $826.92 (W2) $1,146 $319.08 (A)

Direct labour efficiency variance = 24.74 hrs x $12 per labour hr = $296.88 (A)
Direct labour rate variance = $319.08 - $296.88 = $22.20 (A)

Workings

(W1) Direct labour hours

The learning curve ceases once we reach 30 batches, meaning that labour will not get any
faster in the production of any more units. Therefore we need to work out the total time
for 30 batches and compare with the total time for 29 batches to obtain the time taken to
make the 30th batch and use that as the time needed to make any further batches.

178 | P a g e
Using the learning curve formula:

Y = aXb

Y = average time for that (X) number of units or the average cost per unit
a = time for the first unit or the cost for the first unit
X = the number of units you want to calculate an average time or cost for
b = the index of learning (log r/log 2)

a = 10 hours, b = -0.5146

Work out the average time for 30 batches:

Y = 10 x (30 to the power of 0.5146) = 1.737 hours


Total time for 30 batches = 1.737 x 30 = 52.11 hours

Work out the average time for 29 batches:

Y = 10 x (29 to the power of 0.5146) = 1.768 hours


Total time for 29 batches = 1.768 x 29 = 51.27 hours

Time for the 30th batch = 52.11 hours 51.27 hours = 0.84 hours

Total time for 50 batches = 52.11 hours + (20 batches x 0.84 hours) = 68.91 hours

(W2) Direct labour cost

68.91 labour hours x $12 per labour hour = $826.92

Part (b)

The original budget did not take into account the revised expectations of the
learning curve for direct labour and therefore making comparisons is meaningless.

The original budget was not adjusted or flexed for the actual level of output to
obtain a fair basis of what should be expected to be used in terms of resources and
costs.

The revised out-turn performance report analyses the labour cost variance further
into the efficiency variance and labour rate variance, thus allowing improved
understanding of the real cause of the variance and then being able to assign
responsibilities to the appropriate managers.

179 | P a g e
B1 15 PQ (CIMA P2 May 2010)

(i) Selling price

PQs product being a consumer electronic product will have been a product that
customers are willing to pay a premium for but this will not be sustainable as competitors
will eventually enter the market. This will be seen during the growth stage as competition
begins to enter the market, PQ will have to reduce its selling price; however PQ will still
continue to increase contribution through economies of scale.

During the maturity and decline phase PQ will have to continue to reduce the selling
price of their product in order to stay competitive, as closer more competitive substitutes
will enter the market produced by other manufactures, these substitutes would have been
created by manufactures purchasing PQs product and reverse engineering the product.

This approach will aim to sustain demand and contribution from PQs product as long the
selling price is greater than the marginal cost of manufacturing the product. The cash
flow will be used to develop other products in development.
In the decline phase PQ will further reduce the selling price of the product and it will
eventually cease as there will be no extra benefit from this product as the product will
have fallen out of favour with customers, and any of the product left will be sold at a
vastly discounted selling price to clear it out of PQs inventory. The new more advanced
replacement product will have been fully introduced.

(ii) Production costs

There will be a continued reduction in unit production costs of this product during its
growth stage but unlikely to see any further reductions beyond this stage. This will be
attributable to economies of scale being enjoyed by the company through mass
production techniques, such as below:

Learning curve effects enjoyed by the workforce because the production is labour
intensive, repetitive and most of the staff is retained over the long term.

Technical efficiencies learned by all departments in the manufacture of the


product, especially the production department through improved understanding of
how material and machines can be used more efficiently to maximise production.

Stock control systems will be more efficient if the company employs a JIT
approach to stock management. This will reduce stock holding costs to nil and
material is only used or bought if there is a demand.

Discounts will have been received as the company would buy more and more
bigger batches of the raw material throughout the products life cycle.

180 | P a g e
Total quality management techniques will be used which means that the product
will be made to a very high standard reducing internal failure costs such as the
inspection and scrap material and also reduction in external failure costs such as
repairs and replacement.

During the maturity phase there is unlikely to be any further significant cuts in unit
variable costs, and in the decline phase the unit production cost will begin to increase.
This is expected as the demand for this product will be reduced significantly as it will
have fallen out of favour with customers, and therefore less is being made and as a result
fewer materials needed. The lower prices on raw materials cannot be enjoyed because of
the reduced quantities bought by the company and hence the increase in unit production
costs. In addition production costs will increase due to machine breakdowns and
inefficiencies.

(iii) Selling and marketing costs

During the growth stage there will be much reduced expenditure on selling and marketing
costs as there will be wide customer awareness of the product already and a minimal
amount of expenditure will be required to reinforce the continued customer awareness of
the product.

During the maturity stage such costs will be reduced further as the product is being more
sold on reputation and word of mouth. Selling and marketing costs are not bringing any
further benefit.

During the decline stage selling and marketing costs will cease as the product will fall out
of favour with customers and will mainly become obsolete and production of which will
also cease.

B1 16 Timber products (CIMA P2 May 2010)

Part (a) (i) (ii)

Month Demand Basic Inc/(Dec) Closing Ave Inventory Overtime


Std Hrs Production Inventory Inventory Inventory Holding Costs Costs if using JIT
Std Hrs Std Hrs Std Hrs Std Hrs ($6 per month) ($15 per hour) (W1)
1 3,100 3,780 680 680 340 2,040
2 3,700 3,780 80 760 720 4,320
3 4,000 3,780 (220) 540 650 3,900 3,437.55
4 3,300 3,780 480 1,020 780 4,680
5 3,600 3,780 180 1,200 1,110 6,660
6 4,980 3,780 (1,200) 0 600 3,600 18,750.00
Total 25,200 22,187.55

The total production cost savings are $25,200 - $22,187.55 = $3,012.45.

181 | P a g e
Workings

(W1) Overtime costs

Month 3 = 220 std hours / 0.96 = 229.17 hours x $15 per hour = $3,437.55
Month 6 = 1,200 std hours / 0.96 = 1,250 hours s $15 per hour = $18,750

Part (b)

Only 2 factors are needed to be explained from the following:


1. There needs to be close relationships and contractual agreements with the
suppliers which would need to be maintained throughout, since no inventory will
be kept at XY for urgent requests such sales or defective items.
2. Smaller and more frequent deliveries will need to be planned and co-ordinated to
coincide with production needs. The supplier may not have the logistics to
support XY.
3. Higher quality machines will be needed with regular maintenance to avoid delays.
4. There will need to be involvement and training of staff to maintain flexibility of
working hours and skills.
5. Staff need to take responsibility of their quality and so they need to be encouraged
and motivated to do so.

B1 17 LMN (CIMA P2 May 2010)

Part (a)

Performance within an organisation should be focused on assessing what can be


controlled by divisions or individuals and omitting any items which are uncontrollable.
However this is clearly not the case here as there are issues the divisional directors are
responsible for but are not within their control.

Three issues are as follows:

The investment decisions that divisional directors are responsible for maybe
limited due to the $100,000 threshold. It is not clear as to the size of the divisions
and therefore it cannot be ascertained with certainty if divisional directors can
freely make meaningful independent decisions.

Head office costs are apportioned on an arbitrary basis with no consideration for
the activities or costs expended in each of the decisions; furthermore the divisions
do not have control over their own efficiency of resources that they use.

182 | P a g e
The transfer prices are enforced by head office and the divisional directors have
no input into their calculations. It is not clear whether head office is imposing on
divisions to transfer internally or whether they can decide themselves to purchase
internally or externally. There is also no information on the extent of internal
demand for goods and services.

Part (b)

Activity based costing (ABC) will look in more detail about what caused the head office
costs to be incurred and will seek to work out many cost drivers (activities). A cost
driver is any factor that causes a change in the cost activity, so it is important to identify a
causal relationship between the cost driver and the cost. So for example there maybe
head office costs which do not relate to any activities of the divisions but are
administrative expenses to support the organisation as a whole. These should be ignored
as they cannot not be affected by decisions undertaken by the divisions. LMN must only
look at costs which would occur due to the activities of the divisions, this may mean
including other costs and removing others. This would then result in a fairer way to
assess the divisions.

This would allow more efficient management of resources by understanding what drives
costs incurred by divisions. There would be better costing information for planning and
control for example and how different products, customers or distribution channels
consume different resources. Furthermore, more realistic and competitive pricing to cover
overheads and better profitability analysis because of improved accuracy over costs.

B1 18 Production manager (CIMA P2 Nov 2010)

Part (a)

Flexed budget Actual output Revised flexed budget


Output 560 560 560
Direct labour hours 4,480 3,500 1,712 (W1)
Direct labour cost $67,200 $57,750 $25,680

Planning variance = flexed budget revised flexed budget


Planning variance = $67,200 - $25,680 = $41,520 (F)

Labour efficiency variance = (actual hours revised flexed hours) x std cost per hr
Labour efficiency variance = (3,500 -1,712) x $15 = $26,820 (A)

Labour rate variance = actual hours at std rate actual cost


Labour rate variance = (3,500 x $15) - $57,750 = $5,250 (A)

183 | P a g e
Workings

(W1) Direct labour hours

Using the learning curve formula:

Y = aXb

Y = average time for that (X) number of units or the average cost per unit
a = time for the first unit or the cost for the first unit
X = the number of units you want to calculate an average time or cost for
b = the index of learning (log r/log 2)

a = 8 hours, b = -0.1520

Work out the average time for the first 560 units:

Y = 8 x (560 to the power of 0.5146) = 3.057 hours


Total time for 560 units = 3.057 x 560 = 1,712 hours

Part (b)

Target costing is a strategy which seeks to the selling price of a product at the market
price which consumers are willing to pay, being the price that the product should be sold
for in the market. Then deducting a desired level of benefit or profit for the organisation
in order for the manufacture to be commercially viable, and then the product be
manufactured within the value left over thereby becoming the budgeted costs or target
costs.

Market price to achieve desired market share XX


TARGET COST (balancing figure) (XX)
Desired profit XX

Learning curves is an important part of a target costing strategy as it will help in reducing
costs within the business. It is only applicable to those businesses that have a labour
intensive operation where savings can be made through experience and efficiencies. In a
machine intensive operation these savings are limited as machines tend produce at the
same rate. Businesses can achieve target costs once a certain level of activity has been
achieved and so therefore for lessons can be learned and applied to standard costs once it
is known the learning capacity of the labour force.

184 | P a g e
B1 19 CAL (CIMA P2 Nov 2010)

Part (a)

Quality conformance costs are those costs which are spent to try to achieve a standard or
target, such regulations or functional specifications of a product for a customer.

There are two main types of quality conformance costs:

Prevention costs these are spent to try and ensure the product is made to detailed
specifications and regulations. For example training staff, customer surveys,
supplier reviews and investment in machines.

Appraisal costs these are spent to understand how well a process has performed
and corrective action is taken if needed subsequently. For example measuring
equipment, inspections and tests, product quality audits.

Quality non-conformance costs are quality failure costs that are needed to correct
products, as they did not meet expectations or target.

There are two main types of non-conformance costs:

Internal failure costs these are quality failure costs before the products or
services have been transferred to the customer. For example re-inspection of
goods, losses or scrapping of materials and finished goods, additional
administrative costs.

External failure costs - these are quality failure costs before the products or
services have been transferred to the customer. External costs should be avoided
as they expose poor manufacturing abilities to customers. Examples are
administration of customer complaints, administration of customer services,
product liability claims, repairs and replacements, lost goodwill and reputation.

It is clear from the scenario that CAL from a quality perspective provides middle range
quality of solar panels, as they have competitors who sell at a cheaper price but offer an
inferior range of solar panels and others who sell at a higher price but offer a high quality
range of solar panels. CAL is losing out on an increase of 25% in its market share due to
external failure costs of poor assembly skills by staff.

185 | P a g e
Part (b) (i)

Non-conformance cost calculations

Cost of replacing faulty goods for free is that we would have to supply not 20,000 items
but a further 2% in addition to this demand. Therefore, supply 20,000 x 100/98 = 20,408.

408 items represent the free replacement which would cost CAL $45 per unit, therefore a
total of $18,360.

Other non-conformance costs include the lost increase of market share by 25%. This
would result in 20,000 x 25% = 5,000 extra lost sales. This would give a lost contribution
of 5,000 x $15 per unit = $75,000.

Total non-conformance costs = $18,360 + $75,000 = $93,360

Part (b) (ii)

If through an inspection it was found that some solar panels were faulty and not fit for
customer consumption then the lost sale could be avoided as well as the delivery charge.
The item would still be faulty and the only cost incurred would be internal failure costs.
The maximum savings will therefore be $93,360 less the internal costs before delivery
being 408 units x $40 per unit = $16,320, giving a net saving of $77,040.

B1 20 QW (CIMA P2 Nov 2010)

Part (a)

QW currently employs a form of just-in-time (JIT) whereby products should only be


produced if there is an internal or external customer waiting for them.

QW manufacture to specific customer order and so there is no finished goods stockpiled


in the company but they do however hold 1 days stock of raw materials to meet demand
if suppliers are not able to deliver. The system also encourages efficient work of staff as
delays may result in lost sales.

The proposed system aims to procure items to meet constant rates of production. This
would mean holding minimum levels of raw materials as well as finished goods to meet
fluctuations in demand. This is very different to the current system being JIT which aims
to always hold a zero or near zero stock level.

186 | P a g e
This system will ensure demand is met but would lead to inefficient production and
obsolete finished goods. This is because managers would tend to produce more goods
than is necessary to meet demand. This system would also lead to other holding costs
such as damage, deterioration, administration, security and interest costs.

Part (b)

It is essential that a JIT system is underpinned by TQM. This is because under a JIT
system if an item is discovered by the customer as faulty then the company will not be in
a position to replace immediately as it does not hold any stock. It would have to
manufacture the item again and so there will be a delay to customers who may not be
happy about this. The delay may slightly less with QW as they hold 1 days worth of raw
materials but nevertheless there will be waiting time for customers.

In a constant rate production system TQM is not as important because there would be an
inventory of both raw materials and finished goods that can be used to replace the faulty
item. This system has the advantage of customers not having to wait for their replacement
item, however it has the disadvantage of greater costs for holding greater amounts of
inventory and more importantly the company moving away from a focus on quality of
product resulting in the long term loss of customer goodwill and difficulties in
convincing employees that quality is important.

B1 21 Accountancy services (CIMA P2 Nov 2010)

Cost driver rates:

Accounts preparation and advice $580,000 / 18,000 hrs


= $32.22 per hr

Requesting missing information $30,000 / 250 times


= $120 per request

Issuing fee payment reminders $15,000 / 400 times


= $37.50 per reminder

Holding client meetings $60,000 / 250 meetings


= $240 per meeting

Travelling to clients $40,000 / 10,000 miles


= $4 per mile

187 | P a g e
Client A B C
$ $ $
Accounts preparation and advice 32,220 8,055 10,955
Requesting missing information 480 1,200 720
Issuing fee payment reminders 75 300 375
Holding client meetings 960 240 480
Travelling to clients 600 2,400 0
Total costs 34,335 12,195 12,530

Total costs on old basis (W1) 40,280 10,070 13,695

Client fees new basis 41,202 14,634 15,036


Client fees old basis 48,336 12,084 16,434
Difference (7,134) 2,550 (1,398)

Workings

(W1) Total cost on old basis

$725,000 / 18,000 hrs = $40.28 per hr

B1 22 PT (CIMA P2 May 2011)

(i) Growth stage

Unit selling prices

Due to the very short expected life cycle, during the growth stage as competition begins
to enter the market, PT will have to reduce its selling price; in order to stay competitive
as substitutes will be being produced by competitors. These substitutes would have been
created by manufacturers purchasing PTs product and reverse engineering the product.

This approach will aim to sustain demand and contribution from PQs product as long the
selling price is greater than the marginal cost of manufacturing the product. The cash
flow will be used to develop other products in development.

Unit production costs

There will be a continued reduction in unit production costs of this product during its
growth stage but unlikely to see any further reductions beyond this stage. This will be
attributable to economies of scale being enjoyed by the company through mass
production techniques, such as below:

188 | P a g e
Learning curve effects enjoyed by the workforce because the production is labour
intensive, repetitive and most of the staff is retained over the long term.

Technical efficiencies learned by all departments in the manufacture of the


product, especially the production department through improved understanding of
how material and machines can be used more efficiently to maximise production.

Stock control systems will be more efficient if the company employs a JIT
approach to stock management. This will reduce stock holding costs to nil and
material is only used or bought if there is a demand.

Discounts will have been received as the company would buy more and more
bigger batches of the raw material throughout the products life cycle.

Total quality management techniques will be used which means that the product
will be made to a very high standard reducing internal failure costs such as the
inspection and scrap material and also reduction in external failure costs such as
repairs and replacement.

(ii) Maturity stage

Unit selling price

The selling price would be unlikely to be reducing any further because the product would
have now established itself in the market and reached a price that could be maintained
until the end of the life of the product.

Unit production costs

During the maturity phase there is unlikely to be any further significant cuts in unit
variable costs, and costs may begin to increase. This is expected as the demand for this
product will be reduced significantly as it will have fallen out of favour with customers,
and therefore less is being made and as a result fewer materials needed. The lower prices
on raw materials cannot be enjoyed because of the reduced quantities bought by the
company and hence the increase in unit production costs. In addition production costs
will increase due to machine breakdowns and inefficiencies.

B1 23 TQM and JIT (CIMA P1 Pilot Paper 2005)

Note: Only FOUR reasons are required SIX possible reasons are given below.

The TQM philosophy is cultural as well as systems led. It emphasises the importance of
continuous improvement and staff working closer together.

189 | P a g e
Get things right first time e.g. aim for zero defects.
Eliminate waste and inefficiency.
The JIT philosophy requires that products should only be produced if there is an internal
or external customer waiting for them; it aims ideally for zero stock.
The following reasons explain why TQM is important within a JIT environment.
1. Less equipment downtime and major stoppages in production give greater
efficiency of production flow to help meet customer orders quicker.
2. Less reworks, scrap and wastage will ensure faster lead times to meet customer
demand.
3. Avoids sub-standard material from suppliers or sub-standard finished goods
holding up production.
4. More effective teamwork and training of staff will help to improve flexibility to
meet customer orders quicker.
5. Helps staff diagnose and rectify problems quicker.
6. Improved service to customers.
In conclusion within a JIT environment lead time is paramount due to the absence of
stock e.g. a chase demand strategy of meeting customer demand. TQM will help
improve response times by a more efficient and flexible organisation.

B1 24 Standard costing (CIMA P1 May 2006)

Tip: The question requires only three reasons why standard costing may not be
appropriate in a modern business environment. The solution below gives four
possible answers.

Sometimes hard to define an attainable standard especially with the complexity


and diversity of modern manufacturing. Traditionally manufacturers produced a
small range of products using mass production techniques. Nowadays products
have shorter lifecycles and different batches of many different products can be
produced, therefore standard costing is more complex as well as less useful due to
the shorter lifespan of products.
With more automation within operations and less human intervention, standards
are becoming less valuable as information. Automation produces greater
uniformity and consistency of products produced. This places less value on the
use of variances e.g. labour variances, as well as less frequency and materiality of
variances occurring.
Standard costing is an internal not external control measure. Improvement within
organisations needs to consider competition, customers and other global
environmental factors, due to greater intensity of competition. Nowadays external
benchmarking and value analysis is considered more useful to improve an
organisation and its products. Dynamic environments changing frequently do not
support the use of standard costing due to greater flexibility required.

190 | P a g e
Modern manufacturing techniques such as TQM and quality circles, mean as
manufacturers aim for near perfection, the frequency and materiality of variances
should not occur so often. Today focus is more on quality and customer
satisfaction not the minimisation of cost.

B1 25 Marginal v throughput (CIMA P1 May 2006)


Both marginal costing and throughput accounting (TA) concentrate on the maximisation
of contribution; however each approach has a different definition of variable cost and
therefore contribution.
Marginal costing defines variable production cost as material, labour and variable
production overhead and values inventory using this definition. The throughput
accounting approach defines variable cost as material cost only and values inventory
using this definition. TA assumes the only true variable cost when calculating throughput
contribution is the material and component cost only of making a product.
The TA method is perhaps more in line with the modern thought. Traditionally when
marginal costing was invented, staff were paid using piece work schemes and more
flexible hiring and firing of staff existed at this time. Nowadays labour could be assumed
a fixed not variable cost for a manufacturer.
The TA method, according to the theory of constraints, identifies a systems bottleneck
(most limiting factor) that is restricting the flow of throughput contribution and focuses
attention on exploiting or alleviating it. This is similar to the limiting factor approach
using marginal costing.
Both marginal costing and throughput accounting make no attempt to absorb fixed
production overhead into a standard cost unit or the income statement. They both treat
production overhead as a period cost only and charge this entirely to the income
statement for each period.

B1 26 MRPS (CIMA P1 May 2007)

A manufacturing resource planning system is an information system which provides a list


of parts and materials required for the type and number of products entered and also the
planning of labour types or skills or grade required, machine type/hours, priority of other
production and bottlenecks present for the quantity required. It can produce the budgeted
cost for each batch or product to be produced as well as aid effective production planning
and control.

191 | P a g e
In order to effectively support a standard costing system there must be master files set up
containing information about the standard costs, quantities and time needed to
manufacture products. Separate master files for production, materials in stock, lead times
and labour and machine time are needed and then performance can be measured through
variance analysis.

B1 27 JIT (CIMA P1 May 2007)

The JIT philosophy requires that products should only be produced if there is an internal
or external customer waiting for them. Traditionally manufacturers stockpiled and were
not concerned whether or not there was any demand for the products. The main aim was
to have enough stock for each stage of production. This would be able to sustain any
sudden surges in demand but as a result of this there would be an excess of raw materials,
work in progress and finished goods being carried. JIT aims ideally for zero stock e.g.
raw materials delivered immediately at the time they are needed, no build up of work-in-
progress, finished goods only produced if there is a customer waiting for them. It aims to
try to achieve this by doing the following:

1. Closer relationships with suppliers required


2. Smaller more frequent deliveries need to be managed in order to produce
3. Cell manufacturing
4. Reduction in set up time (eliminating non value added activities)
5. Higher quality machines perhaps multi-purpose with regular maintenance
6. Involvement and training of staff to maintain flexibility
7. Elimination of defects

B1 28 Key features of TQM (CIMA P1 May 2008)

There are several key features that are present in an organisation that uses TQM and the
list below is not exhaustive. Any three of these would be adequate in answering the
question:

The company will establish senior management commitment and support for
TQM.

Processes are put in place to facilitate the adoption of standards and procedures
(such as ISO 9000 certification) to monitor and control quality. This is often
achieved by benchmarking of competition and investment in employees.

Production to achieve zero defects and eliminate waste. This would be achieved
by ensuring that adequate time is spent in manufacturing the products correctly
the first time.

192 | P a g e
Employees are to look for continuous improvements in all the processes, resulting
in improved productivity, efficiency and less idle time from work processes
undertaken.

There is greater investment in quality assurance such as training of staff to ensure


high standards, suppliers guarantee high quality, assurance standards and
procedures adopted (feed forward control).

The company manufactures products which are more customer focussed achieved
through the use of added value activities such as continuous feedback from
customers and the reduction of customer complaints.

Stock being held by the company will be zero and there will be no work in
progress as products are being made only if there is an external or internal
demand.

193 | P a g e
Solutions Section B

Part B Cost Planning and Analysis for Competitive Advantage

B2 -1 The Q organisation (CIMA P2 May 2005)

Part (a)

Stages in the life of a product will include:

Introduction e.g. customers unaware, high initial research and development and
marketing cost to create and market the new product

Growth e.g. awareness increases and industry size increases as more and more customers
make purchases of the product, new features to improve or differentiate the product as
more competition enters

Maturity e.g. product development, modifications and improvements to extend the


products life. Mature and a more saturated market as industry growth slows down and
demand reaches a limit, greater production efficiency/economies of scale obtained by this
stage.

Decline e.g. obsolesce of product, usually replaced by a better or newer versions or other
broader substitutes

Sales

Time
Introduction Decline
High unit costs. Maturity Overcapacity
Extensive sales Growth Profits in industry.
promotion Competitors Senility
good. Some players
enter market. Product possibly
High sales leave market
Unit costs fall retained to
promotion enhance product
portfolio

194 | P a g e
Product life spans can be from 1 week to over 100 years.

During an introduction of a new product, the decision is either to charge a very low price
in order to capture a larger market share quicker (market penetration) or to initially
charge a high price, achieve low volume but earn a larger profit per unit sold (market
skimming).

Market skimming

It may extend the life of the product life-cycle. Q sells high technology products and is
also a respected manufactured worldwide as a result there maybe an expectation by
customers to pay a premium for these products.

Q could also use this approach as it is good for innovative high quality products where
little competition exists initially. Therefore a premium can be charged without reducing
demand for it too much.

There is always a choice of lowering the price later on when competition comes it to the
market with cheaper substitutes.

Market penetration

This approach will aim to achieve a wide customer base as quickly as possible by setting
the price very low. It means that because if its affordability a lot of people will be able to
purchase it, as a result of the first 2 stages are quickly achieved in the product life cycle
and also achieve economies of scale far quicker.

It will also be more difficult for other companies to enter the market as they would have
to compete on this low price and may struggle to make a profit.

Recommendation

It would be advisable for Q to start with a market skimming policy because the products
they are selling are high technology items in high demand, and also manufactured from a
well respected company. Once competition has entered their market with close substitutes
then Q should adopt a market penetration policy to maintain its market position.

Part (b) (i)

We know that at 10,000 units the VC affected by the learning curve rate is 30, and at
40,000 units the VC affected by the learning curve rate is 22.71. We know that to get
from 10,000 units to 40,000 units we have to apply the learning curve rate twice.

195 | P a g e
Therefore if the learning rate is r then:

22.71 = 30 x r x r
22.71 = 30r
22.71 / 30 = r
0.757 = r
0.757 = r
0.87 = r

Therefore the learning rate is 87%.

Part (b) (ii)

Work out the variable costs which are affected by the learning curve rate:

Number of Total VC per unit VC affected by VC not affected by


cumulative units learning curve learning curve
10,000 60.00 30.00 30.00
20,000 56.10 26.10 (W1) 30.00
30,000 54.06 24.06 (W1) 30.00
40,000 52.71 22.71 30.00

W1 We can use the formula to work these out

Y = aXb

Where: a = 30, b = log 0.87 / log 2 = -0.201

Therefore the average variable cost for 2 batches or 20,000 units:

Y = 30 (2) (to the power of -0.201)


Y = 26.10

The same principle for 3 batches or 30,000 units:

Y = 30 (3) (to the power of -0.201)


Y = 24.06

Now work out contribution by comparing selling price to variable costs for every demand
level and then working out the optimum price.

196 | P a g e
Demand units Selling price VC per unit Contribution per Total
per unit unit Contribution
10,000 100 60.00 40.00 400,000
20,000 80 56.10 23.90 478,000
30,000 69 54.06 14.94 448,200
40,000 62 52.71 9.29 371,600

The price that optimises contribution is 80 in the initial launch phase of 20,000 units.

Part (b) (iii)

We need to work out target contribution from the given target profit of 30,000 and then
we can work out how many units we need to sell to achieve the target contribution.

If target profit is 30,000 and fixed costs are given as 15,000 then target contribution
must be 45,000.

The initial launch phase is 20,000 units, and so we need to work out contribution per unit
after this and then we can divide this by target contribution to know how many units to
sell.

The problem is that we have a reducing variable cost due to the learning curve for every
batch we produce and so therefore we have to work out the average unit cost for every
batch.

Ave unit cost between 20,000 and 30,000 units:

(30,000 x 54.06) (20,000 x 56.10) = 49.98


10,000

Ave unit cost between 30,000 and 40,000 units:

(40,000 x 52.71) (30,000 x 54.06) = 48.66


10,000

Contribution per unit between 20,000 and 30,000 = 57 - 49.98 = 7.02


No. of units that need to be sold to reach target contribution = 45,000 / 7.02 = 6,410

Contribution per unit between 30,000 and 40,000 = 57 - 48.66 = 8.34


No. of units that need to be sold to reach target contribution = 45,000 / 8.34 = 5,396

197 | P a g e
B2 -2 F plc (CIMA P1 May 2005)

Part (a) (i)

Current system (absorption costing)

Absorption of overhead
880,000 8m sales value of orders = 11%

11% of overhead will be charged every 1 of sales

Order A 1,200 x 11% = 132.00


Order B 900 x 11% = 99.00

Part (a) (ii)

Activity based approach

Cost drivers

Invoice processing

(25% x 280,000) 8000 order/invoices = 8.75 per invoice


(75% x 280,000) 28000 invoice lines = 7.50 per invoice line

Packing

Small package (given) 25.00 per small package


Large package (given) 32.00 per large package

Delivery

(40,000 1000 deliveries) 6 small packages = 3.33 per small package


(40,000 1000 deliveries) 12 small packages = 6.67 per large package

(180,000 - 40,000) 350000 miles = 0.40 per mile

Other overheads

200,000 8000 orders = 25.00 per order

198 | P a g e
Order A B

Invoice processing
8.75 per invoice 8.75 8.75
7.50 per invoice line 7.50 x 2 = 15.00 7.50 x 8 = 60.00

Packing small package 25.00 large package 32.00

Delivery small package 3.33 large package 6.67


0.40 x 8 miles 3.20 0.40 x 40 miles 16.00

Other overheads
25 per order 25.00 25.00

80.28 148.42

Part (b)

Tip: A good report format will make your report be well presented and provide good
structure for your answer. Part (b)(i) in short requires the advantages and
disadvantages of ABC in comparison to absorption costing, so if you know what these
are it will be the backbone to answering this part to the question. A good approach
also would be to think of good examples from the calculations in part (a) to provide
value within your report e.g. why the proposed system has led to a better or worse
understanding of how costs are driven by the different resources consumed, such as
administration or transport.

REPORT
To: Management of F Plc
From: Management Accountant
Date: DD/MM/YY

1.0 Introduction

The purpose of this report is to assess the strengths and weaknesses of the proposed
activity approach for F Plc and recommend actions that the management might consider
in the light of the new data produced using the activity based approach.

199 | P a g e
2.0 Strengths of the proposed activity based approach

The current approach using absorption costing, took the total budgeted selling a
distribution cost for the period and divided this by the budgeted list price of drugs
supplied, in order to find a single overhead absorption rate. The new activity based
costing approach looked in more detail about what causes the selling a distribution cost
for the period to be driven. It worked out many cost drivers rather than just sales value.
It therefore obtained a more accurate way of looking at how overhead is driven and
should therefore give a more accurate picture when costing and pricing for different
orders of pharmaceuticals.

For example order A using absorption costing would charge 132 of overhead, where as
with the proposed system of ABC would charge only 80.28. This charge is lower due to
recognising that order A consumes less resources of F Plc to deliver it.

It required less invoice processing than order B e.g. only two lines on the invoice,
meaning less resource of staff or equipment used to administrate it.
It was a small package and therefore costs less to transport per package, due to a
lorry having the ability to carry more packages per load. It also required less
distance to be transported and this was reflected also in the cost of the order.

The strengths of the proposed activity based approach

More efficient management of resources by understanding what drives selling and


distribution overhead incurred. For example management by ensuring all lorries
carry full loads, can help reduce the frequency of deliveries to help minimise
transport cost.
Better costing information for planning, control or decision making. An
outsourcer could supply an activity externally and this can be benchmarked to
internal costs for a decision to be made. A decision to accept or reject a price
offered by a customer for a long-term contract, can be compared to the cost of
supplying it more realistically.
More realistic pricing to customers in future, to cover overhead being incurred by
them.
Better profitability analysis of different customers and their orders.

3.0 Weaknesses of the proposed activity based approach

To implement a new system even though it could be useful, can be time


consuming and a more expensive system to run and maintain, due to the greater
complexity of how overhead is now allocated to orders. Investment will be
required for bespoke information systems, does this additional cost justify the
benefit?

200 | P a g e
It still assumes fixed overhead is driven by volume as does absorption costing.
The nature of fixed overhead is that it is fixed it is assumed it does not vary
significantly with an activity level of any kind. As an example it would be naive
perhaps for management to believe that less administration of an order will mean
less overhead incurred, given that most of this cost is staff overhead e.g. fixed
salaries regardless of whether staff are processing orders or idle.

4.0 Recommendations

The current approach although relatively simple, does not truly reflect the way an order
consumes the resources and therefore overhead of F Plc. Charging 11% of the list price
of an order does little in the way of recognising that the larger a package, the more
invoice processing it requires and the longer the distance it needs to be carried, then the
higher the cost of supplying it. It also requires the full 8 million list price of
pharmaceuticals to be supplied in order to recover the 880,000 overhead fully.

My recommendations would therefore be as follows.

1. If the new system is not costly to introduce then it perhaps should be, but before
this decision is made it is imperative that research is done fully in terms of price
comparisons between the current and proposed system of charging customers.
Just because it reflects better consumption of resources within the price
ultimately charged, does not mean it should be introduced especially if the new
system will create prices well above prices charged for similar orders by the
competition. Customers may perceive the idea as far too complex.

2. If the system is considered too costly or complex by the management or


customers, then perhaps the following alternative should be considered.

a) Charging a standard additional surcharge to a customer if delivery is


outside a certain radius e.g. within 10 miles, 10 to 25 miles and above 25
miles.
b) Charging a standard additional surcharge to a customer for multiple orders
or larger packages handled.

If this is this case the introduction of a relatively less complex and costly system can be
maintained. At least customers will have a better idea when it comes to how much they
will be charged or quoted. However again research needs to be done, it is important that
standard surcharges when levied, do not put customers off, especially if competition does
not do the same.

201 | P a g e
I would not recommend using the proposed ABC system as a way of pricing orders for
customers, as I consider it too complex, but I would recommend it perhaps internally
within F Plc when it comes to understanding how to reduce consumption of resources and
improve efficiency. I would also recommend it for customer profitability analysis when
it comes to making better management decisions about whether or not to supply a
customer e.g. is the customer losing money for F Plc? Such a system can be maintained
through a simple spreadsheet when doing so and therefore will not require significant
investment.

Signed Management Accountant

B2 3 KL (CIMA P2 Nov 2006)

Part (a)

Total or full cost plus pricing


This approach takes the variable costs involved in manufacturing the product and adding
an apportioned amount of fixed overhead using an absorption costing or activity based
costing system. This is known as the total or full cost. Then by adding a % mark up
which will represent the profit earned from selling the unit this will give us the total or
full cost plus price. This is the selling price of the unit.

Advantages
Ensures fixed overheads are recovered in the long-term as it is included in the
selling price.
Suitable if you already have an absorption costing system in place but without
such a system it would be difficult to know how much to absorb of the fixed
overheads in each of the different products.
Good for long-term profitability analysis as fixed overheads would be aimed to be
recovered fully in all products sold.
Adequate profits are made without the need to seek out a selling price that would
maximise profits as a profit mark up is included in the price.
Easy and quick to produce prices for products and therefore can be delegated to
junior staff.

Disadvantages
The selling price selected may not suit the market price and demand conditions as
it ignores competition.
The method of sharing fixed overhead over the product range is subjective as a
fixed overhead cost driver must be selected to apportion the costs. The driver may
not be representative of how fixed overheads were actually spent.
Different organisations will use different overhead absorption methods for
example an activity based costing approach versus a traditional absorption costing
approach.

202 | P a g e
Apportionment of fixed overhead tends to be irrelevant in decision-making this is
because they are historical costs and do not effect the net benefit to the company.
The company should look at marginal revenue and marginal costs only as these
will change as a result of our decision.

Marginal cost plus pricing

This approach is more likely to be used with traders within the retail sector or when a
company is at full capacity due to a limiting factor (relevant costing or minimum
pricing). Marginal cost plus pricing recognises that fixed cost are not relevant to decision
making and therefore ignores it in its pricing. Therefore to achieve the selling price under
this approach we add the marginal or variable costs plus a % mark up together. The MDs
concern over whether sales may be lost if prices are set too high is important and using a
marginal cost plus pricing approach may mean that prices are set lower and are more
competitive whilst still earning some contribution towards fixed overheads.

Advantages

There is a far better understanding of cost as we are only looking at those costs
that change as a result of our decision that being marginal costs only.
It is useful when we only have information regarding variable costs only.
Good for short-term decision-making as it ignores historical fixed costs.

Disadvantages

Ignores fixed overhead in the long-term and therefore this would result in long
term losses. Fixed overheads need to be recovered over the long term; however
the mark up may be large enough to include this and profit.
The selling price selected may not suit the market price and demand conditions as
it ignores competition.
It maybe all you can use due to the inability to apportion fixed overhead.

Part (b)

Product W X Y
Labour hours per unit $40 / $10 = 4 $30 / $10 = 3 $50 / $10 = 5
Budgeted production 15,000 24,000 20,000
Total labour hours 15,000 x 4 = 60,000 24,000 x 3 = 72,000 20,000 x 5 = 100,000

Total labour hours = 60,000 + 72,000 + 100,000 = 232,000

Overhead absorption rate = $1,044,000 / 232,000 = $4.50 per labour hour

203 | P a g e
W X Y
Material $35 $45 $30
Labour $40 $30 $50
Overhead $4.50 x 4 = $18 $4.50 x 3 = $13.50 $4.50 x 5 = $22.50
Total cost per unit $93 $88.50 $102.50

Part (c)

W X Y
Production levels 15,000 24,000 20,000
Number of batches 15,000/500 = 30 24,000/400 = 60 20,000/1,000 = 20
Supplier orders 30 x 4 = 120 60 x 3 = 180 20 x 5 = 100
Machine hours 15,000 x 5 = 75,000 24,000 x 8 = 192,000 20,000 x 7 = 140,000

Overhead absorption rate:

Material ordering = $220,000 / (120 +180 + 100) = $550 per order


Machine set up = $100,000 / (30 + 60 + 20) = $909 per batch
Machine running = $400,000 / (75,000 + 192,000 + 140,000) = $0.98 per hour
General facility = $324,000 / (75,000 + 192,000 + 140,000) = $0.80 per hour

Cost per unit W X Y


Material ordering ($550 x 120) / 15,000 ($550 x 180) / 24,000 ($550 x 100) / 20,000
= $4.40 = $4.13 = $2.75
Machine set ups ($909 x 30) / 15,000 ($909 x 60) / 24,000 ($909 x 20) / 20,000
= $1.82 = $2.27 = $0.91
Machine running ($0.98 x 75,000) / 15,000 ($0.98 x 192,000) / 24,000 ($0.98 x 140,000) / 20,000
= $4.90 = $7.84 = $6.86
General facility ($0.80 x 75,000) / 15,000 ($0.80 x 192,000) / 24,000 ($0.80 x 140,000) / 20,000
= $4 = $6.40 = $5.60
Direct material $35 $45 $30
Direct labour $40 $30 $50
Total cost per unit $90.12 $95.64 $96.12

Part (d)

W X Y
$ $ $
Current system 93.00 88.50 102.50
ABC 90.12 95.64 96.12
Difference 0.12 (7.14) 6.38

It seems that for product W using Activity based costing (ABC) or the current system
produces very similar figures for unit cost, however for product Y costs are less under
ABC when compared to the current system, but this is not the case for product X where it
seems that there are more costs allocated under ABC than the current system.

204 | P a g e
ABC looks in more detail about what causes fixed overhead to be incurred and works out
many cost drivers rather than just labour or machine hours or products produced, all
driving overheads to be incurred. It is used in order to obtain a more accurate way of
looking at how fixed overhead are driven and should give a more accurate picture when
costing products, budgeting or valuing stock.

ABC can improve profitability by:

Managing resources more efficiently through the understanding of what drives


fixed overheads.

Thereby better costing information for planning and control e.g. how different
products consume different resources or the production of flexed budgets based
upon activity based budgeting (ABB).

More realistic and competitive pricing to cover overheads.

Better profitability analysis of product range.

In the case of KL, profitability could be improved as follows:

Reducing the number of set ups for product X would reduce set up costs. Currently under
ABC $2.27 is allocated to product X for set up costs which is the most out of the product
range. Product X has the highest annual production and it would make sense if these
batches were larger and thus reduction in set up costs.

Product Y may have costs allocated to it under the current system which are not driven by
the activities in this product. This is seen by the lower cost per unit under ABC. Using the
fairer costs allocation under ABC this would mean that a lower selling price could be
charged and thereby increasing sales but still maintaining the same unit profitability.

General facility costs are said to be driven by number of machine hours. It is not said in
the question what the costs are but they may include maintenance to the building or
warehouse, and if this was the case machine hours does seem like an arbitrary driver. It
may be more sensible to leave such costs out of the unit cost calculation when looking at
profitability as it would only serve to distort the overall cost and reduce the
meaningfulness of the figures.

205 | P a g e
B2 4 Retail outlet (CIMA P2 Nov 2007)

Part (a)

Option 1

Sales lost
Weeks 1 to 4 None
Weeks 5 to 8 30%
Weeks 9 to 12 20%
Weeks 13 to 16 10%

Weeks 1 to 12 there would be a 10% discount on the price of basic and canned foods.

$
Basic foods 200
Newspapers 300
Frozen foods 950
Canned foods 1,200
Lost contribution for 1 week 2,650

Relevant costs for option 1


$
Weeks 1 to 4 lost contribution (4 x 2,650) 10,600
Week 1 to 4 freezers not used (4 x 100) (400)
Redecoration costs 2,500
Weeks 5 to 8 lost contribution (30% x 2,650 x 4) 3,180
Weeks 9 to 12 lost contribution (20% x 2,650 x 4) 2,120
Weeks 13 to 16 lost contribution (10% x 2,650 x 4) 1,060
Discount on selling price weeks 5 to 8 896
(10% x (800 + 2,400) x 70%) x 4
Discount on selling price weeks 9 to 12 1,024
(10% x (800 + 2,400) x 80%) x 4
Discount on selling price weeks 13 to 16 1,152
(10% x (800 + 2,400) x 90%) x 4
Total 22,132

206 | P a g e
Option 2

Relevant costs for option 2


$
Closure for a week lost contribution 1,275
($2,650 - $100) x 0.5
Redecoration costs 3,500
Product movement costs 1,000
Lost contribution on frozen foods (W1) 2,550
Lost contribution on canned foods (W1) 6,840
Total 15,165

Conclusion: Option 2 has the lower losses.

Workings

(W1) Lost contribution

For 6 weeks the floor space will be restricted to 40% of original size
= 40% x 400m
= 160m

No reduction in floor space for Basic Foods and Newspapers and Magazines
therefore 60 m is left for Frozen Foods and Canned Foods.

We should do those products that give us the most profitability that being contribution
per m between Frozen Foods and Canned Foods.

Frozen foods contribution per m = 950 / 100 = $9.5 per m


Canned Foods contribution per m = 1,200 / 100 = $6 per m

We should do Frozen Foods first and then Canned Foods if we have any space left.
We can only do Frozen foods in complete freezer units.

There are 4 freezer units, therefore each unit uses 4 / 100m = 25m . We can only do 2
freezer units so therefore 50m is given to Frozen Foods and the rest being 10m is
given to Canned Foods.

Therefore lost contribution:

Frozen Foods = ($950 - $100) x 50m /100m x 6 weeks = $2,550


Canned Foods = $1,200 x 190m/200m x 6 weeks = $6,840

207 | P a g e
Part (b)

Traditional absorption costing takes the total budgeted fixed overhead for a period and
divides by a budgeted (or normal) activity level in order to find the overhead absorption
rate.

Activity based costing (ABC) looks in more detail about what causes fixed overhead to
be incurred and works out many cost drivers rather than just labour or machine hours or
products produced, all driving overhead to be incurred. It is used in order to obtain a
more accurate way of looking at how fixed overhead is driven and should give a more
accurate picture when costing products, budgeting or valuing stock.

ABC is a method which lends itself to a retail company environment where there are a
wide range of products being bought and sold which require different conditions in
storage and display. For example frozen foods and chilled foods require refrigeration
costs where as canned foods do not require this only storage in normal room temperature
conditions.

Main steps in ABC

Group types of fixed overhead together, if they are driven by the same type of
activity or driver. These are known as cost pools. For example quality control
supervisor and quality control rent and overhead, grouped together as both would
be driven perhaps by the number of inspections forecast for the period.

Calculate from these cost pools a fixed overhead cost per driver, for example
similar to an overhead absorption rate when compared with traditional absorption
costing.

Absorb fixed overhead by using many cost drivers rather than just one or two
cost drivers or overhead absorption rates as with traditional absorption costing
this would give management a better idea as to how different products produced
consume resources and therefore overhead.

Activity Cost driver


Material procurement No. of purchase orders
Material handling No. of movements
Quality control No. of inspections

Advantages of ABC

ABC would allow efficient management of resources by understanding what drives fixed
overhead incurred.

208 | P a g e
There would be better costing information for planning and control for example how
different products consume different resources or the production of flexed budgets based
upon activity based budgeting (ABB).

When an organisation is formulating their long term strategic decisions, such as product
pricing, mix of products, discontinuance, launch or promotion of existing products and
the launch of new products, ABC allows for realistic pricing to cover overheads and
therefore superior profitability analysis on product ranges.

ABCs strength lies in the fact it allows accuracy over costs and drivers for products and
as a result a sensible pricing strategy is achieved. It more specifically gives a good long
term understanding of the variable costs being very relevant for decision making.

However ABC information must be put into perspective as these are historic costs and
cannot be used alone to predict future costs. They should be used as a starting point and
other internal and external information should be used to determine future costs. All costs
are variable in the long term and subject to change.

209 | P a g e
Solutions Section A

Part C Budgeting and Management Control

C1 -1 Solicitors firm (CIMA P2 May 2010)

Part (a)

In the present budgeting system the senior partner estimates demand for the year while
the divisional manager creates the cost budgets to support demand. The cost budgets are
submitted for review by the senior partner but any amendments to the budgets are
included without consultation with the divisional partners. This illustrates that the senior
partners do not really allow divisional partners to participate in the budgeting process as
they are not consulted on budget revisions.

This can de-motivate staff leading to a decline in their productivity and efficiency as they
feel their expert opinions are not important and are simply ignored. The divisional
partners would feel that they have wasted their time in creating the budgets and would
begin to feel that they are not part of the process to improve the organisation. There is
great danger of a divide being created between the senior partner and the divisional
partners.

The current approach may also see divisional partners start to find excuses not meeting
targets just to prove to the senior partner that their budgets were correct and the changes
imposed by the senior partner were wrong. Sub-optimisation or dysfunctional behaviour
may occur for example the personal objectives of the divisional partner are not aligned
with the business objectives.

Part (b)

Only 2 are needed to be explained from the following:

Training days per staff measures the level of investment that the firm has
committed to improving staff skills to support the needs of clients.
Staff turnover measures the number of staff leaving the firm indicating that staff
are perhaps dissatisfied with the firms approach to supporting the needs of staff.
Training costs per staff measures the amount of money the firm have set aside
for staff training purposes.
Time taken between client first enquiry and first meeting measures efficiency
and flexibility of the firm to clients needs.

210 | P a g e
C1 2 DW (CIMA P2 Nov 2010)

Part (a)

An annual budgeting system is the process of using existing budgets as a guide to prepare
the budget for the next 12 months which normally coincide with financial year end of the
company.

Rolling or continuous budgeting is when the budget is updated on a regular and frequent
basis. The method is to add a further period immediately to the budget when an earlier
period has expired, for example if Jan to Mar 2005 is the first three months of the yearly
forecast, once this has expired then Jan to Mar 2006 will be created and added.

Two of the key differences between these two systems are:

Rolling budgets always provide a budget for the full period as they get updated,
unlike annual budgets which dont get updated as the periods expire. This enables
management and so enables better long term planning.

Rolling budgets are good for adaptive planning, for example there is a greater
chance that the budgets will be regularly updated to take account of changes
within the environment the organisation is operating within if used.

Part (b)

The new depot manager has two concerns that he raises about the current system:

Budgets are out of date

Annual budgets are set on annual basis and are not revisited on a regular basis. This
means that it is not possible to see if the budget is being met or even if current economic
circumstances have made the budget out of date and not applicable.

Rolling budgets would allow regular comparison to the actual outcomes more frequent
update of forecasts where necessary due to unanticipated changes in the economy. This
would keep the budgets more accurate, reliable and meaningful.

Operational and strategic decisions cannot be taken

Rolling budgets always provide a budget for the full period as they get updated and
approved my management, unlike annual budgets which dont get updated as the periods
expire. This enables depot mangers to take decisions for the early part of next year.

211 | P a g e
Annual budgets only get updated once a year and proves difficult for depot mangers to
make decisions until the budget has been approved by management which maybe close to
the year end restricting decisions being taken by depot mangers in the early part of next
year.

Operational decisions and control must however be dealt with carefully as these entail the
day to day actions of the company and any differences in actual performance must be
noted to ensure that improvements can be made. Rolling budgets should not eliminate
these variances by revising them as it would mean that information about inefficiencies
cannot be isolated and improved.

Strategic decisions and control would benefit from rolling budgets as it help to ensure
that failing strategies are abandoned in favour of successful strategies. Rolling budgets
would allow frequent revisits to the strategy to ensure that it is being met and if not then
the original strategy being revised.

C1 3 JYT (CIMA P2 May 2011)

Target costing is a strategy which seeks the selling price of a product which consumers
are willing to pay, being the price that the product should be sold for in given market
forces. Then deducting a desired level of benefit or profit for the organisation in order for
the manufacture to be commercially viable, and then the product be manufactured within
the value left over thereby becoming the budgeted costs or target costs.

Market price to achieve desired market share XX


TARGET COST (balancing figure) (XX)
Desired profit XX

The target cost is normally achieved over the long term using learning and experience
curve efficiencies and may take several years and so in the meantime the desired profit is
squeezed to compensate.
Target costing is not simply a cost reduction exercise but a quality improvement strategy
over the long term. It sacrifices short term profitability for long term profitability and
combines the use of JIT, TQM, cost reduction, value analysis and benchmarking to
achieve the target cost.
Kaizen costing is the process of continuous improvement through small incremental steps
rather than transformational changes. It also believes strongly in empowerment of
employees to enable them to improve operations. Kaizen costing focuses on reducing
variable costs of future periods below that of prior periods.

212 | P a g e
Human resources are your most valued asset and they should be involved in the
search for perfection. They have superior knowledge as they are in the operation.
This is the opposite of traditional systems where the managers develop standards.
Incremental or gradual improvement.
Perfection should be sought all the time and so there is always room for
improvement.
Cost reduction targets more frequent rather than traditional annual standard costs.
Variance analysis used to help with kaizen costing.
Investigations carried out if targets not met even if improvements have been
made.

Unlike kaizen costing, target costing doesnt focus on gradual cost reduction but starting
from scratch and re-designing methods and processes to achieve large changes. Kaizen
costing relies on all employees taking part in the process because it focuses on making
small improvements by everyone.

Target costing is a strategy implemented before the launch or production of a product


unlike kaizen costing which is employed when production has commenced.

C1 4 DVD (CIMA P2 May 2011)

Part (a) (i)

The sales mix variance shows the change in the different product lines being sold and
the impact it has on profit or contribution.

Actual Actual sales Difference Profit Variance


sales at budget
quantity mix (W1)
DVD 3,000 2,800 200 (F) $25 $5,000 (F)
Blu-ray 1,200 1,400 200 (A) $95 $19,000 (A)
4,200 4,200 $14,000 (A)

Total sales mix profit variance = $14,000 (A)

Workings

W1 Actual sales at budget mix

Revised demand for Blu-ray players of 1,500 units should be used in the budget instead
of 1,000 units this is because of an unexpected growth in the total market for Blu-ray
players.

DVD = 3,000 / 4,500 x 4,200 = 2,800


Blu-ray = 1,500 / 4,500 x 4,200 = 1,400

213 | P a g e
Part (a) (ii)
Units
Did sell (actual quantity sold) X
Should sell (original budget quantity sold) (X)
X
x Standard Profit per unit
Sales volume (profit) variance X

The sales volume (profit) variance measures the difference between the original and
flexed budgeted profit. It measures the impact on profit, when actual sale of units is more
or less than the original budgeted sale of units. This method of calculation would be
applied when absorption costing is used by the organisation.

DVD sales volume variance = (3,000 3,000) x $25 = $0


Blu-ray sales volume variance = (1,200 1,500) x $95 = $28,500 (A)
Total sales volume variance = $28,500 (A)

Part (b)

The planning variance is beyond the operational control of management and staff for
example market size growth of Blu-ray player.

The operational variance is normally within the control of management and now more
realistic as a yardstick because calculations would include any revisions to standard, in
the case of the Blu-ray players, the revised budgeted demand due to changes in the total
market share.

An example of an operational variance is the changes in the selling prices for the DVD
and Blu-ray players resulting in variances which are controllable by management.

Planning variances will help highlight variances between those which are controllable
and those which are uncontrollable. For example the sales managers email boasts a
favourable $19,000 variance however it is not strictly entirely caused by his influences of
sales, some of it is due to the increase in total market share of Blu-ray players which is
not in the control of the sales manager. The budget for Blu-ray players should have been
1,500 units and therefore resulting in an adverse variance.

Planning variances help motivate managers and staff, for example avoids staff being
blamed for faulty planning and gives a fairer reflection of any operational variances
calculated when assessing any operational efficiencies or inefficiencies. Management and
staff would be appraised more fairly for any favourable or adverse deviations that are
within their control.

Planning variances make use of realistic standards in order to measure performance gives
better management information for control purposes.

214 | P a g e
C1 5 SFG (CIMA P2 May 2011)

Part (a)

Non-financial performance measures recognise that the constant drive to increase


profitability can ultimately be self-defeating and that it is imperative that organisations do
not put the needs of shareholders above all else. The primary goal of most profit seeking
enterprises is to increase shareholder value through increasing profit; however non-
financial performance measures are equally as important as they ultimately drive
profitability and therefore shareholder value. For example level of customer satisfaction,
innovation, quality and morale of the work force, all indicate how well the company is
doing things that can lead to future profits.

Long-term benefits of using non-financial performance measures will be improved


profitability when these performance measures are combined with financial performance
measures and are monitored and used for control purposes.

Part (b)

Exam tip: Please note that the question only asks for two reasons however we have
provided a selection of possible reasons. Any two would suffice.

Customer satisfaction surveys measures the number of complaints or


recommendations at the hotel, also giving a rating for service received from the
hotel staff and mangers. Cutting back on customer service costs may save money
in the short term but will impact the long-term reputation of the hotel and reduced
repeat business.
Training days per staff measures the level of investment that the hotel has
committed to improving staff technical and interpersonal skills.
Staff turnover measures the number of staff leaving the hotel indicating that
staff are perhaps dissatisfied with the hotel mangers approach to supporting the
needs of staff.
The number of new hotel deals for accommodation and events The more of
these options are available created my by the managers the greater chance of
increasing sales and repeat business. It may cost money to set up these deals and
some money and in giving discounts or free taster days but in the long-term it
would increase sales.

215 | P a g e
Part (c)

An uncontrollable cost does not mean a manager being assessed should ignore it
altogether. As an example if managers recognise there is an interest charged by head
office based upon the capital employed used within a division, then to hold them more
accountable could help improve efficiency by the minimisation of capital employed. In
the case of head office charges, lack of accountability could encourage over consumption
of these resources provided centrally. However it would be considered fairer if a manager
were not assessed on costs which are not within their own control. This is likely to
improve motivation and morale.

The way to include non-controllable costs when assessing the performance of the manger
on a fair basis is to put them into a separate section with in the report; however it is not
always clear when determining controllable and non-controllable costs. Political
arguments may ensue which are more subjective than objective when determining
controllability.

C1 6 Feedback and feedforward (CIMA P1 Pilot Paper 2005)

An example of a feedback control system is a budgetary control system. This would


gather information on past performance from the output of the system e.g. actual
financial performance, and compare this to a predetermined standard or plan (budget)
using any deviations e.g. variances, as a basis of improving future performance through
control action taken.
A feedback loop is where the output of a system is measured and fed back as input into
the system in order to obtain a desired effect, often done intentionally, in order to control
the behaviour of a system. In a double feedback loop, corrective action is not
automatically taken. The output of the system is measured, however environmental
factors will also be considered, along with internal feedback before any control action is
taken. In a single feedback loop, the output is automatically compared to a pre-
determined standard; any exceptions and control action will be automatically taken.
Feedback contrasted to feed-forward control is like closing the door after the horse has
already bolted, in other words there is little you can do about it now, except try and
rectify the situation to avoid it happening again. Feed-forward control is more prevention
than appraisal, controlling a system by making adjustments now to the system in advance
before any exceptions occur. It does this by trying to predict what will happen in the
future.
Feedback can be transformed into feed-forward control by being more proactive and
predictive as to what will happen in the future, rather than being reactive or backward
looking by historical reflection on the past.

216 | P a g e
C1 7 Profit centre managers (CIMA P1 Pilot Paper 2005)

Tip: The behavioural aspects of budgeting is concerned with how budgets or


standards affect people within an organisation. Poor performance could be
attributable to the method of implementing budgets e.g. ignoring the human side of
participation or the introduction of a standard centrally that is unrealistic or
unobtainable can de-motivate staff.

A profit centre is when a manager will be accountable for creating sales revenue and
controlling cost. The division or centre is assessed on profit earned.
Advantages of participation
Greater motivation for the manager and because of this targets are more likely
to be met. It will also be more interesting and more involving for the manager
increasing job satisfaction.
Participation means that targets are more likely to be accepted by the manager
e.g. less conflict due to setting targets themselves.
The manager maybe more up to date in terms of the market environment and
therefore the targets they set could be more realistic than if imposed centrally.
Disadvantages of participation
If managers are asked to participate in setting their own budgets there is the
possibility they will include slack or padding within the budget e.g. have less
sales units forecast than could realistically be achieved. This is the inevitable
downside of human behaviour e.g. underestimate to avoid blame if they do under
achieve.
Participation could create a slower process to formulate the budget because more
consultation will be required.
The manager could be inexperienced financially. Training would be required in
order for them to participate effectively.

C1 8 Balance scorecard (CIMA P1 May 2005)

Tip: Below are many possible answers for each perspective, the question states that
you are to provide only one performance measure for each perspective and give a
reason to support each measure.

A new approach to strategic management was developed in the early 1990's by Drs.
Robert Kaplan (Harvard Business School) and David Norton. The balanced scorecard
suggests that we view the organisation from four perspectives.

217 | P a g e
Customer perspective e.g. what must we do right for our customers and what do
they value?
Internal perspective e.g. what must we excel at or improve internally to satisfy
shareholders and customers?
Innovation and learning perspective e.g. how can we innovate and improve value?
Financial perspective e.g. how do we satisfy shareholders and create value for
them?

Customer perspective

Percentage of sales which are repeat business.


The number of complaints (analysed by reason).
The percentage number of customers complaining.
Average time to settle insurance claims.
Average of satisfaction ratings from customers e.g. quality of service from 1-5 (5
being the highest).
Percentage market share.

The reason for all the above measures is that they would indicate customer satisfaction
for the quality of service customers are receiving.

Internal perspective

Percentage number of quotations and renewals delivered to customers within


three days of processing their details.
Number of complaints from customers regarding inaccurate information, late or
lost paperwork.
Average time to settle insurance claims.
Staff turnover, percentage staff retention or staff absenteeism.

The reason for all of the above measures is that they would indicate failures in the
insurance companys internal processes which may impact on customer satisfaction.
High staff turnover indicates the replacement frequently of staff which can impact upon
the efficiency and effectiveness of internal activities.

Innovation and learning perspective

Training expenditure or training days per employee.


New insurance products launched in the past 6 months, compared with the
competition.
Percentage of staff suggestions for improvement, actually used by management.
Percentage of sales revenue each year from new products launched.

Training expenditure indicates the learning or staff development perspective within the
insurance company and the remaining three are good measures for innovation.

218 | P a g e
Financial perspective

Share price growth.


Percentage of market share.
Dividend per share.
Earnings per share.
Sales or profit growth.
Return on investment (ROI).
Economic value added.

All the above measures concentrate on maximising financial value for a shareholder.

C1 9 Participation in budgets (CIMA P1 May 2005)

Tip: Only three circumstances briefly explained are required. Six possible answers
are included below.

Circumstances where participation leads to poor performance


If managers are asked to participate in setting their own budgets there is the
possibility they will include slack or padding within the budget e.g. have less
sales units forecast than they could realistically achieve, given a reasonable level
of effort. This is the inevitable downside of human behaviour e.g. underestimate
to avoid blame if they were to under achieve.
Participation could create a slower process to formulate a budget because more
consultation is required. This will inevitably increase the cost and time of
formulating budgets, leading to budgets which are late or costly to produce.
The manager could be inexperienced financially. Therefore they may not be able
to contribute effectively if they were to participate. This could lead to
inaccuracies when standards or targets are developed.
In a stable environment where revenue or costs are certain there would be little
benefit in allowing a manager to participate. All this would do is add to the
complexity of formulating the budget which would be unnecessary e.g. with a
university, central government revenue could be fixed and therefore could not be
influenced by a manager.
The manager may not understand the market or industry environment very well
and therefore the targets they set could be unrealistic than if imposed centrally.
Managers could be demotivated due to going through the motions but with no
feeling of real control or influence over the budget they submit e.g. often even
with participation centralised decisions will often amend variables which are
included. If job satisfaction or morale is diminished because of this then
performance could suffer.

219 | P a g e
C1 10 Beyond budgeting (CIMA P1 May 2005)

Part (i)

Hope and Fraser argued that the traditional approach to budgeting should be abandoned
for the following reasons:

The budget process is too rigid and requires conformance to it with not enough
flexibility. With the constantly changing business environment, managers need to
be having more up to date information to help them make decisions.
The budget process is often too bureaucratic, internally focussed and time
consuming.

Fixed budgets dont work today. A budget is a too static an instrument and locks
managers into the past, something they thought last year that it was right. Managers
instead need to be able to adapt constantly their priorities and put their resources where
they can create most value for customers and shareholders.

The beyond budgeting approach may include the following:

Use of rolling budgets focussing on cash forecasts and not cost control.

Budgets revised more frequently and a shorter time horizon when forecasting.

Performance measures using frameworks like the balanced scorecard, developed


to achieve improvement, with benchmarking for continuous improvement, rather
than an internal focus on actual versus budget and cost control.

Benchmarking for continuous improvement.

Part (ii)

The beyond budgeting approach should lead to

A more dynamic performance management process that will enable organisations


to be more responsive and thus create more value.
A management model that enables greater devolution of responsibility that leads
to increased customer satisfaction through continuous improvement.

In the case of W Limited the environment is intensely competitive and likely that
constant change is taking place in the market for computer games e.g. frequent
innovation. W Limited by using long-term detailed budgets, may not be responding to
these changes occurring due to being too internally focussed and reflecting too much on
the past rather than what is occurring in the market at present. Planning for a three year
time horizon may add little value due to the environment changing so fast.

220 | P a g e
By the creation of more external performance measures, perhaps using the balanced
scorecard it could be monitoring more effective information, such as pricing, innovation
and customer satisfaction. By the use of rolling cash budgets it could be more in touch
with liquidity rather than too much focus on internal cost control.

I would recommend changing the current budgeting system by developing more external
measures that will help W Limited continually adapt to changes within the dynamic
environment they face. I would also recommend a shorter time horizon when planning
and more frequent revision to budgets. Greater participation by those who are more in
touch with the market environment will also help plan more effectively.

C1 11 J Limited (CIMA P1 Nov 2005)

Part (i)

Tip: The current method of budgeting used by J Ltd is incremental budgeting, this
approach updates the budget for each period, by taking the previous period as a base
and adding a certain percentage on top of this to allow for growth and inflation.

Zero based budgeting (ZBB) is a method of budgeting which requires each cost element
within a budget, to be specifically justified as though it was being under taken for the
very first time. Without approval, the budget allowance would be zero. Therefore each
cost, every period, must be justified before it can be included in the budget, with
employees encouraged to find alternative ways of accomplishing the same but for less
money, questions are asked such as;
1. Do we really need this activity or expenditure?
2. What would happen if we removed it?
3. Are there alternatives that could eliminate it or replace it?
4. How much will alternative ways that have been identified cost and what are the
benefits?

Part (ii)

Tip: You would need three of the below problems (or others) to earn full marks.

Problems that may arise when implementing ZBB


Management may think too short-term in view when removing items from the
budget.
Entrepreneurial spirit will be needed from staff in order to add value to the
process of ZBB, this may require a change of staff culture given the existing
incremental approach being used.
Change management will need to be carefully planned for to introduce ZBB
techniques.
221 | P a g e
Training would be required in order for staff to participate effectively.
Current information systems may not be compatible to help with a ZBB approach.
ZBB will be a time consuming and costly system to organise and administrate
once introduced.
Many of the decision packages that will be evaluated, rely heavily on qualitative
rather than quantitative decisions, requiring subjectivity when making decisions to
remove or add items for inclusion within the budget. Many decisions therefore
will be hard to make and fundamental mistakes could arise.

C1 12 ST plc (CIMA P1 Nov 2005)

Tip : A maximum of three marks are available for each requirement within part (e),
therefore you need to produce a brief response to include perhaps two issues for part
(e) (i) and three issues for part (e) (ii) or vice versa. The solutions below include
many possible answers as to what could have been discussed.

Part (i)

ERP integrates all manufacturing and related departments and functions for the entire
organisation into one single database, this enables various departments to share
information and communicate with each other in a real time environment. It is an
extension of MRP II.

The budget setting process will now be more effective because it will include an
understanding of other resource or activities rather than just production related
activities and how these relate to one another e.g. marketing, finance and HR
budgets.
The budget setting process will incorporate a more effective cash-flow forecast
for the entire organisation.
Because more enterprise wide activities are integrated, it would improve the
efficiency of the budget setting process e.g. less management time involved in
trying to link different activities together.
It should lower the cost of the budget setting process because all resources and
activities are integrated into one whole information system.
ERP should allow when planning better what if? or sensitivity analysis for the
whole organisation e.g. increasing or decreasing the volume of sales or production
activities, will give a more effective understanding of the impact this will have on
a greater number functions within the organisation.

222 | P a g e
Part (ii)

ERP operates within a real time environment, therefore you would have
continuous exception reporting (variances) in a real time environment. This
would allow better control rather than traditional periodic review.
Less management time and expense of collecting information for comparison to
the budget or standard due to better integration of systems.
Less analysis work for management due to a greater number of exception reports
produced from one single system.

C1 13 W Limited (CIMA P1 Nov 2005)

Tip: You would need to discuss both systems of setting targets from the perspective
of the senior management and finance department for 2 marks. Also around three
issues for how to alleviate this situation described. Five possible ways have been
provided for how to help alleviate the situation.

Poor performance by staff when meeting budgets can often be attributable to the method
of implementation. Ignoring the human side of participation or the introduction of a
target that is either unrealistic or unobtainable can de-motivate staff, this causing more
harm than good. Getting the balance right in terms of an achievable standard or target
is essential during the planning stage of the budgeting process. It is however a problem
in reality to actually define an attainable standard especially if the environment W Ltd
operates within is dynamic or uncertain.

It seems likely that senior management may be trying to impose a tighter budget to
improve performance by using more challenging targets, perhaps more an ideal standard
e.g. a target attained under very favourable conditions. The finance department requires a
standard that is more realistic or accurate e.g. an attainable or expected target, that can be
achieved with a reasonable level of effort from staff.

Practical action the coordinator could take to alleviate the situation described.

1. Use both challenging and realistic targets, but link higher financial reward to the
achievement of the more challenging targets and moderate incentive for the easier
targets.
2. Allow staff to participate when setting targets or budgets, this can help improve
motivation, reduce frustration and increase their job satisfaction.
3. Clear trust and communication developed between management and staff, as well
as the avoidance of being over critical by management if more challenging targets
are not achieved.

223 | P a g e
4. Good planning to ensure whatever standards or targets that are used, they are
realistic and achievable. Periodic review of standards or targets are essential, as
well as consultation with staff during this process.
5. The use of rolling rather than periodic budgeting if a high degree of uncertainty
exists when setting targets. Rolling budgets are good for adaptive planning e.g.
more likely to be regularly updated to take account of changes within the
environment W Ltd is operating within.

C1 14 T plc (CIMA P1 May 2006)

Part (i)

Tip: Below are many possible answers for the two areas to benchmark, the question
states that you are to provide only one performance measure for each area and give a
reason.

The detection of false claims


Training expenditure (or training days) per employee to aid detection of false
claims.
Percentage false claims detected each year.
Percentage of staff suggestions for improving the method of detecting false claims
actually implemented.
Percentage of claims verified by on site inspection, police or fire reports.
Each of the above measures would indicate whether more or less false claims are being
detected each period or whether T Plc is doing enough to detect false claims.

The speed of processing claims


Percentage number of claims by customers processed within 14 days.
Average of satisfaction ratings from customers from 1-5 (5 being the highest) e.g.
speed of service to finalise claims.
Number of complaints from customers regarding inaccurate information, late or
lost paperwork when handling claims.
Average time to settle insurance claims.
Number of claims handled per member of staff.
Number of claims handled per week.
Staff turnover, percentage staff retention or staff absenteeism.
The reason for all of the above measures is that they would indicate the insurance
companys efficiency in processing claims. High staff turnover indicates the replacement
frequently of staff which can impact upon the efficiency of internal activities.

224 | P a g e
Part (ii)
All performance measures and related information about detecting fraud, should be
gathered, summarised and analysed. This information then compared with relevant
industry information available, or other insurance companies which may participate as
partners in a benchmarking exercise. Such information as the rate of detection, staff
training and internal procedures should be compared externally. Internal benchmarking
could also be used to compare information between other claims departments, as well as
trend analysis of these performance measures compared over time.
For the efficiency of processing claims, once a claim is received it should be dated and
date stamped when finalised. All information should be recorded in a computerised
environment to allow automated and real time information to monitor performance
measures and analyse trends over time. Similar information for comparison should be
obtained from other insurance companies, trade journals, media an customers
complaining to ensure T Plc is doing enough in terms of the norm or industry standard for
settling claims.
For both methods standards should be implemented to be used as a yardstick to view
improvement over time. Internal groups should also be established in order to implement
improvements recommended each time benchmarking is undertaken.

C1 15 Product M (CIMA P1 May 2007)

Part (i)

Labour rate planning variance


(Actual production should take) x (old rate - revised rate)
(680units x (900hrs / 600units)) x (30.00 - 31.20)
1,020hrs x 1.20 1,224 (A)

Operational labour efficiency variance Hrs


Actual production did take 1,070
Actual production should take 1,020
50
50hrs x 31.20 (revised rate) 1,560 (A)

Part (ii)

1. Helps highlight variances between those which are controllable and those which
are uncontrollable.

225 | P a g e
2. Helps motivate managers and staff e.g. avoids staff being blamed for faulty
planning and gives a fairer reflection of any operational variances calculated
when assessing any operational efficiencies or inefficiencies. Management and
staff would be appraised more fairly for any favourable or adverse deviations that
are within their control.

3. Use of realistic standards in order to measure performance give better


management information for control purposes.

C1 16 QBD (CIMA P1 Nov 2007)

Qtr 1 Qtr 2

Opening inventory 5,500 5,400
Production (balance) 9,900 12,200
15,400 17,600
Less: Sales (given) 10,000 12,000
Closing inventory* 5,400 5,600

*Closing inventory for:


Qtr 1 = 45% x 12,000 = 5,400
Qtr 2 = 40% 14,400 = 5,600
Qtr 1
Kg
Opening inventory 4,500
Material purchases (balance) 14,925
19,425
Less material usage (W1) 14,850
Closing inventory* 4,575

Value of Purchases ()
Purchases above x 6 per Kg 89,550

W1 Material usage
Production 9,900 x 1.5 Kg = 14,850 Kg

*Closing inventory for Qtr 1 would be 25% of Qtr 2 production needs.

Therefore:
12,200 units x 1.5 Kg = 18,300 Kg.
18,300 Kg x 25% = 4,575 Kg.

226 | P a g e
C1 17 Budgetary planning and control (CIMA P1 Nov 2007)

Note: Only THREE reasons are required SIX possible reasons are given below.

Budgets are a constraint on doing anything different as they stifle innovation and
creativity.

Budgets are costly systems and consume large amounts of management time to
set up and analyse.

Budgets have limited use in a dynamic and fast changing environment as they
would only serve to reflect on the past and not the future, and so therefore they
would be always out of date.

Budgets are generally too internal in focus, ignoring external variables, which
more importantly should be being monitored for example the focus on sales
targets rather than customer satisfaction.

Budgets create barriers within departments due to the feeling of competition for
resources, and do not encourage fluidity between departments which is necessary
in a rapidly changing environment.

Budgets are too short-term in focus for example they may just look at a year.

C1 18 JIT systems (CIMA P1 Nov 2007)


The JIT philosophy requires that products should only be produced if there is an internal
or external customer waiting for them. Traditionally manufacturers stockpiled and were
not concerned whether or not there was any demand for the products. The main aim was
to have enough stock for each stage of production. This would be able to sustain any
sudden surges in demand but as a result of this there would be an excess of raw materials,
work in progress and finished goods being carried. JIT aims ideally for zero stock e.g.
raw materials delivered immediately at the time they are needed, no build up of work-in-
progress, finished goods only produced if there is a customer waiting for them. It aims to
try to achieve this by doing the following:

Closer relationships with suppliers required


Smaller more frequent deliveries need to be managed in order to produce
Cell manufacturing
Reduction in set up time (eliminating non value added activities)
Higher quality machines perhaps multi-purpose with regular maintenance
Involvement and training of staff to maintain flexibility
Elimination of defects

227 | P a g e
In conclusion within a JIT environment lead time is paramount due to the absence of
stock e.g. a chase demand strategy of meeting customer demand. Total quality
management (TQM) will help improve response times by a more efficient and flexible
organisation.

C1 19 Feedback and forward (CIMA P1 Nov 2007)

An example of a feedback control system is a budgetary control system. This would


gather information on past performance from the output of the system e.g. actual
financial performance, and compare this to a predetermined standard or plan (budget)
using any deviations e.g. variances, as a basis of improving future performance through
control action taken.

Feed-forward control is more prevention than appraisal, controlling a system by making


adjustments now to the system in advance before any exceptions occur. It does this by
trying to predict what will happen in the future. Cash budgets would be an example
where the budget would forecast ahead and allow for planned expenditures, highlight any
cash shortages and cash surpluses. Action can be taken now to prevent problems
occurring in the future.

Feedback contrasted to feed-forward control is like closing the door after the horse has
already bolted, in other words there is little you can do about it now, except try and
rectify the situation to avoid it happening again.

C1 20 Nursing homes (CIMA P1 Nov 2007)

Administration costs for period 3

These are all fixed and we need only apply the relevant index to the cost.

Therefore:

100,000 x 104/100 = 104,000

House-keeping costs for period 3

These are all variable and so it is dependent on patient days as well house-keeping costs.

Therefore:

125,000 x 90/100 x 106/100 = 119,250

228 | P a g e
Nursing costs for period 3

This cost is semi-variable and so it is important that we find out the variable element
from the fixed element by using the high low method. However we must first compare
like with like and so we will need to deflate period 2 nursing costs to obtain the period 1
nursing costs.

Therefore:

Period 1 prices = 324,000 / 1.08 = 300,000

Variable costs per index point in period 1


= (300,000 - 280,000) / (125 100) = 800

We now use either nursing costs at the higher level of 300,000 or at the lower level of
280,000 to work out the variable and fixed costs within.

Period 1
Semi-variable cost Variable cost Fixed cost
300,000 800 x 125 = 100,000 200,000

Period 1 Period 2 Period 3


Variable costs x 125 x 108 x 90 x 105
80,000 108,000 102,060
100 100 100 100
Fixed costs x 108 x 105
200,000 216,000 226,800
100 100
280,000 324,000 328,860

Summary of period 3 budget


House-keeping 119,250
Nursing 328,860
Administration 104,000
552,110

229 | P a g e
C1 21 Participative budgeting (CIMA P1 May 2008)

Tip: Only three behavioural consequences are required. Nine possible answers are
included below.

If participation is allowed then it would motivate the project team as their ideas of
budgeting would be valued and it would mean that the budget would be more
likely to be met by the team.

Participative budgeting would also be more likely to be accepted by the team as


they themselves have helped to devised it.

Team members may be aware of more relevant and up to date reactions in the
market which may be crucial in making informed decisions.

If managers are asked to participate in setting their own budgets there is the
possibility they will include slack or padding within the budget e.g. have less
sales units forecast than they could realistically achieve, given a reasonable level
of effort. This is the inevitable downside of human behaviour e.g. underestimate
to avoid blame if they were to under achieve.

Participation could create a slower process to formulate a budget because more


consultation is required. This will inevitably increase the cost and time of
formulating budgets, leading to budgets which are late or costly to produce.

The manager could be inexperienced financially. Therefore they may not be able
to contribute effectively if they were to participate. This could lead to
inaccuracies when standards or targets are developed.

In a stable environment where revenue or costs are certain there would be little
benefit in allowing a manager to participate. All this would do is add to the
complexity of formulating the budget which would be unnecessary e.g. with a
university, central government revenue could be fixed and therefore could not be
influenced by a manager.

The manager may not understand the market or industry environment very well
and therefore the targets they set could be unrealistic than if imposed centrally.

Managers could be demotivated due to going through the motions but with no
feeling of real control or influence over the budget they submit e.g. often even
with participation centralised decisions will often amend variables which are
included. If job satisfaction or morale is diminished because of this then
performance could suffer.

230 | P a g e
C1 - 22 Rolling budgets (CIMA P1 May 2008)

Rolling or continuous budgeting is when a further period is added immediately to the


budget when the earliest period has expired for example if Jan to Mar 2005 is the first
three months of the yearly forecast, once this has expired then Jan to Mar 2006 will be
created and added.

Rolling budgets are good for adaptive planning e.g. more likely to be regularly updated to
take account of changes within the environment the organisation is operating within.

Advantages

There is always an up to date twelve month budget available to management as


the budget is adjusted on a frequent basis.
Rolling budgets are very useful when there is uncertainty, for example cash
budgets are normally rolling as you can spot early warning signals quicker.
There is regular assessment and amendment, to keep the forecast more accurate.
Decisions made using the information from the rolling budget should be far more
accurate and effective as the budget continually up dated.

Disadvantages

To create and continue to update the budget is time consuming to management


and staff
A rolling budgetary system is a costly system to organise and administrate.
It may not be necessary for some companys who operate in a stable industrial
environment, as the costs may outweigh the benefits of up to date information.

231 | P a g e
Solutions Section B

Part C Budgeting and Management Control

C2 -1 M plc (CIMA P1 May 2006)

Part (a)

Tip: The budget is based upon 6,400 labour hours and is being compared to actual
expenditure based on 7,140 labour hours. This is not comparing expenditure on a like for
like basis. Budgeted expenditure needs to be flexed to represent a budget based upon
7140 labour hours and you are given information in terms of how costs behave in order to
this.

Flexed
Budget Budget Actual Variance

Assembly labour hours 6,400 7,140 7,140

$ $ $ $
Assembly labour 49,920 55,692 W1 56,177 W1 485 (A)
Furniture packs 224,000 249,900 W2 205,000 44,900 (F)
Other materials 23,040 25,704 W3 24,100 1,604 (F)
Variable overhead 34,560 38,556 W4 76,340 W5 37,784 (A)
Total variable cost 331,520 369,852 361,617 8,235 (F)

Fixed overhead $ $ $ $
HQ fixed overhead 9,000 9,000 W4 9,000 nil
Stepped fixed cost 18,500 27,000 W4 27,000 nil
27,500 36,000 36,000
Managers fixed salary 2,050 2,050 W1 2,050 W1 nil
Total fixed cost 29,550 38,050 38,050 nil

Total cost 361,070 407,902 399,667 8,235 (F)

W1 Assembly labour
The original budget has removed the $2,050 managers fixed salary ($51,970 - $2,050) =
$49,920, this cost has then been prorated to reflect this cost at 7,140 rather than 6,400
hours ($49,920 x (7,140 6,400) = $55,692.

The actual cost of assembly labour $58,227 has also removed the $2,050 managers fixed
salary ($58,227 - $2,050) = $56,177. We would now have a better comparison due to
the assembly labour analysed between fixed and variable cost.

232 | P a g e
W2 Furniture packs
Given no other information, furniture packs within the flexed budget have been prorated
on the basis of 7140 rather than 6400 hours ($224,000 x (7140 6400) = $249,900.

W3 Other materials
Given no other information, materials within the flexed budget have been prorated on the
basis of 7,140 rather than 6400 hours ($23,040 x (7,140 6,400) = $25,704.

W4 Overhead analysis between fixed and variable cost


Hours $
10,000 90,000
7,500 76,500
2,500 13,500

Variable cost per hour $5.40

The high low method has been used to derive the variable and fixed cost element for
overhead cost. When assembly hours exceed 7,000 hours fixed cost increases.

The above workings are based on 7,500 and 10,000 assembly hours therefore the fixed
cost included in the total cost would be the same.

Variable cost equals the change in total costs ($13,500 2,500 hours) = $5.40.

Therefore budgeted and flexed budgeted variable overhead has been calculated on the
basis of $5.40 an assembly hour.

Fixed cost is the balance of the total cost using either 7,500 or 10,000 assembly
hours above to work this out; given fixed overhead will be the same for both
activities.

$90,000 = Fixed cost + ($5.40 x 10,000 hours). Therefore fixed cost equals $36,000 at
any activity over 7,000 assembly hours.

Given the variable cost of $5.40 per assembly hour would remain constant (no other
information to assume otherwise). The fixed cost at 5,000 hours would be $54,500 =
Fixed cost + ($5.40 x 5,000 hours) $27,500 rather than as above $36,000.

The marginal or stepped fixed cost above 7,000 assembly hours would therefore be
$36,000 - $27,500 = $8,500.

The fixed overhead within the statement above has also been broken down into the fixed
$9,000 central headquarter charge and the remaining stepped fixed cost for better
analysis.

233 | P a g e
Actual fixed costs for April were as budgeted therefore no variances for this period.

W5 Actual variable overhead


The question states that actual fixed cost is the same as budgeted fixed cost. Therefore if
actual overhead was as stated in the question $112,340 and actual fixed overhead
included within this was $36,000 (based on assembly labour hours being greater than
7000) then actual variable overhead for the period would have been $112,340 - $36,000 =
$76,340.

Part (b) (i)

The revised statement is useful for the following reasons.

1. The original format supplied by M Plc did not provide a like for like comparison
when calculating variances. This is because the original budget provided in the
scenario had not flexed the budgeted costs; to represent what costs should have
been based upon the actual assembly hours of 7140. The revised format will
allow a more accurate comparison between budgeted and actual costs for control
purposes.

2. The original format supplied by M Plc did not sub-divide costs into the
classification of either variable or fixed. By the classification of variable and
fixed cost within the revised format, it will allow management to understand cost
behaviour between costs which may rise and fall with an activity level and those
which remain constant. This will aid more effective planning, control and
decision making by central headquarters.

3. The original format supplied by M Plc did not sub-divide fixed overhead into
what was controllable by the department e.g. stepped fixed cost, and what was
uncontrollable e.g. the headquarter central overhead charged. The revised format
will allow a fairer assessment of the department, by the avoidance of criticising
the department for costs which are not within there own control.

Part (b) (ii)

Whilst assembly labour cost may very well have a strong correlation with labour hours
especially if piecework payment schemes are used, this is unlikely for the relationship
between furniture packs and other materials.

Furniture packs and other materials is more likely to be correlated with the type of
furniture being assembled not direct labour hours. For example a dining room table and
chairs maybe more complex to assemble than a wardrobe, however may require less
finished timber. This maybe evidenced by the revised flexed budget statement prepared.

234 | P a g e
It can be seen that both the furniture packs and other material variances were favourable
$44,900 and $1,604 respectively, where as the assembly labour variance was $485
adverse (which I assume if no change in the actual rate paid to staff when compared to
standard rate, this is explained by efficiency). It seems that assembly was more complex
and took longer by staff, but the furniture packs and materials used were far less. If there
were a relationship between material usage and labour efficiency, we would have seen a
greater dependency between these variances e.g. if one is favourable, so too would be the
other.

It would be a good idea for management to investigate a more effective cost driver for
furniture packs and other materials, than labour hours when preparing budgets.

Part (c)
Advantages of allowing participation
Greater motivation for the assembly department manager and his assembly
staff. Because of this targets are more likely to be met. It will also be more
interesting and more involving for the manager increasing his job satisfaction.
Participation means that targets are more likely to be accepted by the manager
and assembly staff e.g. less conflict due to setting targets themselves.
The manager or assembly staff maybe more aware of the environment they
work within e.g. how long different types of furniture take to assemble and
how much timber and material they would consume. The new manager does
seem to have many years experience working within other assembly
departments. Therefore the targets set could be more realistic than if imposed
centrally.
Disadvantages of allowing participation
If managers or assembly staff are asked to participate in setting their own targets
there is the possibility they will include slack or padding within the budget
e.g. budget longer for how long it would take to assemble different types of
furniture. This is the inevitable downside of human behaviour e.g. over estimate
to avoid blame if they were to under achieve.
Participation could create a slower process to formulate a budget because more
consultation will be required. This will inevitably increase time and cost of
formulating a budget.
The manager seems to have little experience previously of working with budgets.
Training would therefore be required in order for him to participate effectively.
I feel that the manager should be allowed to participate in budget setting, however clear
trust, communication and consultation between him and headquarters needs to be
developed. Feedback at frequent intervals, as well as the avoidance of being over critical
by headquarters will all help improve motivation and performance within the assembly
department.

235 | P a g e
C2 2 RF Ltd (CIMA P1 May 2007)

Part (a)

Mth1 Mth 2 Mth 3 Total



RECEIPTS
Sales (W1) 2,940 10,180 15,545 28,665
Capital 16,250 16,250
Total receipts 19,190 10,180 15,545 44,915

PAYMENTS
Material (W2) 0 3,515 3,420 6,935
Labour (W3) 6,105 5,940 6,666 18,711
Variable O/H (W4) 1,332 2,184 2,318 5,834
Fixed O/H (W5) 3,750 5,625 5,625 15,000
Total payments 11,187 17,264 18,029 46,480

Net cashflow 8,003 (7,084) (2,484) (1,565)


Bal b/f 0 8,003 919 0
Bal c/f 8,003 919 (1,565) (1,565)

Workings

(W1) Sales

Mth 1 Mth 2 Mth 3



Sales (units x selling price) 15,000 17,500 20,000
Receipts in 1st month (20%) 3,000 60 3,500 70 4,000 80
and 2% discount given = 2,940 = 3,430 = 3,920
Receipts in 2nd month (45%) 6,750 7,875

Receipts in 3rd month (25%) 3,750


Total 2,940 10,180 15,545

236 | P a g e
(W2) Material

Mth 1 Mth 2 Mth 3 Mth 4


Units Units Units Units
Forecast sales 1,500 1,750 2,000 2,100
Opening inventory 0 (350) (400)
1,500 1,400 1,600
Closing inventory 350 400 420
Production 1,850 1,800 2,020

Material cost 1.90 x 1,850 1.90 x 1,800 1.90 x 2,020


(1.90 x units) = 3,515 = 3,420 = 3,838

Material payments 3,515 3,420

(W3) Labour

Mth 1 Mth 2 Mth 3


Production units 1,850 1,800 2,020

Labour cost 3.30 x 1,850 3.30 x 1,800 3.30 x 2,020


(3.30 x units) = 6,105 = 5,940 = 6,666

(W4) Variable O/H

Mth 1 Mth 2 Mth 3


Production units 1,850 1,800 2,020

Variable O/H cost 1.20 x 1,850 1.20 x 1,800 1.20 x 2,020


(1.20 x units) = 2,220 = 2,160 = 2,424
Payments
60% in 1st month 1,332 1,296 1,454
40% in 2nd month 888 864
Variable O/H payments 1,332 2,184 2,318

(W5) Fixed O/H

Mth 1 Mth 2 Mth 3



Fixed costs (75,000 / 12) 6,250 6,250 6,250
Payments
60% in 1st month 3,750 3,750 3,750
40% in 2nd month 1,875 1,875
Fixed O/H payments 3,750 5,625 5,625

237 | P a g e
Part (b) (i)
If the cost was 1.50 there would be saving of 0.40 per unit.

Mth 1 Mth 2 Mth 3


Production cost savings 1,850 x 0.40 = 740 1,800 x 0.40 = 720 2,020 x 0.40 = 808

Savings received 740 720

Total saving 1,460

New total budget cash flow = Savings received + Current total budget cash flow

1,460 + -1,565 = (105)

Part (b) (ii)


If the cost was 2.20 there would be an extra cost of 0.30 per unit.

Mth 1 Mth 2 Mth 3


Production extra cost 1,850 x 0.30 = 555 1,800 x 0.30 = 540 2,020 x 0.30 = 606

Extra costs to pay 555 540

Total extra cost 1,095

New total budget cash flow = Extra costs to pay + Current total budget cash flow

-1,095 + -1,565 = (2,660)

Part (c)

Tip: A good report format is essential as it will present your answer well and provide
good structure. In addition there maybe up to 5 marks given for a good report format
style. Part (c) in short requires the benefits of what if when preparing cash budgets,
so if you know what these are it will be the backbone to answering this part to the
question. A good approach also would be to think of good examples from the
calculations in the previous parts which will give value within your report.

REPORT
To: Management of RF Ltd
From: Management Accountant
Subject: The benefits of what if analysis
Date: 19th May 2007

238 | P a g e
1. Introduction

The purpose of this report is to discuss the benefits or otherwise of performing what if
analysis when preparing cash budgets.

2. What if analysis

What if analysis looks at varying or changing the key variables to see how the outcome
would change. These changes would be due to revisements of estimations or probabilities
and typically might be material costs or demand. Spreadsheets are the main tool to help
create a what if analysis which allows you to manipulate key figures and giving you
instant feedback as to the impact on the outcomes. Flexible budgets are an example of
what if analysis where you have figures for different levels of demand or production,
allowing you too appreciate how profit might change as a result of these different levels.

Using our calculations from part (b) we can see that by increasing direct material cost by
16% to 2.20 per component there is a 70% fall in total budget cash flow to negative
2,660. However a 21% decrease in direct material cost to 1.50 per component will
result in a 93% increase in total budget cash flow to negative 105.

It can be seen here that direct material costs is very sensitive and a small change will
result in large change to total budget cash flow.

3. Benefits of what if analysis

What if analysis allows for better planning and can be flexed at the end of a period to
correspond and be compared to actual results achieved, giving better information for
control purposes
It gives management a better understanding of the sensitivity of costs and revenues to
small changes and therefore a better understanding of risk.

It allows management to scrutinise different scenarios such as worst case, expected case
and best case.

What if analysis lends itself to the use of spreadsheets by making calculations more
accurate, faster, easier to update changes and cuts down on the time of management to
calculate and analyse the information

The use of spreadsheets means that budgets are paperless and can be sent to and used by
multiple users.

4. Limitations of what if analysis

What if analysis does not account for qualitative information, only looking at changes
to quantitative information.

239 | P a g e
The changes that are made in what if analysis are in isolation and do not give an
understanding of what would happen to budgeted cash flow if multiple changes happened
at the same time.

What if analysis does not give management any idea of the probability of these changes
happening and therefore no true understanding of exposure to risk.

I hope you have found this report useful but should you require any further assistance or
have any questions please do not hesitate to contact me.

Signed Management Accountant

C2 3 Trackit (CIMA P1 May 2008)

Part (a)

Cash budget for Q

Months 1 2 3 Total
Receipts $ $ $ $
Sales (W1) 20,160 65,240 148,820 234,220
Total receipts 20,160 65,240 148,820 234,220

Payments
Direct materials (W2) 0 106,800 104,640 211,440
Direct wages ($10 per unit) 14500 16500 21200 52,200
Direct variable overheads (W3) 18,850 31,600 39,110 89,560
Fixed overheads (W4) 42,000 52,500 52,500 147,000
Total payments 75,350 207,400 217,450 500,200

Opening cash flow 250,000 194,810 52,650 250,000


Net cash flow in the month (55,190) (142,160) (68,630) (265,980)
Closing cash flow 194,810 52,650 (15,980) (15,980)

240 | P a g e
Workings

(W1) Sales receipts

Selling price of tickets are $140 each


Months 1 2 3
$ $ $
Sales 140,000 210,000 280,000

Immediate payment 20,160 30,240 40,320


(15% of sales and 4% discount)

1 month later (25% of sales) 35,000 52,500

2 months later (40% of sales) 56,000


20,160 65,240 148,820

(W2) Direct materials

Direct materials are $60 per unit


Months 1 2 3 4
$ $ $ $
Material costs used in the month 87,000 99,000 127,200 147,600

Opening inventory 0 19,800 25,440


Purchases (balancing figure) 106,800 104,640 131,280

Closing inventory (20% of next month production) (19,800) (25,440) (29,520)

Material costs used in the month 87,000 99,000 127,200

Materials paid 0 106,800 104,640

(W3) Direct variable overheads

Months 1 2 3
$ $ $
Variable overheads 29,000 33,000 42,400

Variable overheads (65% in month) 18,850 21,450 27,560


Variable overheads (35% in next month) 0 10,150 11,550
Variable overheads paid 18,850 31,600 39,110

241 | P a g e
(W4) Fixed overheads

Fixed overheads are $840,000 per annum


Months 1 2 3
$ $ $
Fixed overheads (spread equally) 70,000 70,000 70,000

Fixed overheads (60% in month) 42,000 42,000 42,000


Fixed overheads (15% in next month) 0 10,500 10,500
Fixed overheads paid 42,000 52,500 52,500

Part (b) (i)

Total cost of direct material for the three month period is $211,440 as per the cash budget
in part (a). The cost of the specialist component is included within material cost at $40
per unit. Total unit material cost is $60 which includes the cost of the component.
Therefore total cost of the component is $211,440 x 40/60 = $140,960.

If the unit cost of the component is now $32 then the total cost of the components will
now be $140,960 x $32/$40 = $112,768.

This will result in component cost savings of $140,960 - $112,768 = $28,192 decreasing
total net negative cash flow from $265,980 to $237,788.

Part (b) (ii)

If the unit cost of the component is now $50 then the total cost of the components will
now be $140,960 x $50/$40 = $176,200.

This will result in component cost increase of $176,200 - $140,960 = $35,240 increasing
total net negative cash flow from $265,980 to $301,220.

Part (c)

REPORT
To: Owners of Q
From: Management Accountant
Date: 20th May 2008
Subject: Profitability of Q

1.0 Introduction
The purpose of this report is to offer advice about the profitability of your business and
the prospects revealed by your business plan.

242 | P a g e
Profitability

Annual forecast sales are 27,700 units resulting in total sales of $3,878,000 and profit of
$545,000 if the component cost is $40 per unit. This gives a net profit margin of 14%.

The break-even level of sales (the amount of units sold where all fixed costs have been
recovered and no profit or loss is made) would be 16,800 units and the margin of safety is
39% which is good as this means that sales could fall by nearly 40% before the company
started to make losses.

If the specialist component cost fell down to $32 per unit then total profit would be
$766,600 but if the component cost went up to $50 per unit then total profit would be
$268,000. The business is profitable given the best and worst case scenarios for the cost
of the specialist component.

1.2 Cash budget

The opening cash position in month 1 is $250,000 however given the expenditure over
the next three months it seems that this alone will not be enough to meet cash
consumption. An overdraft would be needed of approximately $16,000 to cover cash
requirements to Month 3.

If the unit cost of the component is now $32 then the total cost of the components will
now be $112,768. This will result in component cost savings of $28,192 decreasing total
net negative cash flow from $265,980 to $237,788.

If the unit cost of the component is now $50 then the total cost of the components will
now be $176,200. This will result in component cost increase of $35,240 increasing total
net negative cash flow from $265,980 to $301,220.

1.3 Business plan and component costs

The business plan presents an optimistic projection of sales and costs and therefore it is
essential to review the assumptions for behind the plan to ensure that they are sensible
and realistic.

There is uncertainly about the cost of the specialist component and this has major impact
on profitability and cash flow. It is essential that this cost is stabilised to get a more
reliable cash budget and profit analysis.

It would appear that cash flow will be a major problem in the opening months and is vital
that additional financing such as an overdraft is arranged in the short term, because in the
long term the business will return profits.

243 | P a g e
Having a list of preferred suppliers or a single supplier will help stabilise the cost of the
component. Giving the supplier an exclusive contract of components needs should help in
agreeing a unit cost. In addition good relations with the supplier will ensure that quality
of service and material as well cost is achieved.

I hope you have found this report useful and should you have any further questions please
do not hesitate to contact me.

Management Accountant

Workings

Forecast sales for the year


= 1,000 + 1,500 + 2,000 + 2,400 + 2,600 + (7 x 2,600)
= 27,700 units

Component cost $32 $40 $50


Contribution per Trackit $140 - $90 + $8 $140 - $90 $140 - $90 - $10
= $58 = $50 = $40
Total contribution $1,606,600 $1,385,000 $1,108,000
Fixed costs $840,000 $840,000 $840,000
Total profit $766,600 $545,000 $268,000
Break-even units $840,000 / $58 $840,000 / $50 $840,000 / $40
= 14,483 = 16,800 = 21,000
Margin of safety 48% 39% 24%

C2 4 X plc (CIMA P1 Nov 2006)

Part (a)(i)

Note: It is not a requirement to produce the production budget for Qtr 5 but this
information will be useful when working out the closing inventory for Material B.

Qtr 1 Qtr 2 Qtr 3 Qtr 4 Qtr 5

Opening inventory* 675 615 495 615 375


Production (balance) 2190 1930 1770 1810 1490
2865 2545 2265 2425 1865
Less: Sales (given) 2250 2050 1650 2050 1250
Closing inventory 615 495 615 375 615

244 | P a g e
*Opening inventory at the start of Qtr 1 would be the closing inventory from the previous
quarter brought forward; therefore this would be 30% of Qtr 1 sales (0.3 x 2250 = 675).

Part (a)(ii)

Qtr 1 Qtr 2 Qtr 3 Qtr 4


Kg
Opening inventory 2956.5 2605.5 2389.5 2443.5
Material purchases (balance) 6219 5574 5364 4998
9175.5 8179.5 7753.5 7441.5
Less material usage (W1) 6570 5790 5310 5430
Closing inventory* 2605.5 2389.5 2443.5 2011.5

Value of Purchases ()
Purchases above x 7 per Kg 43,533 39,018 37,548 34,986

W1 Material usage
Production 2190 x 3 Kg = 6570
Production 1930 x 3 Kg = 5790
Production 1770 x 3 Kg = 5310
Production 1810 x 3 Kg = 5430

*Closing inventory for Qtr 4 would be 45% of Qtr 5 usage (production Qtr 5 1490 x
usage 3 Kg = 4470 Kg). 4470 Kg x 0.45 = 2011.5 Kg.

Part (b)

Material A has been identified as a limiting factor (or principle budget factor) e.g. a
scarce resource which is in short supply. X Plc should try firstly to identify any further
suppliers, reduce wastage levels and run down any inventory for material A it has in the
meantime. Assuming more material cannot be sourced X Plc should conduct a limiting
factor analysis on its different products sold in order to maximise contribution (and
therefore profit). X Plc will maximise contribution by allocating material A to products
that earn the highest contribution per unit of material A consumed.
Normally sales demand is the most binding constraint and therefore the sales budget is
normally produced first and the production budget second (production driven by sales).
However material A in this case would be the most binding constraint, it should be the
first operating budget prepared, then the production budget (because material A restricts
production). Once budgeted production levels have been determined, all other budgets
such as sales, other material, labour and overhead can be prepared.

245 | P a g e
Part (c)

Flexible budget statement


Original Flexed Actual Total
Budget Budget Results Variance
7700 units 7250 units 7250 units

Skilled Labour (4hrs x 15) x units 462,000 435,000 568,750 -133,750 (A)
Semi-skilled labour (6hrs x 9) x units 415,800 391,500 332,400 59,100 (F)
Variable overhead (10 hrs x 2.18) x units 168,000 158,050 185,000 -26,950 (A)
Fixed overhead (14.55 per unit) 112,000 105,488 105,000 488 (F)
1,157,800 1,090,038 1,191,150 -101,112 (A)

Fixed overhead
40% of overhead is fixed therefore 40% x 280,000 = budgeted fixed overhead 112,000.
Fixed overhead absorption rate per unit 112,000 7700 budgeted units = 14.55 per
unit. Within the flexed budget you could assume fixed overhead is 112,000 rather than
105,488. Because this is an absorption costing company I would assume the fixed
overhead absorbed would have been (7250 units x 14.55) 105,488 and therefore the
over absorption for the period 488.

Variable overhead
60% of overhead is variable and varies with total labour hours. 60% 280,000 =
168,000. Total labour hours forecast 7700 units x 10 hours per unit = 77000 hours.
168,000 77000 hours = 2.18 per labour hour.

Part (d)

Incremental budgeting is the process of using current and past budgets as a guide and
adding or subtracting from these budgets to arrive at income and expenditure for a future
financial period. The main problems with this system is that it can encourage wasteful
expenditure and misallocate resources. Zero based budgeting (ZBB) is a method of
budgeting which requires each cost element within a budget, to be specifically justified as
though it was being under taken for the very first time. Without approval the budget
allowance would be zero.
ZBB attempts to remove for every financial period any obsolescent or inefficient
spending, (slack or budget padding), as a process it forces management and
employees to avoid wasteful spending. Incremental budgeting normally updates
expenditure by taking the previous period as a base and adds a certain percentage
on top of these figures to allow for growth and inflation, it therefore can
compound inefficient or wasteful spending over time.

246 | P a g e
ZBB concentrates on the future and challenges past activities making staff more
flexible to environmental change. Entrepreneurship as a culture is not normally
encouraged with incremental budgeting.
ZBB can increase staff motivation due to a budgeting system that is more
interesting and more involving.

Part (e)

Rolling or continuous budgeting is when a further period is added immediately to the


budget when an earlier period has expired e.g. if Jan to Mar 2007 is the first three months
of the yearly forecast, once this has expired then Jan to Mar 2008 will be created and
added. Rolling budgets are good for adaptive planning e.g. more likely to be regularly
updated to take account of changes within the environment the organisation is operating
within.

Advantages for X Plc

Always an up to date budget available for management. This means the budget is
less likely to be out of date due to it being frequently revised or amended during
the budgeting period.
Useful for when an organisation if facing high uncertainty when forecasting e.g.
cash budgets if rolling can spot early warning signals of cash shortages much
quicker. X Plc does face major uncertainties with regard to increased levels of
competition and issues concerning the availability and cost of material and staff,
these factors will cause problems when trying to forecast accurately. Once the
budget has been established at the beginning of a period, it is then constantly
amended on account of developing circumstances.
Regular assessment and amendment keeps the forecast and standard cost
information more accurate within X Plc. This should prevent a variance arising
from external and uncontrollable circumstances, giving more meaning to the
variances which remain and the operational variances more attributable to staff
and management effort during the period.

247 | P a g e
Solutions Section A

Part D Control and Performance Measurement of Responsibilty Centres

D1 -1 EVA and RI (CIMA P2 Pilot paper 2005)

Tip: Economic value added was developed by Stern Stewart & Co.

The economic value created by a division in a given period of time

EVA =

Net cash operating profit after tax


(adjusted for accounting distortions e.g. add back depreciation)
less
Economic depreciation (a reflection of market values)
less
Amortised R&D, advertising and goodwill
less
(adjusted capital employed* x cost of capital)

* using the economic replacement cost of assets.

Tip: Residual income (RI) is calculated by taking the profit a manager earns for a
division less a notional interest charge for the investment within the division e.g. the
return generated from the division less a finance charge from the holding company or
head office using a cost of capital engaged within it.

Profit after tax X


Capital employed x cost of capital (X)
Residual income X

Similarities with EVA and RI

Both are absolute rather than relative measures.


Both deduct a finance charge for the cost of capital used within a division.
Both exclude interest payments in arriving at profit due to using a notional
interest charge instead e.g. the cost of capital.

Differences with EVA and RI

EVA uses the replacement not historical accounting cost of assets when
calculating a finance charge by head office.

248 | P a g e
The profit calculated under both methods are different. EVA uses economic not
accounting profit, it adjusts for accounting distortions such as adding back
accounting depreciation and deducting economic depreciation instead, based upon
the replacement cost of assets. EVA would also amortise R&D, advertising or
goodwill over a useful economic life when arriving at profit.
EVA capitalises costs such as R&D, advertising or goodwill and includes these in
the replacement cost of assets to apply the finance charge. This is because EVA
considers such expenditure as long-term assets building for the future.

D1 2 Controllability principle (CIMA P1 Pilot Paper 2005)

The controllability principle is concerned with assessing performance based upon


measures that can be controlled only by a manager and omitting any items which are
uncontrollable. As an example the head office or holding company could ensure it does
not include and therefore evaluate a manager on the head office overhead they apportion
or interest charges they apply centrally.
Arguments for the controllability principle
It would be considered fairer by a manager if they were not assessed on costs
which are not within their own control. This is likely to improve motivation
and morale.
If a manager was assessed on costs which were not within their control, it
could be argued there would be little they could do about these costs any way,
even if exceptions were reported.
Arguments against the controllability principle
Political arguments may occur over such costs which are more subjective than
objective when determining controllability. It is not always black and white when
determining controllability from uncontrollability.
Just because a cost is uncontrollable does not mean a manager being assessed
should ignore it altogether. As an example if managers recognise there is an
interest charge by head office based upon the capital employed used within a
division, then to hold them more accountable could help improve efficiency by
the minimisation of capital employed. In the case of head office charges, lack of
accountability could encourage over consumption of these resources provided
centrally.

D1 3 Transfer pricing (CIMA P1 Pilot Paper 2005)

The following problems could arise when divisions in different countries buy and sell to
one another and a transfer price needs to be determined.

249 | P a g e
Exchange rates
Often the buying and selling division within a group will use different currencies and
these can fluctuate from one day to another. The difficulty in these circumstances is what
transfer price to actually set e.g. which exchange rate do you use. Another problem can
arise when settling liabilities between the two divisions e.g. exchange gains and losses
occur when payment arises.
Taxation
Different countries may set taxation rates higher or lower than other countries for profit
earned by resident companies. In cases where the buying divisions country is higher
taxation compared to the sellers, a higher transfer price can shift profit away from the
buyer to the seller, due to a higher cost incurred. This will help reduce the tax liability of
the group. Some countries also apply withholding tax on profits repatriated as dividend
by an overseas subsidiary to its parent.
Protectionism
Certain counties may restrict the flow of goods between divisions due to import quotas in
place or a tariff could be applied to the price of each good sold. In cases where tariffs are
applied, it could be important to minimise the transfer price to the buying division in the
overseas country to minimise this tax liability. Other problems could include a restriction
on foreign currency available to a buying division e.g. exchange controls, when settling
intra-group liabilities. In these cases netting off liabilities between divisions before
settlement takes place may help avoid such restrictions.
Worldwide prices of other suppliers
Due to exchange rate fluctuations a buying division may find it cheaper to buy externally
from another supplier rather than the selling division. If the marginal cost of producing
each unit for the selling division is lower than the price the buyer pays externally, this
would not maximise group profit.

250 | P a g e
D1 4 EVA (CIMA P1 May 2005)

Part (i)

The economic value created by a division in a given period of time

EVA =

Net cash operating profit after tax


(adjusted for accounting distortions e.g. add back depreciation)
less
Economic depreciation (a reflection of market values)
less
Amortised R&D, advertising and goodwill
less
(adjusted capital employed* x cost of capital)

* using the economic replacement cost of assets.

Economic value added was developed by Stern Stewart & Co, a US management
consultancy firm. EVA is an absolute measure of the economic financial wealth
generated by a division or organisation over time. It deducts a finance charge using a
cost of capital, applied to the replacement cost of assets used by the division or
organisation.

EVA capitalises costs such as R&D, advertising or goodwill and includes these in the
replacement cost of assets to calculate the interest charge. This is because EVA
considers such expenditure as long-term assets building for the future.

EVA uses economic not accounting profit, it adjusts for accounting distortions such as
adding back accounting depreciation and deducting economic depreciation instead, based
upon the replacement cost of assets. EVA would also amortise R&D, advertising or
goodwill over its useful economic life when arriving at economic profit.

251 | P a g e
Part (ii)

Tip: Only three reasons briefly explained are required for how EVA might affect the
behaviour of managers. Five possible answers are included below.

EVA may affect the behaviour of divisional senior executives in the following ways.

1. They would concentrate their investment decisions on maximising shareholder


value or financial wealth of their shareholders.
2. They would concentrate on the maximisation of cash or contribution which is
more likely to maximise shareholder value e.g. EVA can not be manipulated by a
managers choice over the accounting policies they might use.
3. They would concentrate on long-term decisions as opposed to short-term
decisions e.g. with relative measures like return on investment (ROI) often new
investments deliver low profit and have high accounting book values in the early
years. This often discourages managers in the short-term from undertaking
investment due to a low ROI.
4. Because a finance charge is applied against the replacement cost of assets, it
forces managers to use and invest in assets more efficiently.
5. EVA will not discourage expenditure on long-term assets building for the future
such as marketing or research and development. This is because these items will
not be deducted entirely when arriving at economic profit, instead amortised over
the period of the expenditures useful economic life. This would lead to perhaps
greater EVA when compared to the measure of accounting profit. With
accounting profit it is more likely the entire cost would be deducted and therefore
could deter a manager if assessed on accounting measures such as residual income
or return on investment.

D1 5 WD, PD & TD (CIMA P1 May 2005)

Part (i)

WD

Two thirds of output sold by WD is to PD therefore the transfer price it sells at will
determine significantly the profits and return on investment (ROI) of WD. It is possible
that WD may not be controlling its cost due to cost-plus pricing used, which may add a
mark-up upon actual not standard cost. This would mean that WD may not be controlling
its costs, passing on any inefficiency by charging higher prices to PD.

252 | P a g e
The behavioural consequences in this case is that WD could in fact manipulate the
internal system of cost-plus pricing used by increasing its own costs in order to add a
mark-up and earn higher profit absolutely. As an example if 25% mark-up was used
100 cost + 25% mark-up would earn 25 profit per unit of timber, but if cost was
increased to 150 a 25% mark-up would earn 37.50 profit per unit of timber. This will
not improve group profit.

WD may therefore have no incentive to control cost and due to two thirds of its output
sold to PD, it may concentrate solely on supplying WD rather than external customers
due to potentially more profit. This will not improve group profit.

PD

The policy of C Plc is that PD must buy all its output from WD and sell all its output to
TD. It seems that PD is wholly reliant on both of the other two divisions, an intermediary
in the supply chain. The same consequences of behaviour as WD can apply here, that is
there would seem to be no incentive for PD to control cost, as well as any inefficiency of
WD passed on to TD in the form of higher prices.

There is also the behavioural consequences of PD and WD minimising much needed


investment in non-current assets as a way of improving return on investment. Under
investment in assets causes the net book value of assets to decrease over time. Even if
profit remains static ROI will improve, yet the managers of both divisions will have done
very little to improve financial results.

TD

Due to TD only purchasing items it sells from PD, it is highly reliant on the cost and
quality of products made by PD to earn profit. Due to the last division in the internal
supply chain it could end up paying for the inefficiencies of the other two divisions, this
effecting its ability to be competitive.

This would be incredibly frustrating for the manager of TD, especially if products can be
procured from external suppliers at far cheaper prices, as well as increasing product range
and quality if they did this. Morale and motivation in this division maybe low as a
consequence, as sales and profit earned are highly reliant on the cost, quality and range of
products sold to them by PD, they have very little control over these factors due to only
selling items purchased from PD.

253 | P a g e
Part (ii)

If the internal transfer price for items sold by PD to TD were at market price e.g. similar
to a price on the open market, many of the frustrations of TD could be avoided. At least
TD will buy at a fair market price and would not be frustrated by the fact it could procure
products from external suppliers cheaper. Also there would be greater incentive for PD
to control cost, the only way to improve profit would be to control cost and improve
efficiency, due to its prices set by the open market.

The problems of this approach is that it still may do very little to motivate PD to improve
quality, or increase or innovate its products made, due to the fact that TD according to
company policy will buy all of its products from PD anyway. It would also frustrate the
manager of division PD, if inefficiencies are still being passed on by WD due to cost-plus
pricing still applying between these two divisions. There is also the difficulty of
determining what is a definitive market price for PD products sold.

There is good argument that PD should in fact be controlled as a cost rather than profit
centre, due to the fact it does not buy or sell outside the group. If this were the case then
supply should take place at the marginal cost of production according to economic
theory. Perhaps at standard rather than actual marginal cost as an incentive for PD to
control its efficiency and cost. There is the added advantage in this case that TD could
purchase items made by PD at far less than market price, this could allow them to sell to
garden centres or similar outlets at reduced cost improving sales volume and profit,
however this also could be dangerous due to the fixed cost of PD not being considered.

If marginal costing was used there would also be little incentive for the manager of PD to
supply, as marginal cost would not cover the fixed costs of the PD division and no profit
could be earned. Perhaps instead either full cost-plus (using a standard full cost) or a
two-part tariff system should be used instead. A two-part tariff system could include an
element of profit to give PD the necessary motivation to supply, as well as control its
own cost e.g. a fixed fee to cover fixed cost and some profit, and a charge for each unit
supplied to cover the marginal cost of production.

D1 6 G group (CIMA P1 May 2007)

A transfer price must be selected by group however it must recognise the value expected
by both the buying and selling divisions. The price set should encourage internal transfer
between divisions as this would serve in the best interests of the group as a whole and not
just for themselves. The recognition of a satisfactory transfer price will also allow for a
better understanding of performance of each division by head office.

254 | P a g e
If there is an external market price for a product that could be transferred internally the
transfer price should be set at this price less any savings made by transferring internally.
This would mean that there would be an incentive by both divisions to transfer internally.
If there is a limitation on external demand then the transfer price should be set at the
marginal cost needed to manufacture the product, but a bonus incentive is needed to
manufacture and transfer.

A two-part tariff system could include an element of profit to give the necessary
motivation to supply, as well as control costs e.g. a fixed fee to cover fixed cost and some
profit, and a charge for each unit supplied to cover the marginal cost of production.

Participation of the divisions in the creation of the bonus scheme means that it would be
more likely to be accepted and there would be less conflict because they had accepted
and developed the scheme themselves.

The scheme should be clear and simple to understand by all staff and should motivate and
reward divisions on a responsibility accounting basis. This means that they should be
assessed on those costs and revenues that they can only control.

D1 7 Digital equipment (CIMA P1 May 2008)

Part (a)

To measure technology leadership we could compare the number of new products


launched each year versus the expenditure on research and development. This will tell us
how effective innovative ideas developed by the company have been by looking at
whether or not they have gone into commercial production for sale to customers.

Part (b)

The data that should be collected would include customer ratings and feedback logged
when support is sought by customers. This could be done on the phone after support has
been given or on the internet if support is web-based. This would help determine how
satisfactory support given to customers is.

Part (c)

Benefits of internal benchmarking over external benchmarking (please note just three are
needed):

Internal benchmarking information has been created by the company and not a
third party like external benchmarking information, therefore internal
benchmarking information may not be more accurate and timely than external
benchmarking information.

255 | P a g e
Internal benchmarking information has specifically created for the company
unlike external benchmarking information, and so comparisons made with
internal benchmarking information are more relevant.
External information maybe incomplete as it is difficult to get information about
all competitors but internal information is complete as it is generate by the
company and there are no restrictions to access.
Internal information will allow continuous improvement by sharing ideas between
managers this is not possible with mangers of other companies.

Part (d)

Reasons why ROI may not be a good performance measure (please note just three are
needed):

The biggest drawback of ROI is that it is an accounting and not a cash-based


measure, which it can be manipulated. The choice of accounting policy (stock
valuation or depreciation) can also distort and create different profit and fixed
asset levels within divisions.

ROI creates an incentive not to invest in the companys long-term future and
creates short-termist behaviour by divisional managers. Under investment in
fixed assets causes the net book value of assets to decrease over time. ROI
improves over the life of an asset where little or no reinvestment takes place.

If profits remain static then ROI will improve, yet the manager may have done
very little in terms of improving results.

Managers may also be over zealous to cut back expenditure in order to improve
the profit of the division e.g. advertising or training of staff, and this can
jeopardise the longer-term profit of the business.

ROI may create political arguments over such costs as head office apportioned
overhead or interest charges by head office which have direct negative impact on
ROI.

Goal incongruent decisions maybe taken for example where an asset generates a
positive net present value but would fail on the criteria of ROI used by the
manager, and hence the project is rejected.

256 | P a g e
Solutions Section B

Part D Control and Performance Measurement of Responsibilty Centres

D2 -1 Y and Z (CIMA P1 Nov 2005)

Part (a)

Tip: Total assets less current liabilities (TALCL) or net assets is also equal to shareholders
equity + long-term liabilities. It is imperative to read the question; the operating statements are
for a single month (October) therefore profits before tax must be annualised in order for return
on investment to be calculated.

Return on investment (ROI) or Return on capital employed (ROCE)

ROI = Operating profit before interest and tax


Capital employed (TALCL)
Division Y

15.0% = (0.122 million x 12 months)


(9.76 million net assets)

Division Z

20.0% = (0.021 million x 12 months)


(1.26 million net assets)

Discussion of relative performance


Division Z has the highest return on investment (20%) in comparison to division
Y (15%).
Both divisions exceed the target of 12% per annum set by the parent company.
However division Y will be at greater risk if the target return on investment is
increased.
Both are profitable and generate a positive contribution for the group.
In absolute terms division Y is the largest division in terms of net assets and
generates a greater absolute profit than division Z (122,000 compared to 21,000
per month). This is almost six times the level of absolute profit in comparison to
division Z.

257 | P a g e
Both divisions operate in similar markets however division Z has almost the same
absolute level of variable cost as division Y, even though its sales revenue is
almost half the amount. Division Y has variable cost to sales of 38.3% (0.345m
0.9m) and division Z 56.2% (0.312m 0.555m). This indicates that
division Y looks more operationally efficient. Division Z has a much lower net
assets value than division Y which could indicate that its assets are older and
therefore more inefficient.
Division Y has a greater level of apportioned central cost (338,000 per month),
which is almost twice the amount that division Z is charged. This arbitrary
amount charged will effect the profitability of the two divisions by a great extent
e.g. for division Z an 11.7% increase in apportioned central cost would reduce
profit per month to zero (21,000 180,000).
If the uncontrollability principle is applied and central apportioned cost were to be
removed then the ROI of the two divisions would be as follows
Division Y (0.46m x 12) 9.76m = 56.6%
Division Z (0.201m x 12) 1.26m = 191.1%
More information will be needed for how central apportioned costs are allocated to each
division, as well as information on the age of the net assets used within each division, to
make a more effective comparison of financial performance between the two.
Part (b)

Tip: Residual income (RI) is calculated by taking the profit a manager earns for a division less
a notional interest charge for the investment within the division e.g. the profit generated from
the division less a finance charge from the holding company or head office using a cost of
capital. Accounting profit is calculated the same way as for return on investment (ROI).
Profit before interest and tax X
Capital employed x cost of capital (X)
Residual income X

Residual income in comparison to return on investment


Absolute rather than a relative measure
Deducts a finance charge for the cost of capital used for a division

Division Y
m
Profit before interest and tax (0.122 million x 12 months) 1.464
Capital employed x cost of capital (9.76 million net assets x 12%) (1.171)
Residual income 0.293

258 | P a g e
Division Z
m
Profit before interest and tax (0.021 million x 12 months) 0.252
Capital employed x cost of capital (1.26 million net assets x 12%) (0.151)
Residual income 0.101

Even though division Y has a lower return on investment (15%) compared to


division Z (20%), it does create greater wealth for the group in terms of the
absolute size of residual income it earns.
This is something that return on investment considered in isolation will not
demonstrate because it is a relative not absolute measure of return.
The implications of this information is that it demonstrates that division Y
contributes greater wealth to the profits of the group and therefore its
shareholders. It is a superior measure when contrasted to return on investment.
However one single measure by itself will never allow a complete understanding
of financial performance.

Part (c)

Tip: The biggest drawbacks of both ROI and RI are that they are accounting not cash-
based measures. Such financial measures can create short-term behaviour by
divisional managers. If divisions under invest in non current assets, this causes the
net book value of net assets or capital employed to fall in value over time. If profits
remain static both ROI and RI will improve, yet the manager would have done little in
terms of improving financial results.
The choice of accounting policies used e.g. stock valuation, depreciation methods or
the way central costs are apportioned, will also distort and create different profit and
net asset levels within divisions. Managers may also be over zealous to cut back
expenditure in order to improve the profit of the division e.g. advertising, training,
research and development, and this can jeopardise the long-term profit of the division.
These methods also can frustrate managers and can cause political argument occur
over the allocation of cost centrally such as central apportioned overhead or interest
charges.
The below gives a comprehensive listing of the advantages and disadvantages of
using the two methods, however one mark, up to a maximum of 3 would have been
awarded for each brief comment you make. The level of comprehension within the
solution below would therefore not be needed.

259 | P a g e
Advantages of ROI
A relative measure so different sized divisions can be compared better than RI
when assessing financial performance.
Well understood by users of accounts.
Forces the manager to be efficient with resources (assets) used.
Disadvantages of ROI
Disincentive to invest in net assets to improve ROI. ROI improves over the life of
an asset where little or no reinvestment takes place.
Goal incongruent decisions where a new investment generates a positive net
present value, but would fail on the criteria of ROI used by the manager. This is
because new projects often will have low profit and high net book values in the
early years of investment.
An accounting not cash based measure therefore ROI can be distorted.
Advantages of RI
Consistent with profit maximisation and an absolute rather than relative measure.
Brings home the idea about cost of finance for a manager.
Unlikely when contrasted with ROI to act as a disincentive to invest e.g. as long
as profit is earned it should improve RI.

Disadvantages of RI
An accounting not cash based measure therefore RI can be distorted.
Cannot compare divisions of different sizes very well.
May discourage investment in net assets in order to lower the interest or finance
charge applied to a division.
RI improves the older net assets become e.g. a lower finance charge when applied
to the historical cost of assets within the division.

260 | P a g e
Tip: Only a few sentences about two further methods of assessment would be
required for one mark each. Possibilities could include the following.

Controllability principle applied when calculating ROI or RI e.g. ignoring


central costs apportioned.
Cash-based methods such as throughput accounting, net present value or
economic value added.
Variance analysis and budgetary control through exception reporting.
Ratio analysis e.g. profitability, liquidity and investor ratios.
Other non-financial ratios e.g. sales per square metre, number of complaints,
staff turnover, market share, sales growth, new customers or repeat business.
Using performance frameworks like the balanced scorecard.

Other methods of appraising divisions


Economic value added is an absolute cash based measure of the economic financial
wealth generated by a division over time. It deducts a finance charge using a cost of
capital, applied to the replacement cost of assets used by a division. They method
concentrates on the maximisation of cash or contribution which is more likely to
maximise shareholder value. EVA can not be manipulated by a managers choice over
the accounting policies they might use.
A new approach to strategic management was developed in the early 1990's by Drs.
Robert Kaplan (Harvard Business School) and David Norton. The balanced scorecard
suggests that we view an organisation from four perspectives.

Customer perspective
Internal perspective
Innovation and learning perspective
Financial perspective

261 | P a g e
D2 2 FP (CIMA P1 May 2006)

Part (a)

Tip: A good approach here would be to work out a standard cost for the service
department for each repair, breaking down the standard cost into variable (marginal)
and fixed cost. This will help understand the principles of how servicing is being
charged by the service department and also make the profit calculations easier.

Standard cost of one repair



Parts 54.00
Labour (3 x 15 per hour) 45.00
Variable overhead (3 x 10 per hour) 30.00
Variable (marginal) cost 129.00
Fixed overhead (3 x 22) 66.00
Full cost per repair 195.00
40% mark up on full cost 78.00
Standard price 273.00

Repairs carried out by the service department (at full cost plus 40%)
Sales Service FP
Department Department Group

Sales 120,000 W1 136,500 W3 120,000 W5
Cost of sales 136,500 W2 97,500 W4 97,500 W6
Profit -16,500 39,000 22,500
W1 2000 x 60
W2 500 x 273
W3 500 x 273
W4 500 x 195
W5 2000 x 60
W6 500 x 195

262 | P a g e
Repairs carried out by the service department (at marginal cost)
Sales Service FP
Department Department Group

Sales 120,000 W1 64,500 W3 120,000 W5
Cost of sales 64,500 W2 97,500 W4 97,500 W6
Profit 55,500 -33,000 22,500
W1 2000 x 60
W2 500 x 129
W3 500 x 129
W4 500 x 195
W5 2000 x 60
W6 500 x 195

Repairs carried out by RS


Sales Service FP
Department Department Group

Sales 120,000 W1 0 W3 120,000 W5
Cost of sales 90,000 W2 33,000 W4 123,000 W6
Profit 30,000 -33,000 -3,000
W1 2000 x 60
W2 500 x 180
W3 500 x nil
W4 500 x 66 F/OH per unit
W5 2000 x 60
W6 (500 x 180) + 33,000 F/OH

Part (b) (i)

The transfer price for repairs undertaken by the service department on behalf of the sales
department, will determine significantly the profits of the sales department. If full cost
plus a mark up is being used, it is possible that the service department may not control its
cost and pass on any inefficiency, by charging higher prices to the sales department. In
effect the more inefficient the service department becomes, the higher the cost and
therefore price it would charge to the sales department. The sales department paying for
these inefficiencies by reduced profit, due to the higher price.

263 | P a g e
Another problem of full cost plus is that the price charged maybe too high, ignoring
competition in terms of how much other companies would charge for the same service.
The price charged by the service department is currently 273 per repair to the sales
department. Compared with what RS would charge this would be 180 per repair. If
these departments are to run as profit centres, the sales department may go elsewhere to
improve its own profitability.

This would actually be a goal incongruent decision because it would lower overall
profitability for the group. Fixed cost would be incurred regardless by the service
department, whether it supplied the service department or not. The actual cost to the
group in relevant costing terms would be the internal marginal cost of each repair,
currently budgeted at 129. If the sales department started to use RS the external
marginal cost to the group would be 180, therefore the group profit would fall by (500
repairs x (180 - 129) = 25,500. The group would therefore be worse off.

Part (b) (ii)

Tip: One mark will be available for each relevant issue you discuss. Six possible
solutions have been provided below.

Does RS have a solid track record for delivery e.g. references from other satisfied
customers in terms of the quality of service they provide should be sought.
The financially stability of RS should be investigated e.g. FP should obtain
previous sets of accounts and obtain credit ratings.
Does RS have the resources required to work with FP e.g. will they cope with
peak periods when demand for there services may be high.
Will RS be increasing prices above 180 per repair in the long-term?
Is any of the fixed overhead currently committed by the service department
avoidable, if there is an internal reduction in demand for there services.
Can the service department find external work to fill any spare capacity caused by
a decline in internal demand?

It will be important to have a good service level agreement (SLA) with RS that gives a
minimum level of service expected and offers rescission of the contract, should certain
conditions be broken by RS e.g. for non-performance or a low standard of service over a
period of time.

264 | P a g e
Part (c)

Tip: A profit centre is when a manager will be accountable for both creating revenue
and controlling costs. The sales and service departments will therefore be assessed on
profit earned. One mark will be available for each relevant issue you discuss. Many
possible solutions have been provided below, it is important you give both advantages
and disadvantages not just one or the other.

Advantages of profit centres

Linking financial reward to the achievement of profit targets set for staff will be
more motivating for the department managers, encouraging them to increase sales
for their departments and be more efficient in controlling costs.
A quicker response to changes in the market environment by departments due to
having greater autonomy to make decisions. Greater decentralisation of decision
making will also give greater responsibility to each manager improving their job
satisfaction.
Due to less centralised decisions it should create a culture of greater
entrepreneurial spirit and initiative from managers within departments.
Less bureaucracy and centralised interference by senior management should
provide greater efficiency of decision making by managers in each department.
Senior management can free up time consuming affairs of running each
department and concentrate on the corporate rather than business strategy of each
department.

Disadvantages of profit centres

Each department maybe run from the individual managers view rather than the
corporate point of view. As an example the sales department may choose RS as a
supplier over the service department leading to dysfunctional behaviour e.g. loss
of group profit.
Because of less senior involvement there maybe less control and coordination of
activities within each department.
Profit centres may duplicate many activities such as human resource management,
finance or IT, placing less reliance on centralised activities being offered. This
could increase fixed overhead for the group as a whole. For centralised services
there could be a central overhead charge by senior management for these services
which often creates political argument and may frustrate managers if these
charges effect their own profit.
Because of more delegation to middle management this may add further fixed
overhead of running FP due to greater supervision required. It may also create
less efficiency and consistency of decisions within departments.

265 | P a g e
The department managers may have little experience previously of working
autonomously; this could lead to poor decisions being made. Training may be
required in order for them to participate effectively.

D2 3 ZZ group (CIMA P1 Nov 2006)

Part (a)

Income statements Division X Division Y



Sales (W1) 100,000 270,000
Cost of sales
Variable costs (W2) (50,000) (144,000)
Contribution 50,000 126,000
Fixed costs (15,000) (100,000)
Profit 35,000 26,000
Less finance cost (W3) (6,000) (11,000)
Residual income 29,000 15,000
ROCE (W4) 58.33% 23.64%
Operating profit margin (W5) 35.00% 9.63%
Asset turnover (W6) 1.67 2.46

Working 1 Sales

Division X would sell a the external market price of 20 per unit

Currently 3,000 units transferred to division Y and 2,000 units sold externally. This
gives the current sales of {(3,000 x 10) + (2,000 x 20)} 70,000

Revised sales would be the entire 5,000 units are sold at 20 = 100,000

Division Y sales would remain unchanged as they dont transfer internally.

Working 2 Variable costs

Division X remain the same at 50,000 (10 per unit marginal cost)

Division Y 3,000 units transferred from Division X would now cost 20 per unit.

Current variable costs 114,000


Less 3,000 x 10 (30,000)
Add 3,000 x 20 (60,000)
Revised variable costs 144,000

266 | P a g e
Working 3 Finance charge

Capital employed x cost of capital

Division X - 60,000 x 10% = 6,000


Division Y - 110,000 x 10% = 11,000

Working 4 Return on capital employed (ROCE)

ROCE = Profit / capital employed x 100%

Division X - 35,000 / 60,000 x 100% = 58.33%


Division Y - 26,000 / 110,000 x 100% = 23.64%

Working 5 Operating profit margin

Profit / turnover x 100%

Division X - 35,000 / 100,000 x 100% = 35.00%


Division Y - 26,000 / 270,000 x 100% = 9.63%

Working 6 Asset turnover

Sales / capital employed

Division X - 100,000 / 60,000 = 1.67


Division Y - 270,000 / 110,000 = 2.46

Part (b)

Performance measures Division X Division X Division Y Division Y


before after before after
Residual income (1,000) 29,000 45,000 15,000
ROCE 8.33% 58.33% 50.91% 23.64%
Operating profit margin 7.14% 35.00% 20.74% 9.63%
Asset turnover 1.17 1.67 2.46 2.46

The proposed change in policy will benefit Division X greatly and but at the expense of
Division Y.

Division Xs revenue and therefore profit increases by 30,000. This is because they are
now selling 3,000 units at 10 extra (30,000). This therefore increases their residual
income to a positive 29,000. Their return on capital has increased hugely from 8.33% to
58.33%. The operating profit margin has also increased to 35% from 7.14% and their
asset turnover is much improved.

267 | P a g e
However Division Ys performance is not so good. With the increased cost of
component C, their variable costs have increased to 144,000 from 114,000. This is the
30,000 increase which has been passed on from Division X. The result of these
increased costs has resulted in lower residual income (only 15,000), the ROCE is more
than half its original value at 23.64%. The profit margin has reduced from a healthy
20.74% to a depressing 9.63%. Their asset turnover remains the same.

The result of altering the transfer price to 20 per component will be great for Division X
as their performance measures will be greatly improved. Therefore the managers of
Division X will really want to push for this new proposal.

However for Division Y, their performance measures will be vastly reduced, resulting in
lower moral. Therefore Division Y may choose to source their component C from
elsewhere at cheaper rates. This will lead to goal incongruence which is not in the best
interest of the group.

Part (c) (i)

With the new proposal, the managers of Division X will want to set the transfer price at
the same rate as the external market price of 20 per unit. This will improve their
financial performance immensely.

Division Y will lose out if the transfer price is set at 20 as their performance measures
deteriorate drastically. So therefore division Y managers will want to negotiate a lower
transfer price. If Division X does not agree to a lower price, Division Y may purchase
component C externally.

The marginal cost to the group of producing component C is 10 and if Division Y


purchases externally at a price higher than 10, the group as a whole is losing out.

If Division Y does source component C externally, Division X will have spare capacity.
Currently there are only 3,000 units of external demand, which means that there will be
2,000 units of spare capacity. If the fixed costs cannot be avoided, this again means that
ZZ group as a whole is losing out and it will impact the bottom line profit.

A good transfer price is one where both divisions are happy with and it doesnt impact
the group as a whole in a negative way. This usually means that divisions buy and sell
internally and do not source goods from outside the group if they can buy them internally.

With the current situation it is unlikely that both divisions can agree on a suitable transfer
price. This may cause hostility between both divisions leading to goal incongruence and
low morale. The group may have to intervene to ensure that profitability of the group as
a whole is not negatively impacted.

268 | P a g e
Part (c) (ii)

A good way of pleasing both divisions where there is a problem of a suitable transfer
price could be methods such as a dual pricing or two-part tariff system. These methods
of transfer pricing ensures both divisions are happy and that they buy and sell to each
other.

A dual transfer price is achieved by setting one transfer price for Division X and another
transfer price for Division Y. The transfer price for Division X to sell will be set at the
external market rate and the transfer price for Division Y will be set at the marginal cost
of producing component C. The difference between the two transfer prices would need
to be reconciled by head office, which is one of the major drawbacks of this method as it
is very time consuming.

A two-part tariff system is where a fixed charge per period is given to the seller
Division X - irrespective of the amount of units transferred by the seller plus a fixed rate
(at marginal or variable cost) charged for each unit transferred. Such a system would
include an element of profit to give Division X the necessary motivation. Such a system
aims to ensure the seller covers the fixed cost of production, and receives a selling price
for each unit supplied to cover the variable or marginal cost of production.

Both of these transfer pricing policies would give autonomy to Division X and Y.
However agreeing a transfer price can be very time consuming especially if the divisional
managers are not experienced in this area. Some involvement by the group may be
necessary to ensure that negotiations go ahead and that both divisions do agree.

D2 4 Computer manufacturer (CIMA P1 Nov 2007)

Part (a)

The European subsidiary has offered price $95 to supply the chips to American
subsidiary and it would earn the same contribution (see last paragraph).
Therefore the contribution earned from this would be the same as to external sales
($105 - $60) $45.
The spare capacity will be used resulting in higher fixed costs.
Assume all the American chips are delivered (300,000) and the rest sold to
externally.

Increase in European contribution 200,000 x $45 $9 million


Increase in fixed costs $26m - $20m ($6 million)
Increase in European profits $3 million

Extra cost to American subsidiary 300,000 x ($95 - $90) ($1.5 million)

Net increase in profit on group ($3m - $1.5m) 1.5 million

269 | P a g e
Part (b) (i)

Assume all orders to American fulfilled first 300,000 units and remaining go to external
sales 500,000 units. This is a loss of 100,000 units sold externally.

Current contribution from external sales (105-60) per unit $45


Lost contribution due to sales internally 100,000 units x $45 $4.5 million
Additional fixed costs $6 million
Total additional contribution required ($4.5m + $6m) $10.5 million
Contribution required per unit sold to America ($10.5m / 300,000 $35
units)
Variable cost of selling a unit to America ($95 - $45) $50
Transfer price $85

Part (b) (ii)

The return on assets = profit / annual fixed costs x 100%

This is currently 35%

Profit / $26m x 100% = 35%


Profit = $26m x 35/100 = $9.1 million

Therefore a profit of $9.1 million needs to be generated.

Target profit $9.1 million


Add back fixed costs $26 million
Total contribution ($9.1m + $26m) $35.1 million
Contribution from external sales (500,000 units x $45) $22.5 million
Contribution required from American sales (35.1 22.5) $12.6 million
Contribution per chip to American ($12.6m / 300,000) $42
Variable cost of selling a unit to America ($95 - $45) $50
Transfer price $92

Part (c)

REPORT
To: Managing Director
From: Accountant
Date: November 2007
Subject: Transfer pricing and performance measures

This report will discuss issues raised by the directive and the introduction of the
performance measures.

270 | P a g e
Introduction

The American subsidiary currently sources the chips externally paying $90 per chip for
the requirement of 300,000 chips. The European subsidiary has spare capacity of
200,000 chips and here there is an opportunity to source the chips internally rather than
sourcing from external suppliers.

The groups profit increases by $1.5 million if the European supply chips to America for
the price of $95. This is a significant increase which will benefit the organisation.

Transfer Pricing

A good transfer price is one where both divisions are happy with and it doesnt impact
the group as a whole in a negative way. This usually means that divisions buy and sell
internally and do not source goods from outside the group if they can buy them internally.

The issue of supplying chips internally is what price to transfer them at. Here the needs
of the both organisations require careful consideration as not to de-motivate the
managers.

The American subsidiary is currently buying externally at $90 and this is effectively the
maximum they will pay. So the European can only really charge a maximum transfer
price of $90. The minimum price the Europeans could charge is $85 {part b(i)}, so a
range from $85 to $90 exists. However if the managers performances are going to be
measured based on returns, then the minimum price the European will want to charge is
$92 and this would clearly not be acceptable to the Americans.

From the groups perspective the transfer price is based on opportunity cost approach (or
relevant costing) to ensure goal congruence

Minimum price to a seller (the European subsidiary)

Marginal cost of each unit sold normally if seller is not at full capacity or no
intermediate market exists
Marginal cost + opportunity cost e.g. normally the external market price for the
product sold if an intermediate market exists or the division is at full capacity

Maximum price for a buyer (the American subsidiary)

Lower of

Market price the buyer could obtain the product from elsewhere
Market price the buyer sells the product for, less any further costs the buyer has
before resale of the goods

271 | P a g e
What the above method will do is to set a range; so long as it is profitable to do so for the
group e.g. a range actually exists, any price within this range once set will then ensure
goal congruence e.g. seller and buyer operate in the best interests of the group as a whole.

The actual transfer price within this range would normally have to be found by politics
and compromise between the two managers.

Performance measures

Setting performance measures based on profit will encourage goal congruence. The
transfer price is $85 and this would be acceptable to both parties. This would also result
in higher profits for the group.

If the return on assets is used as a performance measure the transfer price will be too high
for the American subsidiary to accept $92 and this will result in goal incongruence. This
means chips will be sourced from outside the group resulting in lower profits.

A single performance measure is not an ideal way to measure performance of divisions as


this could result in short term decisions being taken which do not benefit in the long term.
A range of performance measures with both financial and non financial aspects is better
in the long term for the group.

Conclusion

The European subsidiary should use its spare capacity and sell chips to the American
subsidiary at $85. However the loss of external sales of 100,000 units by the European
could have a negative impact on future sales. Increasing capacity will help increase
profits.

A range of performance measures should be used to encourage goal congruence and long
term success of the group.

Part (d)

Different countries have varying tax rates. A multi-national company can set up
subsidiaries in different countries to take advantage of these tax rates through transfer
pricing to reduce their overall tax charge.

Where there are high tax rates in a country, goods can be supplied to that subsidiary at a
high transfer price to reduce profits, from a subsidiary in a low tax rate country. The idea
is to increase profits in lower tax rate country and decrease profits in a higher tax rate
country. This can be done through transfer pricing.

272 | P a g e
National tax authorities thoroughly investigate accounts of these types of companies to
ensure that a market price has been used and that the profits have not been manipulated
by transfer pricing to reduce tax payments. Fines and penalties can be imposed on
organisations who deliberately try to manipulate profits by the improper use of transfer
pricing.

D2 5 Perfumes and cosmetics (CIMA P2 May 2010)

Part (a) (i)

Return on capital employed (ROCE)

= Profit before interest and tax (PBIT) x 100%


Capital employed

ROCE is also referred to as return on investment (ROI) and return on net assets (RONA).
ROCE measures profitability and shows how well the business is utilising its capital to
generate profits.

Process Profit & loss Capital employed Monthly ROI Annualised ROI
$ $ % %
5
B 18,800 800,000 x 0.8 = 262,144 18,800 / 262,144 = 7 7 x 12 = 84
2
C (15,550) 500,000 x 0.8 = 320,000 (15,550) / 320,000 = (5) (5) x 12 = (60)
10
D (5,000) 300,000 x 0.8 = 32,212 (5,000) / 32,212 = (15) (15) x 12 = (180)

Part (a) (ii)

Capital equipment bought in each division has been purchased at different points in time
compared to other divisions; therefore it means that the ROI comparison between
divisions will be inconsistent. This is due to the fact that the price paid will be affected by
inflation and in addition depreciable years will be different between divisions.
Some of the costs are not within the control of the divisional managers namely head
office costs which are apportioned arbitrarily; therefore it would be unfair to measure
performance based on these divisional profits.

Transfer prices are being used between divisions which are a mark up on the total costs of
the process. This is unfair as the receiving division has no control over the costs of the
supplying division on which the transfer price calculated. The receiving divisions
performance will be affected by the costs of the supplying division.

273 | P a g e
Division C and D have fixed costs occurring regardless of the volume received from
division B, therefore the managers of C and D have little control over their activity and in
addition they are highly geared and so dependent on high levels of activity in order to
cover their fixed costs.

Part (b) (i)

If investment was undertaken:

Extra income generated 1,500 litres $30,000


from abnormal losses at $20 per litre
Abnormal sales forgone ($750)
1,500 litres x $0.50
Increase in depreciation (W1) ($11,333)
$16,666 - $5,333
Loss in 2010 ($15,550)
Profit $2,367

ROI = ($2,367 / $1,000,000) x 100% = 0.24%


Annualised ROI = 0.24% x 12 = 2.88%

If investment was not undertaken:

Fall in depreciation (W1) $1,066


$5,333 - $4,267
Loss in 2010 ($15,550)
Loss ($14,484)

ROI = [($14,484) / $256,000] x 100% = (5.66%)


Annualised ROI = (5.66%) x 12 = (67.89%)

Workings

(W1) Depreciation

Year NBV b/f Deprecation NBV c/f


(20% reducing balance)
$ $ $
2008 500,000 100,000 400,000
2009 400,000 80,000 320,000
2010 320,000 64,000 256,000
2011 256,000 51,200

274 | P a g e
Monthly depreciation charge on old machine in 2010
= $64,000 / 12 = $5,333
Monthly depreciation charge on old machine in 2011
= $51,200 / 12 = $4,267
Monthly depreciation charge on new machine in 2011
= ($1,000,000 x 0.2) / 12 = $16,666

Part (b) (ii)

This investment decision shows a small increase in ROI of 2.88% per annum and should
be accepted if the increase is adequate. There maybe objections from division C as the
manager may feel that this is not enough and there is a good chance it may cause a fall in
ROI if the figures used are not accurate. The choice should be based on using NPV based
on the companys cost of capital. If it is positive then the investment should be accepted.

Using ROI the profits can be manipulated because it includes some non-cash based items
such as depreciation. For example the depreciation rate could be decreased to increase the
profit and thereby increasing the ROI. NPV uses relevant costing and so only includes
those cash flows that will change as a result of the investment decision and so there is
less scope for manipulation.

ROI ignores the time value of money as future cash flows are worth less today and that
they should be discounted to reflect this. NPV takes account of the time value of money
as it applies discounting to future cash flows to give them same real value if they were
received today.

Part (c)

Manager of division B

The manager of B would be in favour of the current transfer pricing policy as all of his
output is transferred to C or D and furthermore a by-product is created (due to this
process) which is sold externally. All of his costs are covered and receives a profit
through the transfer price which is budgeted cost plus 15% mark up. The problem with
this is that there is no incentive for manager B to control any inefficiencies in his process
as he knows that all costs will be paid by the transfer pricing policy, furthermore, if there
are any efficiencies made he will be rewarded for it.

Manager of division D

The manager of D can buy the material D externally for $7.50 per litre or pay $9.20 per
litre internally from B. It is easy to see that the manger of D is going to purchase
externally as it is cheaper than buying from B, and so the manger of D is not in favour of
the current transfer pricing policy.

275 | P a g e
Group perspective

It is important that divisions transfer material between them as far as possible otherwise
there maybe materials that the group make which are not used internally but instead the
same items are being bought externally at a high cost, leading extra unnecessary costs.
Clearly the manager of D needs to be encouraged not to make the sub-optimal decision of
purchasing material externally when it is being made by division B. Furthermore, the
material D being produced by B is part of a joint process and so in any event will be
made and so if it is not transferred to division D then it will be wasted. There would be no
change to the costs of process B and external costs spent by the group will rise.

The transfer price between division B and division D should at least match the market
price of $7.50 and so division D would be encouraged to accept the transfers from
division B. One way to do this is perhaps by reducing the costs being apportioned to the
manufacture of material B and so therefore division B can charge a lower price to
division D.

A dual pricing approach can be adopted to help in this situation, as this would mean we
give the transfer price to each of the divisions that would want in order to agree to
transfer. For example we allow division B to transfer at $9.50 but give a transfer price of
$7.50 to division D. This would encourage the transfer and the difference in the prices
can be resolved though head office re-apportioning it as charges to all divisions.

D2 6 SWZ (CIMA P2 Nov 2010)

Part (a)

Return on capital employed (ROCE)

= Profit before interest and tax (PBIT) x 100%


Capital employed

The ROI for the three years:


Year ROI
$
2008 40 / 400 = 10%
2009 56 / 320 = 17.5%
2010 62 / 256 = 24.2%

276 | P a g e
There is a very good improvement of the ROI over the last 3 years from 10% up to nearly
25%. Inflation has been removed according to the question and the figures the same for
turnover and cost of sales throughout the 3 years which means there has been no increase
in products sold over the last 3 years. The gross profit has been constant at 40% and
therefore indicating no change in quantity sold and prices. Operating costs appear to be
falling over the last three years, but if depreciation is removed it shows that operation
costs have increased by $4,000 in 2009 and more significantly in 2010.

In conclusion the improvement in ROI over the last three years is largely down to the
depreciation policy rather than improved performance by the division.

Part (b)

The investment results in a positive NPV of $24,536 and so from a group company
perspective it will be accepted by head office to go ahead, however from the perspective
of the manger of S division it will depend on whether or not ROI of the division is
improved. This is because his performance is assessed on ROI. To assess this we will
look at the divisional ROI if the investment is undertaken compared to the divisional ROI
if the investment is not undertaken.

If investment was undertaken: $


Sales 400,000
Cost of sales ($240,000 x 90%) 216,000
Gross profit 184,000
Other operating costs (W1) (97,200)
Pre-tax operating profit 86,800
Capital invested at the end of the year (W2) = $252,800

ROI = ($86,800 / $252,800) x 100% = 34.3%

If investment was not undertaken: $


Sales 400,000
Cost of sales 240,000
Gross profit 160,000
Other operating costs (W3) (85,200)
Pre-tax operating profit 74,800

Capital invested at the end of the year ($256,000 - $51,200) = $204,800

ROI = ($74,800 / $204,800) x 100% = 36.5%

277 | P a g e
Workings

(W1) Other operating costs


$
Other operating costs 98,000
Less: Depreciation ($320,000 x 20%) (64,000)
Add: New depreciation (W2) 63,200
Total 97,200

(W2) New depreciation and capital invested at the end of the year
$
Current total NBV at the end of 2010 256,000
Less: NBV of machine sold (40,000)
Add: Cost of new machine 100,000
316,000
Less: New depreciation (316,000 x 20%) (63,200)
Revised total NBV at the end of 2011 252,800

(W3) Other operating costs


$
Other operating costs 98,000
Less: Old depreciation ($320,000 x 20%) (64,000)
Add: New depreciation ($256,000 x 20%) 51,200
Total 85,200

Part (c)

Profit before interest and tax (PBIT) X


Capital employed x head office % interest charge (X)
Residual Income X

If investment was undertaken: $


Pre-tax operating profits 86,800
Notional capital charge ($252,800 x 8%) (20,220)
Residual Income 66,580

If investment was not undertaken: $


Pre-tax operating profits 74,800
Notional capital charge ($204,800 x 8%) (16,380)
Residual Income 58,420

Difference is an increase of Residual Income if investment undertaken of $8,160

278 | P a g e
D2 7 DE company (CIMA P2 May 2011)

Part (a)

Division E could have sold a further 42,000 components but was constrained by capacity
and had to transfer 70,000 components internally to division D. Division E therefore lost
out on achieving the market price for 42,000 components, and therefore the transfer price
on these to division D should be market price. The rest of the components should be at
variable cost.

Analysis
Internal Internal External Total
at variable cost at market price
Number 28,000 42,000 70,000 140,000
of components
$000 $000 $000 $000
Variable cost 28,000 42,000 70,000 140,000
Sales value 28,000 65,100 108,500 201,600

Part (b)

External demand is very important as it determines what profits are accrued to divisions
D and E internally. Division E has sold 28,000 components at variable cost to division D.
It could have sold them externally at $1550 and could have made a profit instead of
making no profits through the internal transfer. If division D had bought externally rather
than internally then it would have had to spend $43.4m to obtain the 28,000 components
transferred to them at variable costs. This is an extra $15.4m.

Division D appears to enjoy higher profits because of the items transferred at variable
cost, which may not be fair as division E is receiving no recognition for the manufacture
of these items. There should be some benefit accrued to division E and maybe not full
market price as it cannot sell these components externally. For example if the profits
were shared equally between the divisions each division would have a profit of $7.7m.

If the market demand increased for components then the transfer price charged to
division D would increase and thereby shifting more of the profits to division E. If market
demand fell for the components then the transfer priced charged to division D would fall
and thereby shifting more of the profits to division D.

Part (c) (i)

Es capacity will increase by 10% and variable costs will fall by 20% if the new
investment is undertaken, however this benefit is reduced because of the internal transfer
policy.

279 | P a g e
Any increase in capacity for division E will have the effect of increasing external sales
but at the same time reducing the volume of forgone external sales when transferring
components to division D. Therefore there is no additional financial benefit for division
E.

Half of Es components are sold to D and 28,000 of these are transferred at variable costs.
Cost savings are therefore transferred to D because of the transfer pricing policy. E will
only enjoy cost savings on those items which are sold at market value.

Division E benefits:

VC of items sold at market value = $140m x 80% = $112m per annum

20% cost savings every year = $112m x 20% = $22.4m

PV of cost savings over 5 years at 8% cost of capital = $22.4m x 3.993 = $89.4m

The cost of the investment is $120m with no residual value and the cost saving from
investment is going to only be $89.4m, therefore it is not financially viable from division
Es perspective.

Part (c) (ii)

We need to compare the original situation to the revised situation from a group
perspective. We already know the benefits that would accrue to division E if the
investment was undertaken (see part (i) above) and therefore we now need to do the same
for division D.

Revised transfer value for division D:


42,000 components original cost of $42m x 80% = $33.6m
28,000 components market value of $1,550 per unit = $43.4m
Total $77.0m

Original transfer value for division D = $28m + $65.1m = $93.1m

Saving = $93.1 - $77.0m = $16.1m

Total saving = $22.4m + $16.1m = $38.5m

PV = $38.5 x 3.993 = $153.73m

From a group perspective the investment of $120m is beneficial because the PV of


$153.73m is greater than this cost.

280 | P a g e
Part (d)

It should be fair and consider those items that can be controlled by the manager.

It should simple to understand and easy to calculate so the impact of decisions taken by
managers can be seen and measured.

281 | P a g e

También podría gustarte